Главная Юзердоски Каталог Трекер NSFW Настройки

Космос и астрономия

Ответить в тред Ответить в тред
Check this out!
<<
Назад | Вниз | Каталог | Обновить | Автообновление | 641 125 136
Тред Тупых Вопросов №169 Szapka Edition [ТТВ 169] Аноним 01/12/22 Чтв 21:26:43 735673 1
Marsnorthpole12[...].jpg 501Кб, 1280x960
1280x960
1 oh8ttMecdCIs7[...].png 191Кб, 574x547
574x547
968b78dbfecfc58[...].png 1026Кб, 1024x889
1024x889
JupiterPolarAur[...].jpg 1165Кб, 2224x2216
2224x2216
Тред вопросов о жизни, Вселенной и всём таком.

Спрашиваем то, за что в других местах выдают путёвку в биореактор. Здесь анонимные учёные мирового уровня критически рассмотрят любые гениальные идеи и нарисованные в Paint схемы.

Q: Можно быстрее?
A: Быстрее до Антарктиды раком.

Q: Почему бы не привязать ракету к воздушному шару или стартовать с горы?
A: Космос - это не как высоко, а как быстро, большая часть энергии ракеты уходит на разгон вбок.
Подробнее тут https://what-if.xkcd.com/58/ (английский) https://chtoes.li/orbital-speed/ (перевод)

Q: В космосе горячо или холодно?
A: В космосе почти вакуум. То разреженное вещество, которое там есть, очень горячее, но на объекты в космосе оно почти не влияет. Влияет излучение. В тени будешь остывать, на солнце нагреваться.

Q: Есть там кто?
A: Не исключено, что нет: https://www.youtube.com/watch?v=PqEmYU8Y_rI
A: Не исключено, что есть: https://www.youtube.com/watch?v=r6rPNPVQp0Y

Q: Я начитался охуительных историй про уфологию, че делать, нам жопа?
A: Да, тебе жопа, можешь сгонять в зогач или куда оттуда пошлют.

Q: Если два корабля летят со скоростью 0.8с навстречу друг другу, то их скорость сближения будет 1.6с АЗАЗАЗЗАЗ Я НАЕБАЛ ИНШТЕЙНА))
A: Не будет, для релятивистских скоростей нужно использовать преобразования Лоренца. Вот хороший цикл коротких видео по СТО, есть англ и ру сабы: https://www.youtube.com/watch?v=1rLWVZVWfdY

Прошлый тред простудился без шапки: >>733086 (OP)
Аноним 01/12/22 Чтв 21:32:05 735676 2
>>735673 (OP)
>Можно быстрее?
Я слышал, что ученые уже почти надули пузырь Альбукерке, это правда?
Аноним 01/12/22 Чтв 21:52:51 735678 3
>>735676
Ебанный стыд...
Во-первых, Алькубьерре.
Во-вторых, не упасть, а создавать вокруг корабля изнутри (иначе кина не будет).
В-третьих, НАСА искривляет пространство на десятимиллионную часть, контролируя это сверхточными интерферометрами, до самого варп-привода здесь - как до Антарктиды раком.
Аноним 01/12/22 Чтв 21:55:26 735680 4
если телепортация возможно как в кино, то разве она будет мгновенной? По идее она должна ограничиваться скоростью света. Иначе это уже будет нарушать принцип причинности. А раз так то нахуй она тогда такая нужна? Телепортнулся до андромеды и назад - а тут уже 5 млн лет прошло. Или же телепортация в принципе невозможна в нашей реальности.
Аноним 01/12/22 Чтв 21:56:55 735681 5
>>735680
хуй знает как у тебя телепортация нарушает причинность
напротив, любое движение выше скорости света будет по сути телепортацией для наблюдателя
Аноним 01/12/22 Чтв 21:59:38 735682 6
>>735680
Ну собственно да, не очень понятно, в чём твой вопрос. Именно поэтому считается, что телепортация невозможна.
Аноним 01/12/22 Чтв 22:08:24 735683 7
>>735681
Любое распространение взаимодействия выше скорости света нарушает причинность. Если событие в точке А повлияло на событие в точке Б быстрее, чем от А до Б мог бы дойти сигнал со скоростью света, то можно выбрать систему отсчёта, в которой события А и Б произошли в одной точке, причём А (которое повлияло на Б) позже, чем Б.
Аноним 01/12/22 Чтв 22:09:49 735684 8
>>735681
>хуй знает как у тебя телепортация нарушает причинность
Сверхсвет нарушает причинность.
Аноним 01/12/22 Чтв 22:24:30 735689 9
>>735684
>>735683
вы немного не туда смотрите
в контексте сверхсвет-лимита телепортация это не движение
вообще на телепортацию нужно смотреть с точки зрения единой системы отсчета
да, это хуета и кринж, но только потому что телепортации нет и не предвидится, никто всерьез не рассматривает
Аноним 01/12/22 Чтв 22:27:35 735690 10
Вообще, приведите пример, когда телепортация реально нарушает причинность, а не просто нууу быстрее света значит нельзя
Аноним 01/12/22 Чтв 22:30:20 735691 11
>>735689
>в контексте сверхсвет-лимита телепортация это не движение
Вообще без разницы. Если есть факт того, что ты смог из одной точки повлиять на другую быстрее, чем туда свет долетел, то ты нарушаешь причинность.

>вообще на телепортацию нужно смотреть с точки зрения единой системы отсчета
Какой ещё единой системы отсчёта?

>да, это хуета и кринж, но только потому что телепортации нет и не предвидится, никто всерьез не рассматривает
Именно по той причине, что телепортация нарушает причинность, её всерьёз и не рассматривают.
Аноним 01/12/22 Чтв 22:31:00 735692 12
>>735690
Что значит "раельно"? Реально телепортацию тебе никто не продемонстрирует.
Аноним 01/12/22 Чтв 22:42:43 735695 13
Download.jpg 7Кб, 355x142
355x142
Парни, как съебать с этой планеты в скором времени до ядерного аппокалипсиса?

Есть ли хоть какой-то шанс на это у простого горожанина?
Аноним 01/12/22 Чтв 22:43:47 735696 14
image.png 7Кб, 395x302
395x302
>>735690
Просто так на пальцах в посте на двачике не объяснить, тема чрезвычайно контринтуитивная. Даже я, интеллектуальный монстр и гений, долго вдуплял в этот вопрос. Но то сложно было на словах.
Проще всего понимать с диаграммами Пенроуза, то объяснение дошло весьма быстро, всего с пары страниц.
Аноним 01/12/22 Чтв 22:44:32 735698 15
>>735695
>Парни, как съебать с этой планеты в скором времени
Куда ты съебывать собрался? Только на орбиту летаем.
>Есть ли хоть какой-то шанс на это у простого горожанина?
Нет.
Аноним 01/12/22 Чтв 22:45:52 735700 16
>>735691
ты кукбой, сойжак или кукож?
Аноним 01/12/22 Чтв 22:48:10 735702 17
>>735698
>Куда ты съебывать собрался?
Я бы пожил на Луне годик. Интересно пожить при низкой гравитации и не обосраться от этого. Наверное мог бы копать или плавить чего-нить. Или гайки крутить. Или менять сгоревшие платы в каком либо оборудовании.

Вот, кстати, интересно - а если плавательный бассейн зафигачить на Луне (не на поверхности, конечно), там нырнуть можно будет? Или как поплавок буду плавать?
Аноним 01/12/22 Чтв 22:52:26 735703 18
>>735702
>Я бы пожил на Луне годик.
Там негде жить.
>Интересно пожить при низкой гравитации и не обосраться от этого.
Обосрешься по возвращению или спустя некоторое время когда кости сами по себе потрескаются.
>Наверное мог бы копать или плавить чего-нить. Или гайки крутить. Или менять сгоревшие платы в каком либо оборудовании.
Выгоднее роботв заслать чем тебя.
>Вот, кстати, интересно - а если плавательный бассейн зафигачить на Луне (не на поверхности, конечно), там нырнуть можно будет?
Можно будет.
> Или как поплавок буду плавать?
Будешь плавать абсолютно так же как на сраной земляшке.
Речь исключительно о плавучести, а не о поведении воды и ощущениях при плавании в низкой гравитации.
Аноним 01/12/22 Чтв 22:52:36 735704 19
>>735690
Бля, ну вот и для кого я в шапку ссылки пихал?
https://www.youtube.com/watch?v=SrNVsfkGW-0&list=PLoaVOjvkzQtyjhV55wZcdicAz5KexgKvm&index=4
Вот на 0:56 пример двух коробок, которые загораются одновременно с точки зрения оранжевого чувака, а с точки зрения синего чувака правая загорается раньше левой. Представим, что у нас возможна телепортация, и с точки зрения оранжевого левая коробка подожгла правую через телепортатор. А с точки зрения синего получается, что сначала загорелась правая коробка, а потом левая её подожгла (о чём синий чувак может мгновенно узнать с помощью телепортатора).
Аноним 01/12/22 Чтв 22:56:09 735705 20
>>735702
>Наверное мог бы копать или плавить чего-нить. Или гайки крутить. Или менять сгоревшие платы в каком либо оборудовании.
Это очень многие могут. Туда сначала будут брать только оверквалифицированных уберлюдей, но если базу начнут пилить, то требования ослабят (чуть-чуть, ты всё равно не подойдёшь).

>Вот, кстати, интересно - а если плавательный бассейн зафигачить на Луне (не на поверхности, конечно), там нырнуть можно будет? Или как поплавок буду плавать?
Можно будет нырнуть, вообще плавание будет ощущаться так же, как на Земле, потому что твой вес меньше, но и вес вытесняемой тобой воды во столько же раз меньше. Зато там ещё можно будет выпрыгивать из воды аки дельфин.
Аноним 01/12/22 Чтв 23:02:24 735707 21
>>735703
>Там негде жить.
На орбите тоже негде жить, но МКС обитаема.

>Обосрешься по возвращению или спустя некоторое время когда кости сами по себе потрескаются.
Российский космонавт Валерий Поляков провел почти 438 дней подряд на борту космической станции «Мир» с 8 января 1994 года по 22 марта 1995 года. Это самое длительное постоянное пребывание в космосе, которое вот уже почти три десятилетия никто не может преодолеть. Как отмечают в Роскосмосе, исследования Полякова помогли доказать, что человеческий организм готов не только к полетам на околоземную орбиту, но и к миссиям в дальний космос.

>Выгоднее роботв заслать чем тебя.
Ха. Так я только за. Давно жду, когда на Луну забросят роботов.
Пусть готовят плацдарм для человеков.
Аноним 01/12/22 Чтв 23:09:47 735712 22
>>735707
>На орбите тоже негде жить, но МКС обитаема.
На орбите есть МКС и Тяньгун, на Луне нет ничего.
>Валерий Поляков
Ну и где он теперь?
>Ха. Так я только за. Давно жду, когда на Луну забросят роботов.
А что сделал ТЫ чтобы на Луне были роботы?
Аноним 01/12/22 Чтв 23:18:37 735716 23
>>735712
>На орбите есть МКС и Тяньгун, на Луне нет ничего.
Ну как бы рано или поздно аналог МКС на Луне таки появится. Иначе зачем всё это. Я имею в виду жизнь. Иначе у жизни, как явления, нет перспектив.

>>Валерий Поляков
>Ну и где он теперь?
Дата рождения27 апреля 1942
Место рожденияТула, РСФСР, СССР
Дата смерти7 сентября 2022 (80 лет)

Нелохо. Весьма неплохо.

>А что сделал ТЫ чтобы на Луне были роботы?
Сделал почти ничего. Спроси что ДЕЛАЛ и может быть я даже деанонимизируюсь. Но это неточно.
Аноним 01/12/22 Чтв 23:22:00 735717 24
InstantaneousSi[...].gif 44Кб, 594x751
594x751
>>735696
То есть нужно упростить весь мир до диаграмм Пенроуза?
Аноним 01/12/22 Чтв 23:27:36 735718 25
FeWiPMWAAI7OYr.jpg 187Кб, 1218x1015
1218x1015
Бля.
Вы не понимаете что такое телепортация.
Это не движение в привычном смысле и даже не в релятивистском.
Это перемещение из одной системы отсчета в другую, заодно с физическим местоположением это вторично в нашем контексте
Например, есть объекты на 1 световом часу друг от друга, пусть Земля и Сатурн, вроде подходит.
Если я телепортируюсь на Сатурн, я попаду не то состояние что я вижу с Земли, а на Сатурн в его системе отсчета
Если я буду смотреть на Землю - я увижу себя, да. Но это не будет нарушением причинности, так как если я телепортируюсь обратно якобы к себе - я перемещусь в систему отсчета Земли, где я телепортировался какое-то время назад и нет никакой хуйни с копиями и причинностью, какие обычно подразумевают при перемещении быстрее с.
Аноним 01/12/22 Чтв 23:28:26 735719 26
Ну можно допустим заниматься спортом или построить центрефугу для слабой гравитации. Солнце есть, ставь солнечные и питай центрефугу.

По указания великого вождя лунного моря, в центрефуге нужно будет проводить минимум 3 часа с день и потом показать qr-code. Иначе твоя виза на Землю анулируется навсегда.
Аноним 02/12/22 Птн 02:43:55 735734 27
>>735719
Чел, ты кому отвечаешь, на какой пост?
Аноним 02/12/22 Птн 05:34:01 735738 28
image.png 364Кб, 602x377
602x377
>>735716
>Ну как бы рано или поздно аналог МКС на Луне таки появится. Иначе зачем всё это. Я имею в виду жизнь. Иначе у жизни, как явления, нет перспектив.
Разделяю твою тягу к развитию, но Гейтвей пока только в планах, и пока его не запилят и не начнут летать еще хуй знает что произойдет. СЛС-то с какой болью полетел спустя столько лет, что еще с этим проектом будет.
>Нелохо. Весьма неплохо.
Вот, а не летал бы, может и жив был бы!
Вопрос на самом деле такой - что у него со здоровьем было, как он прилетал и его выгружали, сколько времени на реабилитацию уходило.
После долгих полетов, насколько я в курсе, огурцы сами ходить особо не могут.
>Сделал почти ничего. Спроси что ДЕЛАЛ и может быть я даже деанонимизируюсь. Но это неточно.
Не буду спрашивать, сам такой.
Аноним 02/12/22 Птн 05:36:58 735739 29
>>735718
Ты думаешь в классической механике со статичными не-релятивистскими системами отсчета и потому у тебя не ломается относительность одновременности. Добавь в свои телепортирующиеся схемы курьеров на релятивистских скоростях и все сломается >>735717
Для понимания этого аспекта - да, имеет смысл упростить модель до диаграмм Пенроуза.
Аноним 02/12/22 Птн 06:49:42 735742 30
image.png 242Кб, 800x289
800x289
Бамп вопросу.
>>735422 →
>Почему после того как с мороза заходишь в помещение очки запотевают понятно - на холодной поверхности конденсируется вода потому что холодный воздух около очков не может столько влаги удерживать.
>Вопрос в том - почему когда из помещения выходишь на мороз - конденсат пропадает? Вода осела на холодную поверхность, с чего бы ей испаряться если вокруг холодно?
>Чтобы вопрос был релевантным космосу спрошу про телескопы - как с влажностью и конденсацией в обсерваториях борятся?
Аноним 02/12/22 Птн 09:20:54 735746 31
>>735739
>Добавь в свои телепортирующиеся схемы курьеров на релятивистских скоростях и все сломается
Нет, потому что происходит перемещение между системами отсчета
А так как движение выше с именно движение все еще невозможно, мы получаем просто коэффицент от 0 до 1, что никак не меняет причинность
Аноним 02/12/22 Птн 10:59:48 735751 32
>>735742
а схуяль испарение должно прекратиться? Оно просто замедляется, но зато все, что испарилось, перестает конденсироваться обратно. Воды там оседает хуй да нихуя насамделе.

в обсерваториях чо только не делают. В основном оптику либо термостатируют, либо медленно подводят к окружающей температуре. Могут осушенным воздухом обдувать.
Аноним 02/12/22 Птн 11:15:00 735752 33
>>735704
у меня к таким примерам основной вопрос - ну и чо? События происходят в своем порядке, мимокрокодил пусть их видит в любом другом - что это поменяет?

к примеру, винтовочная пуля сначала уебет челу в жопу, а только потом он услышит выстрел - ряяяя, причинность нарушена! Хотя по сути та же хуйня, материальное тело перемещается быстрее воспринимаемой информации.
Аноним 02/12/22 Птн 11:25:43 735753 34
>>735636 →
>Какое предназначение у Вселенной?
Никакого. "Предназначения" нет ни у чего.
>Или жизнь это просто побочный эффект
Просто побочный эффект.
>>735676
Нет, это пиздеж.
>>735680
>если телепортация возможно как в кино
Невозможна.
Аноним 02/12/22 Птн 11:32:36 735754 35
>>735752
Мимокрокодил видит, как загорелась коробка, и используя телепортатор тушит другую коробку до того, как она успеет зажечь первую.
Аноним 02/12/22 Птн 11:37:59 735755 36
>>735754
он думает: ебать я какой умный, ща я потушу вторую коробку и создам вселенский парадокс! Телепортируется к ней и видит, что она уже давно сгорела кхуям, просто наблюдать сей факт в телескоп из его родных галактических пердей можно будет только через стопицот тысяч лет.
Аноним 02/12/22 Птн 12:01:40 735763 37
>>735695
>до ядерного аппокалипсиса
Даже самое подходящее место - Марс, по условиям гораздо хуже чем Земля после ядерного аппокалипсиса.
Аноним 02/12/22 Птн 12:15:53 735765 38
>>735752
Пример с звуком это не подходит, так как перемещение пули не связано с звуковыми волнами
Информация это сама пуля тут, а не звук выстрела
В контексте движения скорость света предельна, да, так как информацию об этом движении не получится передать быстрее скорости света, следовательно эту скорость никак не превысить
Аноним 02/12/22 Птн 12:17:26 735766 39
>>735751
>а схуяль испарение должно прекратиться?
Так вся суть была что из-за холода вода не может в воздухе держаться и конденсируется. Ты выходишь на мороз и в итоге еще холоднее же.
>Оно просто замедляется, но зато все, что испарилось, перестает конденсироваться обратно. Воды там оседает хуй да нихуя насамделе.
Почему оно перестает конденсироваться понятно, вопрос почему оно быстро испаряется на морозе.
>В основном оптику либо термостатируют, либо медленно подводят к окружающей температуре. Могут осушенным воздухом обдувать.
Внутри телескопов замкнутая атмосфера, какой-нибудь азот или вакуум пониженное давление, или там такой же воздух что и снаружи?
Аноним 02/12/22 Птн 12:24:36 735769 40
>>735765
ну так с мгновенным телепортом у нас тоже перемещение перестает быть связанным со светом, который становится эдакой вспомогательной хуитой.
Аноним 02/12/22 Птн 12:29:12 735770 41
>>735769
Это так
Но так как мы не знаем как работает телепорт и каким образом он передает информацию - мы не можем на полном серьезе рассуждать о свойствах телепортации
Аноним 02/12/22 Птн 12:34:53 735771 42
>>735766
>Ты выходишь на мороз и в итоге еще холоднее же.
погугли чо такое точка росы.
>почему оно быстро испаряется на морозе
быстро потому, что "его" мало. Еще раз - испарение никуда не исчезает. Мамка у тебя никогда на морозе белье после стирки не сушила?
>или там такой же воздух что и снаружи
в целом обычно да. Все серьезные оптические телескопы - зеркальные, там охуеешь замкнутую среду создавать. Могут, опять же, продувать приповерхностный слой чем-нибудь.
Аноним 02/12/22 Птн 12:38:20 735772 43
>>735770
ну так пока что рассуждение не о свойствах, а о последствиях. И как-то так получается, что грозное вселенское нарушение причинности (которое преподносится некоторыми как одна зи базовых причин невозможности телепортации) выливается только в то, что блять Боб увидит какую-нибудь хуйню позже, чем Алиса.
Аноним 02/12/22 Птн 13:04:27 735774 44
>>735772
База.
Но мочоные поступиться своим положением и врут.
Аноним 02/12/22 Птн 13:05:32 735775 45
>>735774
>сукаблят
>Но мочоные не могут поступиться своим положением и орут.
Аноним 02/12/22 Птн 13:48:34 735777 46
>>735673 (OP)
Колонизация марса не как земля 2.0
а как аутпост земли на случай пиздеца на земле


вот есть сай фай или не сай фай на тему как там можно содержать колонию и чтобы она была 100%self sustained? все медицаменты деллаись там, стекли, трубки ИВЛ и кондиционеров, хладаген и фреон, топилво для ракет, металлы и ткани. Каким образом?
Аноним 02/12/22 Птн 13:51:15 735778 47
68562511.png 110Кб, 500x500
500x500
За все миллионы лет существования многоклеточной жизни атмосферу должны были основательно пропердеть, но куда тогда делся весь метан, если его, в отличие от углекислого газа, не усваивают ни растения ни животные? Кто спиздил наш халявный газ? Инопланетяне?
Аноним 02/12/22 Птн 13:56:59 735779 48
>>735772
>>735775
>>735774
Бля, суть в том что такое телепортация. Это даже не гипотетическая хуйня типа двигателя на антиматерии с 0,99с, а полностью фентезийный концепт.
Поэтому ее нельзя всерьез ввести в более-менее серьезные обсуждения перемещений быстрее света.
И еще, саму информацию для телепортации нужно как-то передавать.
Аноним 02/12/22 Птн 13:58:14 735780 49
>>735778
некоторые бактерии в почве таки жрут метан, гугли "метанотрофы".
алсо в атмосфере он потихоньку реагирует с кислородом, превращаясь в плесень и липовый мед воду и углекислый газ.
Аноним 02/12/22 Птн 14:00:45 735781 50
>>735777
Очевидно, что нужен полный цикл промышленности. То есть, нужно повторять все то, что есть на Земле. Разумеется, пропорции разных индустрий будут отличаться от наших, но в любом случае нужен полный цикл от добычи руды до финального продукта. И, самое главное, производство оборудования для всего этого.
Аноним 02/12/22 Птн 14:03:14 735782 51
>>735779
>Поэтому ее нельзя всерьез ввести в более-менее серьезные обсуждения перемещений быстрее света.
чойта нельзя?
Аноним 02/12/22 Птн 14:05:53 735783 52
>>735782
Эммм, потому что для нет никакого хоть немного правдоподобного теоретического обоснования?
И еще потому что даже если мы обоснуем телепортацию, остается обосновать передачу информации в место телепортации, ведь без этого телепортация не будет быстрее света.
Аноним 02/12/22 Птн 14:13:27 735785 53
>>735771
>погугли чо такое точка росы.
Я погуглил и это вроде примерно то самое про что я говорю - когда воздух настолько насыщенный что вода конденсируется, т.к. пар не может быть в воздухе уже.
>Еще раз - испарение никуда не исчезает.
А почему в тепле оно не происходит, а на холоде происходит?
Вот вошел в тепло - возле холодных линз воздух холодный, влажность не может держаться и конденсируется. Вышел на холод - воздух такой же холодный или даже холоднее чем на линзах, ведь они близко к лицу, но роса/иней не остаются.
>Мамка у тебя никогда на морозе белье после стирки не сушила?
Сушила, охуенно пахло свежестью. Помню как у пятилетнего (или сколько мне там было) меня шаблон порвался когда впервые увидел замороженное белье.
Аноним 02/12/22 Птн 14:15:26 735787 54
image.png 136Кб, 350x285
350x285
image.png 40Кб, 225x225
225x225
image.png 1249Кб, 1553x831
1553x831
Аноним 02/12/22 Птн 14:19:24 735788 55
image.png 4Кб, 731x39
731x39
Какая самая большая (по модулю) степень использующаяся в реальных исчислениях?
https://en.wikipedia.org/wiki/Eighth_power тут сказано что эффект Казимира Малевича работает в (минус) восьмой степени, но я не вижу этого в уравнениях самого малевича, кстати.
Аноним 02/12/22 Птн 14:20:49 735789 56
>>735785
>А почему в тепле оно не происходит, а на холоде происходит?
Возможно потому что холодный воздух на улице сухой а теплый в помещении - влажный.
Аноним 02/12/22 Птн 14:24:05 735790 57
image.png 32Кб, 200x133
200x133
>>735789
Сука, я дебил, бля.
Аноним 02/12/22 Птн 14:24:21 735791 58
image.png 244Кб, 823x818
823x818
>>735787
Это искривление пространства в первую очередь
Ты не перемещаешься быстрее света по факту, как показывают все эти мемные объяснения чревоточин
Но с точки зрения наблюдателей на сторонах чревоточин 1 и 2 - это будет телепортацией. Как и другие "общепринятые" способы сверхсвета, типа альбукерке.
И по логике причинность-шизов даже такой приемлемый концепт должен нарушать причинность, ведь условный путешественник 3 может пройти через чревоточину, увидеть 4, который двигается по "прямому" не как рисунке пространству, вернуться обратно и может как-то нарушить причинность, хуй пойми что в голове у причинность-шизов тут, я их реально не понимаю.
Аноним 02/12/22 Птн 14:24:50 735792 59
>>735788
> эффект Казимира
Это про ведро вопрос?
Аноним 02/12/22 Птн 14:25:37 735793 60
>>735792
Это про степени вопрос. Квадраты в уравнениях легко, кубы - запросто, регулярно. Четвертые степени иногда встречаются, дальше уже интересно.
Аноним 02/12/22 Птн 14:26:39 735795 61
>>735791
Ты если не понял как работает нарушение причинности - не встревай. Посиди и пойми суть перед тем как спорить. Твое "я нипанимаю значет ниправда" никому не помогает, ты этим ничем от плоскоземельщика не отличаешься.
Аноним 02/12/22 Птн 14:46:01 735800 62
>>735795
Это ты не понимаешь, ты думаешь в рамках линейного движения, это вообще не то.
Аноним 02/12/22 Птн 14:51:09 735801 63
А ну приведите пример нарушения причинности
Аноним 02/12/22 Птн 14:59:08 735802 64
>>735800
Я как раз понимаю хотя бы в каких-то рамках.
Диаграмму Пенроуза для червоточин я пока не представил, этот вопрос надо обдумать. А не отрицать с ходу как бред потому что не понимаю. Если окажется что причинность не нарушается - значит она не нарушается. Если нарушается - значит нарушается.
Аноним 02/12/22 Птн 15:01:44 735805 65
>>735802
Да бля, ты понимаешь в рамках линейного движения со своими системами отсчета у каждого участника
А тут нет движения, тут перемещение между системами отсчета
Аноним 02/12/22 Птн 15:01:58 735806 66
>>735801
Я тебя затралил выебав твою мамку.
У тебя бомбанул пукан и ты летишь на околосветовой из солнечной системы.
Ты с помощью телепорта моментально передаешь своему знакомому питурду который тоже бомбанул и тоже летит из солнечной системы что у тебя бомбанул пукан.
Тот пока был рядом телеграфировал твоей мамке что я пидр.
Твоя мамка мне не дала до того как я ее выебал и у тебя пукан не бомбанул.
Аноним 02/12/22 Птн 15:03:26 735807 67
>>735805
Так добавь движение. Если брать простую систему без лоренц-фактора, то и без червоточин причинность не нарушишь.
Проблема проявляется при наличии движущихся агентов на релятивистских скоростях, потому мы это и рассматриваем.
Аноним 02/12/22 Птн 15:07:32 735808 68
>>735806
Как он мог телеграфировать до? У нас пуканы бомбанули одновременно, или у него после меня?
Аноним 02/12/22 Птн 15:09:58 735809 69
>>735808
>У нас пуканы бомбанули одновременно
Считай что да. Совпадение просто, летите в одном направлении.
Аноним 02/12/22 Птн 15:13:03 735811 70
>>735809
И где нарушение? Как он пошлет телеграмму до твоего страстного слияния с моей мамкой?
Аноним 02/12/22 Птн 15:14:14 735812 71
>>735811
В его системе отсчета соития еще не произошло.
Аноним 02/12/22 Птн 15:17:46 735814 72
>>735806
Мне кажется тут изъян в том, что наблюдаемое воспринимается как объективная реальность, а не как круги на воде и прочая такая хуйня.
Аноним 02/12/22 Птн 15:19:03 735815 73
>>735812
Почему? Я вас застукал, бомбанул и улетел и мой друг улетел одновременно со мной. Соитие произошло. Опиши конкретную последовательность событий и кто на ком стоял.
Аноним 02/12/22 Птн 15:19:26 735816 74
>>735811
Лол, это же похуй. Если мы имеем телепортацию, то скорость света и "обычная" информация становятся как звук выстрела и сверхзвуковая пуля.
Аноним 02/12/22 Птн 15:23:38 735817 75
>>735807
Бля, ты пытаешься усложнить когда мы еще с основами этой концепции не разобрались, не оч хороший подход.
При сильном релятивизме сложновато выходит, но мне кажется что это не нужно учитывать, так как суть именно в переходе между системами отсчета, а не физическом перемещении .
Аноним 02/12/22 Птн 15:27:53 735818 76
>>735812
Не так. Он не получил информацию о соитии. Произошло или нет - не имеет значения.
Аноним 02/12/22 Птн 15:30:30 735819 77
А вообще, для реального нарушения причинности нужно чтобы течение времени было отрицательным
То, о чем вы говорите - нарушение причинности только если считать системы отсчета неизменными, что неверно
Аноним 02/12/22 Птн 15:43:59 735820 78
Два объекта с одинаковой массой движутся навстречу со скоростью 0,9с и сталкиваются
Какую скорость использовать для вычисления энергии от столкновения, 1с же?
Если так, то куда девается остальная энергия?
Аноним 02/12/22 Птн 15:45:31 735821 79
>>735820
>со скоростью 0,9с
В смысле, со скоростью 0,9с относительно стороннего наблюдателя.
Аноним 02/12/22 Птн 16:09:29 735823 80
Аноним 02/12/22 Птн 16:12:24 735824 81
>>735806
пчел, путешествия в прошлое или передача туда информации - это уже несколько другой жанр фантастики.
Аноним 02/12/22 Птн 16:23:09 735825 82
>>735783
>Эммм, потому что для нет никакого хоть немного правдоподобного теоретического обоснования?
из попыток умозрительного анализа "представим такую-то гипотетическую хуиту, что тогда и как будет происходить?" фундаментальная наука состоит чуть менее чем наполовину.

>остается обосновать передачу информации в место телепортации
ага, естественно. Соственно, об этом и идут вскукареки - дескать, если вдруг можно передать информацию быстрее света - неважно как, главное сам фак - то все, причинность гроб гроб кладбище пидор.
Аноним 02/12/22 Птн 16:44:03 735826 83
>>735825
>"представим такую-то гипотетическую хуиту, что тогда и как будет происходить?" фундаментальная наука состоит чуть менее чем наполовину
Не совсем. Фундаментальная наука использует гипотетическую хуйню для объяснения наблюдаемых событий, типа темная материя/энергия. Мы не придумываем новую хуйню, мы используем гипотетические концепты как способ заткнуть пробелы в знаниях и для исследования самой этой хуйты.
Телепортация же полностью гипотетическая. Если бы мы наблюдали некую передачу информации быстрее света - мы бы могли на полном серьезе ввести концепт телепортации, но пока ничего такого нет.
Поэтому пока телепортация это мыслительный эксперимент, а не компонент обсуждений релятивизма и сверхсвета.
Аноним 02/12/22 Птн 16:47:51 735828 84
Как вообще телепортация может нарушать причинность лол
Вот я телепортировался на Марс, до которого 30 световых минут условно, отправил себе сигнал на Землю, вернулся обратно, через 30 мин получил сигнал. Где тут нарушение? Я же не в прошлое слетал.
Аноним 02/12/22 Птн 16:49:53 735829 85
>>735828
Или лол, причинность-шизы принимают получаемые фотоны за реальность и думают что я в двух местах одновременно? Так это же выходит что как раз они не понимают относительность.
Аноним 02/12/22 Птн 16:52:34 735830 86
>>735826
>Мы не придумываем новую хуйню
вот те же черные дыры были примером чисто карандашно-бумажной "новой хуйни", родившейся из того самого мысленного эксперимента "а что если гравитация некоего объекта подведет его вторую космическую скорость к скорости света?".

>Поэтому пока телепортация это мыслительный эксперимент
спасибо кэп.
Аноним 02/12/22 Птн 16:58:41 735832 87
>>735830
Ну так всем похуй на них было пока их реально не нашли
Аноним 02/12/22 Птн 17:02:12 735833 88
Гравитоны существуют?
Если у гравитации бесконечная дальность, значит ли это что энергия тоже бесконечна или ее можно сделать таковой?
Аноним 02/12/22 Птн 17:28:38 735836 89
>>735833
>Гравитоны существуют?
Экспериментально не обнаружены и обнаружены, скорее всего, не будут никогда. Но моченые склонны думать, что они должны существовать. Хотя даже теоретического обоснования для этого никакого нет, т.к. до сих пор еще не построена теория квантовой гравитации. Поэтому тут по большей части только вера в то, что они есть.
>Если у гравитации бесконечная дальность, значит ли это что энергия тоже бесконечна
Что значит "бесконечная дальность"? Гравитация - это искривление пространства-времени под действием массы тела. Сила этого искривления прямо пропорциональна массе тела, воздействующего на метрику. Любое тело обладает конечной энергией, в реальном мире вообще бесконечностей не бывает.
Аноним 02/12/22 Птн 17:43:11 735839 90
Почему электроны урана всегда крутятся влево?
Аноним 02/12/22 Птн 19:46:20 735853 91
>>735817
>Бля, ты пытаешься усложнить когда мы еще с основами этой концепции не разобрались, не оч хороший подход.
Так нет, это самый простой пример ломающий причинность. Без движущихся агентов не могу привести пример нарушения причинности. Это не значит что она не ломается, она ломается если они есть, потому мы и рассматриваем такой вариант.
Осло, если окажется что движущиеся агенты не ломают причинность в картине с червоточинами, это не значит что причинность не ломается с ними 100%, возможно есть другой пример где это происходит, тогда и его стоит рассмотреть.
Отрицать нарушение причинности при каких-то ситуациях это как утверждать что все твердые тела плотнее их жидких фаз и говорить "ты усложняешь" упоминая воду или кремний. Раз есть какой-то германий что в жидком виде плотнее чем в твердом, то утверждение "все твердые тела плотнее" становится неверным. Как и "нет нарушения причинности" потому что оно с релятивистскими агентами проявляется.
>При сильном релятивизме сложновато выходит, но мне кажется что это не нужно учитывать, так как суть именно в переходе между системами отсчета, а не физическом перемещении .
Сама тема сложная, я ж не спорю. Надо сперва осознать почему так происходит, понять объяснение и потом честно обсуждать.
Аноним 02/12/22 Птн 19:51:56 735854 92
image.png 48Кб, 602x433
602x433
>>735824
Это не фантастика. У тебя появляются "путешествия в прошлое" со сверхсветом и при релятивизме.
В плоском мире без лоренц-факторов с линейным пространством-временем можешь хоть обтелепортироваться, причинность не нарушится. Но оно не линейно.
Аноним 02/12/22 Птн 20:24:59 735855 93
image.png 1Кб, 146x52
146x52
>>735820
>Два объекта с одинаковой массой движутся навстречу со скоростью 0,9с и сталкиваются
>Какую скорость использовать для вычисления энергии от столкновения, 1с же?
0,99447513812154696132596685082873 c
Аноним 02/12/22 Птн 20:31:01 735856 94
>>735828
В твоем примере она не нарушается.
Добавь два космических корабля на релятивистских скоростях возле Марса и Земли соответственно.
На Земле ты телепортируешься если получаешь сигнал "все чисто" с корабля около Земли.
На Марсе корабль увидев тебя моментально отправляет кораблю у Земли "там занято, не давать сигнал все чисто", который корабль у земли получает до того как ты отправился.
Ты не получаешь сигнал "все чисто" и не телепортируешься.
Причинность нарушена.
Смотрите диаграммы Пенроуза, еще раз, это нихуя не интуитивно и не осознавая изменение временных линий это не дается простому пониманию с наскока.
Аноним 02/12/22 Птн 20:40:43 735857 95
>>735856
телепортируешься откуда куда? С корабля у Земли на корабль на Марсе?
>корабль у земли получает до того как ты отправился.
схуяль вот это вот взялось?
Аноним 02/12/22 Птн 21:33:55 735858 96
>>735856
>корабль у земли получает до того как ты отправился
С точки зрения кого, а? Марса? Тут нет нарушения причинности.
я жду сигнал на земле +
получаю сигнал +
телепортируюсь
марс видит меня на марсе и отправляет сигнал -
сигнал - приходит на землю через 10 мин после телепорта или сколько там до марса в данный момент
меня на земле уже нет, всем похуй. более того, сигнал - показывает реальную обстановку, так как я на марсе в момент его отправки
релятивизм тут вообще хуй знает каким боком, он может только изменить относительное время между получением сигнала, но он никак не может сделать его негативным, причинность никак не нарушить
Аноним 02/12/22 Птн 21:35:24 735859 97
>>735853
>Так нет, это самый простой пример ломающий причинность. Без движущихся агентов не могу привести пример нарушения причинности.
Не-не, просто с введением движения ты тут же отождествляешь телепортацию с движением, что уже неверно.
Еще раз - телепортация это смена системы отсчета, почему ты постоянно игнорируешь это?
Аноним 02/12/22 Птн 21:39:53 735860 98
image.png 7Кб, 395x302
395x302
>>735857
Когда летишь на релятивистской скорости у тебя другое "сейчас", одновременность другая.

>>735858
>марс видит меня на марсе и отправляет сигнал -
>сигнал - приходит на землю через 10 мин после телепорта или сколько там до марса в данный момент
Нет, они тоже используют моментальную телепортацию. Если просто так радиосигнал отправить, то нарушения не будет.

>>735859
>с введением движения ты тут же отождествляешь телепортацию с движением
Не отождествляю, с чего ты взял?
>Еще раз - телепортация это смена системы отсчета, почему ты постоянно игнорируешь это?
Не игнорирую, это действительно смена системы, моментальная при этом.
Ты не понимаешь как я понимаю и поэтому думаешь что я не понимаю.
Аноним 02/12/22 Птн 21:51:25 735861 99
>>735860
мне кажца ты не так эту диаграмму рисуешь
Аноним 02/12/22 Птн 22:12:59 735863 100
>>735860
>Нет, они тоже используют моментальную телепортацию
Лол ну ок
я жду сигнал +
получаю
телепортируюсь
марс посылает сигнал -, телепорт на земле его принимает
мне похуй я на марсе

В чем проблема тогда?
Для нарушения причинности нужно негативное течение времени, не?
Аноним 02/12/22 Птн 22:17:23 735865 101
>>735683
Это какую же такую систему отсчёта можно выбрать, а? Пруфируй свой бредок на понятных примерах или жопошник!
Аноним 02/12/22 Птн 22:22:32 735867 102
>>735860
>Не отождествляю, с чего ты взял?
>Ты не понимаешь как я понимаю и поэтому думаешь что я не понимаю.
Потому что иначе остается только проблема разного течения времени, все схемы и диаграммы тут становятся ни к месту, так как они завязаны на движении.
Аноним 02/12/22 Птн 22:26:39 735868 103
>>735861
Я первую попавшуюся в гугле запостил.

>>735863
>марс посылает сигнал -, телепорт на земле его принимает
>мне похуй я на марсе а ты еще не отправился
Починил тебя.
Сигнал приходит раньше из-за того что "сейчас" у движущихся агентов отличается от твоего "сейчас".

>>735867
Я объяснял почему надо движущиеся объекты для нарушения причинности. Без релятивистских агентов причинность (по-моему) не нарушается.
Аноним 02/12/22 Птн 22:39:33 735869 104
>>735868
>"сейчас" у движущихся агентов отличается от твоего "сейчас"
у них отличается восприятие "сейчас" друг друга.
Аноним 02/12/22 Птн 22:55:15 735870 105
Аноним 02/12/22 Птн 23:21:46 735871 106
>>735868
> "сейчас" у движущихся агентов отличается от твоего "сейчас"
Бля, а что такое "сейчас" лол? Такое впечатление как будто ты вводишь общее время и на основе разницы его для разных объектов делаешь вывод, типа я в 10:00 телепортировался, а там на месте 9:55 и все пиздец, обратный сигнал приходит мне в 9:55 тоже
Это же хуета изначально, так не надо делать
>>735868
> надо движущиеся объекты для нарушения причинности
Нет. Движущиеся объекты мы воспринимаем через обычную лол! относительность и все вытекающие отсюда выводы я не оспариваю ото/сто если че, а нам нужно смотреть на чисто разницу в течении времени.
Аноним 02/12/22 Птн 23:44:46 735873 107
image.png 23Кб, 1156x927
1156x927
>>735871
>Бля, а что такое "сейчас" лол? Такое впечатление как будто ты вводишь общее время и на основе разницы его для разных объектов делаешь вывод, типа я в 10:00 телепортировался, а там на месте 9:55 и все пиздец, обратный сигнал приходит мне в 9:55 тоже
Для тебя статичного и там и там 10 часов.
Для оранжевого летящего на релятивистской скорости "сейчас" на земле 9:55, а на Юпитере, скажем 10:05.
С этим никаких проблем пока мы не задействуем сверхсвет.
Аноним 02/12/22 Птн 23:45:44 735874 108
>>735873
Ой, блжад, линии не так загнул. Ну да лдано, суть ясна.
Спать пора, обсираюсь уже из-за чего объяснения страдают.
Аноним 03/12/22 Суб 03:37:25 735882 109
>>735779
>нельзя всерьез ввести в более-менее серьезные обсуждения перемещений быстрее света

Почему? Гей Хуйнштейн так сказал? Да похуй на него.
Аноним 03/12/22 Суб 09:37:35 735888 110
>>735882
Антисетипетух, не трясись.
Аноним 03/12/22 Суб 10:46:32 735896 111
>>735888
>Антисетипетух
Кто это? Местная аттеншн хор?
Аноним 03/12/22 Суб 11:37:58 735903 112
>>735882
>Почему? Гей Хуйнштейн так сказал? Да похуй на него.
Ты сам подсознательно обожествляешь Хуйнштейна!
Его гипотезы, формулы работают не от того, что он какой-то там блядь гений или злодей - а потому что он тупо подгонял их под реальность!
Если исходить из гипотезы среды из плотноприжатых сферообразных фотонов, то скорость распространения импульса через такую среду будет ограничена временем взаимодействия между плотноприжатыми фотонами - вот и ограничение скорости света для конкретной среды (да, потому что в другой солнечной системе фотоны могут быть иного диаметра - тогда и скорость света будет другой)
Теперь что касается движения - абсолютной пустоты не существует и поэтому при движении тело должно проталкивать плотноприжатые фотоны из фронта движения в тыл. То есть движение в нашей Вселенной на самом деле охуенно сложная штука и максимальная скорость такого движения ограничена опять же тем самым временем взаимодействия между плотноприжатыми фотонами - то есть тем, насколько быстро они могут перетекать с фронта движения в тыл.
Но вероятно можно наебать природу - например аналогом водомета для плотноприжатых фотонов, то есть этакий бублик и в центре он просто прогоняет фотоны в нужном направлении. Если в таком потоке удастся придать фотонам скорость большую, чем скорость света, то и двигаться можно быстрее скорости света.
Аноним 03/12/22 Суб 12:01:56 735906 113
>>735903
чем этот бред про фотоны отличается от теории эфира?
Аноним 03/12/22 Суб 12:10:59 735907 114
>>735906
Ну эфир-то разоблачили что он не работает, а эту свежую шизу никто не удосужился развенчать.
Аноним 03/12/22 Суб 12:18:39 735909 115
>>735873
Бля, у тебя получается что смотрим с общей системы отсчета, хотя по факту тут выходит что-то типа местного времени
9:55 и 10:05 могут свободно обмениваться информацией по факту, описываемая тобой проблема это просто попытка приравнять время к абсолютному, что уже противоречит самым базовым понятиям относительности, даже не говоря про ту хуйню что мы обсуждаем
Аноним 03/12/22 Суб 12:18:42 735910 116
>>735903
>Его гипотезы, формулы работают ... потому что он тупо подгонял их под реальность!
Ну вобще так наука и работает.
Аноним 03/12/22 Суб 12:24:48 735911 117
>>735909
Э-э-э, но я и не считаю что время абсолютно, с чего ты это взял? Я как раз опровергаю это.
Опять
>Ты не понимаешь как я понимаю и поэтому думаешь что я не понимаю.
Аноним 03/12/22 Суб 13:31:24 735917 118
>>735906
>чем эта гипотеза про плотноприжатые фотоны отличается от теории эфира?
Эфир никто не нашел, а плотноприжатые фотоны ебут колбочки и палочки в твоём глазу 24х7
Аноним 03/12/22 Суб 13:34:53 735918 119
9.jpg 26Кб, 686x464
686x464
>>735910
>Ну вобще так наука и работает.
Ахаха, ну типо того) У ученых ветер дует потому что деревья шатаются )))) Так и работают, так и работают мля))))
Аноним 03/12/22 Суб 13:48:38 735919 120
>>735673 (OP)
Какими программами и кодеками обрабатывают сигналы в космической сфере сейчас, как самыми безатратными? Интересует видеосигнал, хочу так же перекодировать видосы, чтобы не требовали рескрсов при воспроизведении на обычном ПК.
Аноним 03/12/22 Суб 14:07:37 735922 121
>>735911
Ну бля, если не считать что время абсолютно тогда эти случаи с 9:55 и 10:05 ничем не нарушают причинность, так как время в этих точках не равнозначно
Если мы говорим "сейчас" - мы не говорим одинаковое время, мы говорим именно сейчас
Аноним 03/12/22 Суб 15:24:02 735925 122
>>735918
> У ученых ветер дует потому что деревья шатаются
Это как раз таки подгонка фактов под теорию

Ученные же сначала наблюдают факты (деревья шатаются) и уже на их основе строят теории (ветер), то есть подгоняют теории под реальность.
Аноним 03/12/22 Суб 15:44:55 735928 123
>>735925
ты чё вообще несешь, ананас?
Теория это доказанная гипотеза, теорию никто никуда не подгоняет. А вот гипотезу, да))) Хех, костылями, бредом, подгоном подводят под действительность и если работает на прикладном уровне, то заебися! Главное вовремя переименовать в теорию или же сразу объявить такой - даже забив на доказательства.
Это как с теми фотонами, где регистрируют только конечный фотон - частицу с энергией. А прилетела она или же её толкнули соседние плотноприжатые фотоны - этим пидарасам похую абсолютно) Ведь для прикладной физики похуй - похуй! А то что их летающие фотоны полностью нарушили картину Мира - изъяв основы для понимания причин инерции, гравитации и тд - им тоже похую!
Аноним 03/12/22 Суб 16:11:28 735931 124
>>735925
>Ученные же сначала наблюдают факты (деревья шатаются) и уже на их основе строят теории (ветер), то есть подгоняют теории под реальность.
На примере темной энергии/материи очевидно что как раз наоборот, объективная реальность подгоняется под теории.
Аноним 03/12/22 Суб 16:16:23 735933 125
вопрос про инопланетян. Если злобно настроенные инопланетяне прилетят к нам на Землю - есть ли хоть малейший шанс их победить? Или уже сам факт того что они прилетели (у них есть такие технологии) означает что мы против них бессильны? Тогда все фильмы про пришельцев - чушь полная. Т к раз они долетели до нас - то нам уже пизда в любом случае.
Аноним 03/12/22 Суб 16:21:42 735934 126
>>735933
>Или уже сам факт того что они прилетели (у них есть такие технологии) означает что мы против них бессильны
Это.
Если только вдруг это не цивилизация аутистов, которая освоила путешествия в космосе, но не освоила производство оружия
Аноним 03/12/22 Суб 16:26:16 735935 127
Если на планету с жидкой водой уронить земные микроорганизмы, они приживутся?
Аноним 03/12/22 Суб 16:26:30 735936 128
>>735931
Вероблядь, спокойствие.
Аноним 03/12/22 Суб 16:26:38 735937 129
>>735933
встречный вопрос - а если предположить, что Земля полая и внутри обитает гораздо более развитая цивилизация в силу того, что она защищена земной корой и не подвержена внешним стихиям и катаклизмам. То кого считать инопланетянами - тех, кто живет внутри Земли или снаружи?
Аноним 03/12/22 Суб 16:27:13 735938 130
>>735935
Смотря какие там условия: химический состав, температура, давление и т.д.
Аноним 03/12/22 Суб 16:30:07 735939 131
>>735933
> Или уже сам факт того что они прилетели (у них есть такие технологии) означает что мы против них бессильны?
Сам факт ещё не означает. Если прилетели 2 туриста на местном пепелаце, или даже космолинкор алиенов, нас тупо больше и мы в подходящих условиях.

Хотя, может прилететь и рой наноботов, которому похуй вообще на всё
Аноним 03/12/22 Суб 16:35:24 735941 132
Почему нет космических лифтов с какими-нибудь плазменными двигателями через каждые 50 метров и подачей рабочего тела с Земли, а липестричесва от солнечных панелей на орбите?
Аноним 03/12/22 Суб 16:41:05 735942 133
>>735933
>есть ли хоть малейший шанс их победить?
Есть ли у шимпанзе шанс победить людей?
Аноним 03/12/22 Суб 16:58:19 735944 134
>>735941
Потому что это еще менее эффективно, чем нынешние ракеты
Аноним 03/12/22 Суб 17:00:04 735947 135
ОТО-шизы, а правда что даже в случае свехсветового движения без телепортации причинность не удастся нарушить, так как по мере приближения системы отсчета будут выравниваться?
Аноним 03/12/22 Суб 17:02:44 735948 136
>>735941
Хотя, есть такой концепт в архитектуре будущего, когда структуры поддерживает некий луч снизу, забыл как называется
Аноним 03/12/22 Суб 17:14:17 735949 137
>>735931
С чего вдруг? Сначала заметили что вселенная расширяется/галактики вращаются не как у всех нормальных людей и придумали под это дело теории. Никто же не подумал: "темная энергия/материя звучит круто, прям как в аниме" и стал искать куда бы это применить.
Аноним 03/12/22 Суб 17:18:28 735950 138
>>735941
Потому что суть проблемы КЛ не в том как поднимать груз - плазменным двигателем или лебедкой на беличьем двигателе, а в материале из которого его можно построить.
Аноним 03/12/22 Суб 17:19:54 735951 139
>>735922
>тогда эти случаи с 9:55 и 10:05 ничем не нарушают причинность
Как это не нарушают ты влияешь на причину события после события?
Аноним 03/12/22 Суб 17:20:54 735952 140
>>735948
Космический фонтан?
Аноним 03/12/22 Суб 17:43:53 735953 141
>>735949
>Сначала заметили что вселенная расширяется/галактики вращаются не как у всех нормальных людей
Они ПРЕДПОЛОЖИЛИ это про вселенную и галактики. Если углубиться в методологию оценки масс галактик и расстояний до них, то вообще оно не смешно становится.
Аноним 03/12/22 Суб 17:49:24 735954 142
>>735953
>Они ПРЕДПОЛОЖИЛИ это про вселенную и галактики
Уебище, ускоренное расширение Вселенной и наличие дополнительной массы в галактиках, скоплениях галактик - это наблюдательные факты. Не юродствуй.
Аноним 03/12/22 Суб 18:14:51 735955 143
>>735951
Бля мне кажется ты дико тупишь и не можешь отвязать свое представление от единого времени
Аноним 03/12/22 Суб 18:34:55 735956 144
>>735955
Так нет единого времени, ты чего?
Аноним 03/12/22 Суб 18:44:55 735958 145
>>735954
>это наблюдательные факты. Не юродствуй.
Наблюдать мы можем только пятнышки на небе где-то там, в разных диапазонах спектра. Все остальное это интерпретации моченых.
Аноним 03/12/22 Суб 19:00:36 735959 146
>>735958
Наблюдаются процесс расширения Вселенной и распределение массы в галактиках и скоплениях галактик, для этого используется множество методов. Ты срешь себе в штаны, антисетипетух.
Аноним 03/12/22 Суб 19:03:46 735961 147
>>735959
>множество методов
Каких?
100% с неверной интерпретацией результатов в силу ошибочного представления о мироустройстве.
Давай гони прям железобетонные, а не блядь о всяких красных смещениях, что может быть в тч следствием того, что фотоны с большим диаметром выбрасываются на периферию вихря в солнечной системе и тд.
Аноним 03/12/22 Суб 19:22:16 735962 148
>>735961
А, бля, это ты, плотноприжатый поехавший. Опять таблетки забыл принять?
Аноним 03/12/22 Суб 19:24:53 735963 149
>>735962
> пук сереньк
Слился как всегда, да что же это такое?!
Аноним 03/12/22 Суб 19:32:24 735964 150
>>735956
Вот именно, поэтому передача информации из 10:05 в 9:55 не будет ничего нарушать, ведь время у каждого свое
Аноним 03/12/22 Суб 19:40:58 735966 151
>>735963
>Слился
За щеку слил тебе своих плотноприжатых шарообразных фотонов, проверь.
Аноним 03/12/22 Суб 19:58:39 735970 152
>>735959
>распределение массы в галактиках и скоплениях галактик, для этого используется множество методов
С дисперсией в 2-3, знаем мы эти оценки.
Аноним 03/12/22 Суб 20:05:51 735971 153
>>735966
еле нашёл в пизде твоей мамаши, норм фотоны!
Аноним 03/12/22 Суб 20:08:22 735972 154
Как будет выглядеть зеркальный шар, летящий на 0.9с, спереди, сзади и сбоку?
Аноним 03/12/22 Суб 20:12:22 735973 155
>>735972
Скорее, лучше куб, для удобства
Аноним 03/12/22 Суб 20:13:38 735974 156
>>735970
Оценки основываются на точных наблюдениях. Да и наблюдения проводятся множество раз с помощью разных методов разными командами исследователей, независимых друг от друга. Все они дают один и тот же результат о наличии темного вещества и темной энергии.
>>735971
У тебя из жопы вылезли световые вихри из плотноприжатых фотонов, посмотри там. Поменьше хоть толкай хуев в жопу за раз.
Аноним 03/12/22 Суб 21:38:57 735976 157
>>735974
>о наличии темного вещества и темной энергии.
ты вообще в своём уме? нахуй эти воображаемые костыли?
Аноним 03/12/22 Суб 21:44:57 735977 158
>>735974
>темной энергии
Ты хотя бы в курсе, что энергия это мера? Она блядь сама по себе не существует!
Аноним 04/12/22 Вск 06:27:13 735988 159
>>735964
Будет нарушать, ведь причина для того чтобы что-то случилось в 10:00 перестанет существовать в 9:55.
Аноним 04/12/22 Вск 09:49:03 736000 160
>>735988
Так где нарушение лол
Если нет общего времени, мы не перемещаемся в прошлое или будущее, мы просто перемещаемся между системами отсчета с разными показателями времени
Аноним 04/12/22 Вск 11:21:57 736003 161
Почему звёзды так долго живут? Миллиарды лет это же пиздец дохуя, как всё вещество не выгорело?
Аноним 04/12/22 Вск 11:25:49 736004 162
>>736003
Они большие и вещества в них много. Но что любопытно, чем больше в них вещества тем быстрее они выгорают.
Аноним 04/12/22 Вск 11:50:42 736006 163
>>736000
Нет общего времени, но есть земное время и там событие в 10:00 наступает по причине которая предотвращена в 9:55 по земному времени.
Аноним 04/12/22 Вск 11:51:04 736007 164
>>736004
>что любопытно, чем больше в них вещества тем быстрее они выгорают
Почему это любопытно?
Аноним 04/12/22 Вск 11:55:34 736008 165
>>736006
Хуй знает как у тебя в двух точках одинаковое время стало
Аноним 04/12/22 Вск 11:57:37 736009 166
>>736008
>в двух точках
Я в этом посте говорил только про землю и ее время.
Аноним 04/12/22 Вск 12:04:47 736011 167
>>736007
Кошки от этого мрут.
Аноним 04/12/22 Вск 12:05:32 736013 168
>>736009
Так где нарушение епт
Мы телепортировались из 10:00 Земли в 9:55 Марс, дальше что блядь
Вернувшись на Землю мы попадем в 9:55 по "местному" времени по твоей логике, время с собой принесем или как?
Аноним 04/12/22 Вск 12:32:49 736015 169
>>735976
Это наблюдательные факты.
>>735977
Антисетипетух, спокойствие. Это условный термин, принятый для обозначения физической силы, которая стоит за ускоренным расширением Вселенной. В космологии и астрофизике такие термины - не редкость. Большой взрыв, черная дыра, темное вещество. Все спокойно пользуются этими терминами, потому что они уже устоялись, и только у всяких альтернативно одаренных шизиков, ниспровергающих Эйнштейна лежа на проперженном диване, постоянные анальные боли в связи с этим.
Аноним 04/12/22 Вск 12:38:18 736016 170
>>736003
Потому что в звездах очень много вещества, а в маломассивных звездах, которые как раз долго живут, термоядерные реакции идут не так интенсивно, как в голубых гигантах, и поэтому они могут жить миллиарды и даже сотни миллиардов лет, как красные карлики.
Аноним 04/12/22 Вск 12:38:35 736017 171
>>736013
>Мы телепортировались из 10:00 Земли в 9:55 Марс, дальше что блядь
Нет, мы телепортировались в 10:00 по земному на марс в 10:00 по марсианскому, у них одинаковое время (разница из-за релятивистских эффектов настолько мала, что можно пренебречь).
>Вернувшись на Землю мы попадем в 9:55 по "местному" времени по твоей логике, время с собой принесем или как?
Нет, телепортировавшись обратно ты попадешь в то же самое время.
Аноним 04/12/22 Вск 12:55:14 736019 172
164582723321-vs[...].png 806Кб, 1500x843
1500x843
>>736003
>Почему звёзды так долго живут? Миллиарды лет это же пиздец дохуя, как всё вещество не выгорело?
Потому что это центр вихря из плотноприжатых фотонов как глаз урагана и тд. Пока есть вихрь, круговое движение - будет и солнце.
Аноним 04/12/22 Вск 13:01:12 736021 173
>>736019
Но ведь под действием центробежных сил фотоны разожмутся и улетят.
Аноним 04/12/22 Вск 13:02:01 736022 174
>>736015
>Это наблюдательные факты.
И чё? Важно не наблюдение, а интерпретация - а то так дети и черта в темных кустах наблюдать могут.
>Антисетипетух
Блядь, ты реально шиз, какой из меня нахер Антисетипетух? Я на спейсач вкатился когда он отсюда уже сбежал.
>Это условный термин, принятый для обозначения физической силы, которая стоит за ускоренным расширением Вселенной.
Кручу-верчу, обмануть хочу? Ты чё несешь, Вася мля! Какая хер разница "энергия" или "физическая сила" - это всё равно к херам меры, которые сами по себе не существуют.
Аноним 04/12/22 Вск 13:04:16 736023 175
>>736021
>Но ведь под действием центробежных сил фотоны разожмутся и улетят.
Улетят только в том случае, если им есть куда лететь. А поскольку абсолютной пустоты не существует, Вселенная никуда не расширяется и фотонам улетать некуда. Так это всё и работает.
Аноним 04/12/22 Вск 13:16:20 736026 176
image.png 92Кб, 246x250
246x250
>>736023
>Вселенная никуда не расширяется
>спиздану-ка без пруфов и даже элементарных выкладок что-то что противоречит общепринятым теориям
>буду бугуртить когда меня закономерно называют шизом
Аноним 04/12/22 Вск 13:24:38 736027 177
>>736023
>и фотонам улетать некуда
А как ты тогда видишь?
Аноним 04/12/22 Вск 13:25:23 736028 178
>>735865
Незпруфировал, так и запишем. Теперь ты жопошник беспруфный ВАХАХАХАХАХАХААА ЧМОНЯ
Аноним 04/12/22 Вск 13:30:04 736029 179
кртзнств.png 2Кб, 91x113
91x113
>>736026
>буду бугуртить
Зачем ты меня преследуешь, ещё и пикчу с собой выложил. У тебя какое-то экстра-эго раз ты считаешь, что для меня важны мыслительные процессы в твоём кочане - насрать абсолютно!
>Вселенная никуда не расширяется
Да не расширяется - это дедуктивная логика, которую способны понять даже дети, но очевидно не ты)
Абсолютной пустоты не существует. Потому что абсолютная пустота означает отсутствие всего, в тч размерностей. Абослютная пустота это ничто - то бишь её и не существует. Любой объем пространства напротив обладает размерностью, а размерностью может обладать только материя - значит любой объем пространств на 100% заполнен материей. Так вот чтобы расширить Вселенную - нужно породить материю - а она увы не рождается и не исчезает, а лишь дробится и приобретает различные движения на уровне отдельных частиц.
Аноним 04/12/22 Вск 13:33:57 736030 180
6fa92ce1-e5f3-4[...].jpg 47Кб, 510x510
510x510
>>736027
>А как ты тогда видишь?
Передачей импульса через плотноприжатые фотоны - конечный фотон в цепочке проявляет себя как частица с энергией.
Аноним 04/12/22 Вск 13:35:14 736031 181
>>736017
Так где нарушение, я вообще твою позицию не понимаю
Аноним 04/12/22 Вск 13:35:45 736032 182
Аноним 04/12/22 Вск 13:37:59 736033 183
image.png 49Кб, 300x168
300x168
>>736029
>Да не расширяется - это дедуктивная логика
А, ну по твоей манялогике и квантмех хуйня. Да что там, по такой логике и земля плоская может быть, это же способны понять даже дети)))
Аноним 04/12/22 Вск 13:38:00 736034 184
>>736022
>И чё?
И хуй тебе в очко.
>Важно не наблюдение
Как раз оно в фундаментальной науке и важно. Наблюдения дают факты, которые подтверждают или не подтверждают наши модели. Либо же они дают основу для построения новых моделей, как произошло с темным веществом и темной энергией. Обнаружили существование во Вселенной темного вещества и темной энергии - построили модели, описывающие эти сущности.
>Блядь, ты реально шиз, какой из меня нахер Антисетипетух?
Тише, миша, не трясись.
>Ты чё несешь, Вася мля!
Хуй тебе за щеку несу.
>Какая хер разница "энергия" или "физическая сила"
Никакой, т.к. все эти термины используются для обозначения физического явления не очень понятной природы, которое стоит за ускоренным расширением Вселенной.
Аноним 04/12/22 Вск 13:38:46 736035 185
>>736031
Сигнал который мешает твоему отправлению в 10:00 приходит к тебе в 9:55. Но он отправляется если ты уже отправился. Объяснялось уже ведь.
Аноним 04/12/22 Вск 13:42:23 736037 186
>>736033
Это местный фрик, не обращай внимания. Он со своими плотноприжатыми фотонами тут уже неделю сидит слюни пускает.
Аноним 04/12/22 Вск 13:43:52 736039 187
>>736033
>земля плоская может быть, это же способны понять даже дети)))
ну это уже твоя больная логика)))
не проецируй)))
Аноним 04/12/22 Вск 13:46:37 736040 188
>>736035
Бля
я из 10:00 в точке А отправлюсь в точку Б, там 9:55
возвращаюсь обратно в точку А
какое время будет в точке А епт?
Аноним 04/12/22 Вск 13:49:48 736041 189
>>736034
>Как раз оно в фундаментальной науке и важно.
Это ты так решил? Строить модель на "наблюдениях" это извиним меня - логика попуасов, ибо все чувственные восприятия по факту лживы.
>Обнаружили существование во Вселенной темного вещества и темной энергии - построили модели, описывающие эти сущности.
Во-первых нихуя не обнаружили - а неверно интерпретировали полученные данные, а потом на их основе построили ложную модель, далекую от реальности.
>Хуй тебе за щеку несу.
>Тише, миша, не трясись.
Не индульгируй!
>Никакой, т.к. все эти термины используются для обозначения физического явления не очень понятной природы, которое стоит за ускоренным расширением Вселенной
- которого нет и не может быть! ОК
Аноним 04/12/22 Вск 13:52:44 736042 190
Аноним 04/12/22 Вск 13:57:29 736043 191
>>736042
С учетом что я потратил время на переходы туда-сюда, +1 минута ок
Но почему я перемещаюсь в нормальное время, а сигнал в примере перемещается в прошлое?
Аноним 04/12/22 Вск 13:58:05 736044 192
>>736040
Чёт вы заебали уже...
Аноним 04/12/22 Вск 14:07:10 736045 193
>>736041
>Это ты так решил?
Так работает наука.
>все чувственные восприятия по факту лживы
Какие "чувственные восприятия", долбоеб? Наблюдения ведутся с помощью высокоточной современной техники. Добро пожаловать в ХХI век.
>которого нет и не может быть!
Это опять ты, плотноприжатый шизик? У тебя там световой вихрь из жопы вылез, сходи проверь.
Аноним 04/12/22 Вск 14:19:13 736048 194
>>736045
>Так работает наука.
Хуево работает - костыли понаставляли и сидят довольные.
>Какие "чувственные восприятия", долбоеб? Наблюдения ведутся с помощью высокоточной современной техники. Добро пожаловать в ХХI век.
Переход от органов чувств человека к высокоточной современной технике никак не изменяет суть - "восприятия по факту лживы"!
>У тебя там световой вихрь из жопы вылез, сходи проверь.
Не индульгируй!
Аноним 04/12/22 Вск 14:35:17 736049 195
>>736048
Да вынь ты уже плотноприжатый хуй из своей сефрообразной жопы и сядь ровно, заебал ерзать.
Аноним 04/12/22 Вск 14:37:42 736050 196
15791637238-23.jpg 131Кб, 900x900
900x900
Аноним 04/12/22 Вск 15:22:21 736053 197
human-conscious[...].png 109Кб, 1200x630
1200x630
Что за спор в науке, о возможности влияния квантовых эффект на работу мозга?
Кто там фрик, кто там нормис?
Аноним 04/12/22 Вск 15:29:44 736055 198
>>736053
А в чем спор? Квантовые эффекты происходят, мозги работают.

>>736048
>восприятия по факту лживы
То есть твои теории о плотноприжатых фотонах основываются исключительно на голых фантазиях, ведь никакие наблюдения не могут ни подтвердить не опровергнут твою теорию.
Аноним 04/12/22 Вск 15:37:18 736056 199
>>736055
>То есть твои теории о плотноприжатых фотонах основываются исключительно на голых фантазиях, ведь никакие наблюдения не могут ни подтвердить не опровергнут твою теорию.
Ты же не будешь отрицать наличие фотонов, шиз?
А вот голые наблюдения привели к неверной интерпретации о "летающих со скоростью света" фотонах - потому что использовался первобытный подход и "восприятия по факту лживы".
Только подключив логику, дедуктивный метод можно познать истиную картину, а уже потом найти подтверждение на практике, а не наоборот - когда картина строится из неверно интерпретированных наблюдений.
Аноним 04/12/22 Вск 15:43:37 736057 200
Аноним 04/12/22 Вск 15:44:49 736058 201
>>736040
>я из 10:00 в точке А отправлюсь в точку Б, там 9:55
Там 10:00. Релятивистский зритель у точки Б телепортирует сигнал и он приходит в точку А в 9:55. Из-за этого ты из точки А не отправляешься.
Аноним 04/12/22 Вск 15:48:40 736059 202
>>736056
>Ты же не будешь отрицать наличие фотонов
Откуда ты знаешь что фотоны существуют? Ведь "восприятия по факту лживы", воспринимай их хоть глазом хоть высокоточными научными приборами.
Аноним 04/12/22 Вск 15:55:04 736061 203
>>736058
Лол а почему я отправляюсь в 10:00, а сигнал попадает в 9:55?
Аноним 04/12/22 Вск 15:58:20 736062 204
>>736061
Потому что у движущегося на релятивистской скорости другое "сейчас", для него "сейчас" у Марса 10:00, у Земли 9:55, а у Питера 10:05, например.
Было же уже. >>735873
Надо отдельный тред запиливать потому что вопрос сложный для понимания и совсем не тупой.
Аноним 04/12/22 Вск 16:42:19 736065 205
>>736053
Да шиза это ебаная. Старик Пенроуз впал в маразм.
Аноним 04/12/22 Вск 17:26:51 736069 206
>>736062
Бляяяяя у тебя тогда выходит не телепортация с перемещением в иную систему отсчета, а просто появление в наблюдаемую картину.
Типа если я сейчас телепортируюсь на альфа центавра, то я попаду на 4 года назад в то место, которое вижу с земли сейчас, а не то что на самом деле там сейчас происходит. Такая шиза выходит у тебя.
Аноним 04/12/22 Вск 17:49:43 736070 207
>>736059
>Откуда ты знаешь что фотоны существуют? Ведь "восприятия по факту лживы", воспринимай их хоть глазом хоть высокоточными научными приборами.
Прикладная физика не отрицается и работает - фотоны по факту существуют. Я тебе говорю про ложную интерпретацию физической среды - первобытный Вася видит "пустоту" - первобытный Вася воспринимает фотон как прилетающий из "пустоты". Восприятием картину Мира не познать - нужно исходить из дедуктивной логики. Это я ещё не говорю про то, что на уровне элементарных частиц перестает работать инерция, масса и тд - потому что эти явления возникают благодаря среде из плотноприжатых фотонов. А при взаимодействии того же фотона с фотоном нет ни массы, ни инерции - там работают уже другие законы и ты их никогда не познаешь восприятием - только логикой!
Аноним 04/12/22 Вск 19:04:24 736077 208
>>736069
Нет, если ты моментально телепортируешься на Альфу Центавра сейчас то окажешься там сейчас сейчас, а не в твои "сейчас" сейчас, ты не будешь на четыре года назад.
Почему ты упорно отказываешься меня понимать и придумываешь что-то чтобы я звучал глупо? Я не туплю и не вижу очевидные ошибки, попытайся ЭТО понять и представить что это не я косячу, а ты не понимаешь о чем я говорю, такое тоже случается внезапно.
Аноним 04/12/22 Вск 19:17:16 736079 209
>>736077
Потому что все твои объяснения завязаны чисто на прямом толковании ТО, без учета того что мы вводим перемещение между системами отсчета
И опять же, в ситуации с пониманием "сейчас" у тебя шаблоны из ТО, где сейчас не существует в привычном понимании что не мешает строить графики с единым временем лол
Аноним 04/12/22 Вск 19:27:33 736081 210
о чем сейчас спорят, какие тезисы?
Аноним 04/12/22 Вск 19:34:50 736082 211
типа про то что ТО не про объектиную реальность? а про наблюдателя(систему остчета) который наблюдает картину мира через фотончики с конечной скоростью
Аноним 04/12/22 Вск 19:43:05 736083 212
>>736030
потноприжатые протоны.
Аноним 04/12/22 Вск 20:13:05 736085 213
1670173985178.png 48Кб, 1280x393
1280x393
1670173985180.jpg 70Кб, 900x696
900x696
Бля поясните по хардкору разве эта хуета по центру с зеркалом не закрывает весь обзор?
Аноним 04/12/22 Вск 20:16:06 736086 214
image.png 192Кб, 699x396
699x396
>>736082
Примерно
Вообще сама суть в том что ОТ убирает понятия типа сейчас, единовременно и подобное, так как по понятным причинам их нет как таковых
Но если говорим про такие вещи как телепортация, эти привычные нам понятия снова становятся реальными
Надо просто не строить из себя релятивистского чепушилу и вернуться к базовым понятиям
Аноним 04/12/22 Вск 20:46:03 736088 215
>>736086
> Вообще сама суть в том что ТО убирает понятия типа
> единовременно
вообще говоря оно всё ещё доступно через синхронизацию времени постфактум, тогда когда фотончики событий дошли до наблюдателя, и даже разницу времени ускоренных систем можно реконструировать, но опять же постфактум
> Но если говорим про такие вещи как телепортация
да даже если говорить про сверхсвет, если предполагается что есть передача взамодействий выше с - пытаться аргументировать световыми конусами вообще нихуя некорректно
Аноним 04/12/22 Вск 22:07:42 736095 216
>>736085
Телескопы заточены для далекие источники, поэтому они делаются так, чтобы волновой фронт минимум пидорасился от множественный переотражений иначе дифракция и мыло. Маленькая хуита по центру имеет маленькую площадь по отношению по основному зеркалу, поэтому она не значительно влияние на светосилу при этом сильно корректирует волновой фронт, тем самым критично повышая апертуру.
Аноним 04/12/22 Вск 22:50:04 736096 217
undefined 182Кб, 557x321
557x321
undefined 1213Кб, 1200x630
1200x630
Вспомнил фильм Нолона, подумал про лунные приливы и представилось мне такое. В системе тел "Каменная планета + её луна" мы находимся на поверхности планеты и тут тоже есть жидкая вода, которая накатывает на какую-то конкретно взятую точку достаточно гладкой поверхности планеты, чтобы всякими неровностями - холмами, горами, обрывами и озёрами -
пренебречь, двумя гигантскими волнами раз в какой-то промежуток времени. При этом, вода во время таких отливов уходит СОВСЕМ, полностью оголяя хоть и сырую, но песчаную или глиняную поверхность, и снова вода приходится сперва подъёмом воды, а потом валом из стены воды спустя определённое время.
А при каких условиях такое возможно?
Я думаю так:
Ну, во-первых, раз вода жидкая, то планета находится в зоне комфорта и на планете есть атмосфера, причём достаточно густая, чтобы дать воде быть жидкой
Во-вторых, вода на планете должна быть в небольшом количестве, чтобы уместиться в 2 вала воды - подлунный и противолунный.
В-третьих, чтобы такие валы были возможны, они должны вмещать в себя много воды,а чтобы это было возможно, на них должна влиять большая гравитация, значит, места луна либо существенно больше луны, и/или ближе, при этом цикл огибания подлунной и противолунной водными валами будет строго определяться скоростью вращения луны вокруг планеты (при условии что луна с планетой не взаимоприловнозахвачены) и высотой орбиты луны над поверхностью планеты, что даёт высоте предел - предел Роша. Ну и чем ниже луна к пределу Роша, тем с быстрее и чаще волна будет огибать планету. Значит, луна в такой системе близкой, ближе земной, наверное тяжелее, но и не настолько, чтобы вызывать вулканизмы и прочие неприятности для планеты.
В чём я не прав из того, о чём подумал? А что насчёт того, о чём я ине подумал?
Аноним 04/12/22 Вск 23:27:31 736099 218
>>736079
>Потому что все твои объяснения завязаны чисто на прямом толковании ТО, без учета того что мы вводим перемещение между системами отсчета
Там буквально разные системы отсчета, статичные и релятивистские, окстись!
>И опять же, в ситуации с пониманием "сейчас" у тебя шаблоны из ТО, где сейчас не существует в привычном понимании что не мешает строить графики с единым временем лол
Прямые линии t и x это пространство-время в не-релятивистских системах, наклонные - в релятивистских, они разные.
Аноним 04/12/22 Вск 23:35:08 736100 219
image.png 15Кб, 1824x990
1824x990
>>736085
Возьми спичку и поставь ее перед глазом и посмотри на фонарь - спичка закроет свет от фонаря.
Теперь закрой ей свет от звезды. Она не закроет свет от звезды.
Потому что свет от звезд параллелен и попадет на зеркало. А вот свет от ближайшей вещи на зеркало не попадет и будет закрыт этой хуйней.
Аноним 04/12/22 Вск 23:44:03 736101 220
>>736100
Если я возьму спичку на расстояние вытянутой руки и направлю её на любую планету, то головка спички полностью закроет планету. Каким образом вот та здоровенная залупа с зеркалом по центру не закрывает?
Аноним 05/12/22 Пнд 00:05:28 736103 221
image.png 13Кб, 1824x990
1824x990
>>736101
С какого-то очень близкого расстояния телескоп начинает видеть, т.к. часть света от объекта попадает на основное зеркало.
На каком-то тоже астрономически близком расстоянии телескоп видит практически все что может упасть на зеркало.
Аноним 05/12/22 Пнд 00:35:49 736105 222
>>736085
по-хардкору в двух трех словах - нет, не закрывает.
Аноним 05/12/22 Пнд 01:58:52 736106 223
>>735766
>вопрос почему оно быстро испаряется на морозе.
Потому что конденсат на твоих очках теплее окружающего морозного воздуха. Невероятно, правда?
Аноним 05/12/22 Пнд 11:07:57 736126 224
Это тред тупых вопросов? Верно?

Внимание, самый тупой вопрос за всё время существования темы:

"Текущее состояния Роскосмоса это тяжёлая больезнь или предсмертная агония?"
Аноним 05/12/22 Пнд 11:40:39 736127 225
>>736126
Выглядят довольно бодро, по крайне мере по сравнению с большинством космических агентств.
ЕКА чёт вообще клоуны какие-то. Честно разочарован отсутствием космонавтики в Европе. То что там происходит - какая-то показуха, даже 1% своего потенциала не отрабатывают. Чисто пиндосам слились и добровольно отказались от космоса. Такая же история с Япошками и тд.
Аноним 05/12/22 Пнд 11:45:14 736128 226
>>736126
Ни то, ни другое.
Это норма.
Аноним 05/12/22 Пнд 11:48:21 736129 227
>>736127
роскосмослахта?
Что-то новое...
Аноним 05/12/22 Пнд 11:52:29 736131 228
>>736127
> Такая же история с Япошками
[✓] Ракета-носитель среднего класса, семейства H-II
[✓] Японский экспериментальный модуль «Кибо» на МКС
[✓] Японский зонд "Хаябуса-2" завершил свою многолетнюю миссию и доставил на Землю образцы грунта с астероида Рюгу
[✓] Доставили спутники из ОАЭ к Марсу в рамках программы Emirates Mars Mission
[✓] Nearly 100 satellites have been launched by and for Japan since the island country became the fourth nation ever to launch its own satellite to Earth orbit back in 1970
Аноним 05/12/22 Пнд 11:53:07 736132 229
>>736129
>кто не такой жопошник как я тот проплатка!!11
Ебало дегенерата представили?
Аноним 05/12/22 Пнд 11:54:22 736133 230
image.png 764Кб, 982x726
982x726
image.png 891Кб, 1140x540
1140x540
image.png 1067Кб, 1200x675
1200x675
image.png 177Кб, 1200x630
1200x630
>>736131
>[ ] Пилотируемая космонавтика
Ой.
Аноним 05/12/22 Пнд 11:56:04 736134 231
>>736129
Ну а в чём я не прав?
НАСА выруливает чисто на огромном бюджете, который нужно во что вкладывать - типо SLS за $4млрд/1полет, Орион и тд.
Но каких-то реальных технологических подвижек нет.
Ну а Китайцы недавно выдали на оф уровне, что хотять сравняться с ЕКА к 2035, а с НАСА к 2045 - пиздец) И в Китае та же самая история - свободного бабла много, вкладывают бездумно в космос.
Аноним 05/12/22 Пнд 11:57:32 736135 232
>>736131
Херня же всё - даже пилотируемой космонавтики нет, Маядзава вон по заграничным хуям скачет.
Аноним 05/12/22 Пнд 12:10:31 736137 233
image.png 351Кб, 960x720
960x720
>>736135

А куда им летать? На МКС? Предложение превышает спрос.
Вангую что Mitsubishi будет первой компанией, предложившей регулярные рейсы на Луну.
Аноним 05/12/22 Пнд 12:13:33 736138 234
image.png 585Кб, 879x486
879x486
image.png 584Кб, 1280x720
1280x720
>>736134
>Но каких-то реальных технологических подвижек нет.
Реюз же. Правда хуй знает насколько это выгодно, т.к. цены особо не поменялись, процентов на десять дешевле что ли стало.
Аноним 05/12/22 Пнд 12:14:09 736139 235
>>736133
А они по-русски говорят? Есть видео?
Аноним 05/12/22 Пнд 12:54:54 736142 236
>>736137
>А куда им летать? На МКС? Предложение превышает спрос.
Ой, ну что за детские оправдания?
Частные станции - туризм, наука, промышленность. С энергией плохо - стройте солнечные электростанции на орбите и тд.
В планах япошек кстати есть пилотируемые корабли по типу "миниСтаршипов" - если не ошибаюсь - к годам так 2035...2040
И у Европы тоже есть что-то по типу помеси Клипера и CrewDragon.
Аноним 05/12/22 Пнд 12:57:36 736143 237
Аноним 05/12/22 Пнд 13:02:25 736145 238
>>736142
>детские
>>736142
>стройте солнечные электростанции на орбите

Кто из нас ребёнок?
Аноним 05/12/22 Пнд 13:02:27 736146 239
>>736138
>Реюз же. Правда хуй знает насколько это выгодно, т.к. цены особо не поменялись, процентов на десять дешевле что ли стало.
Реюз-хуюз
Реюз оправдан когда есть спрос на частные полеты. Маск это обеспечивает Старлинками - которые по факту являются интернетом от Гугла, а Маск посредник, обеспечивающий технологическую реализацию. То есть здесь уже речь идет о реализации проекта от трансатлантической корпорации с захватом интернет-сектора в зависимых от сша странах.
А без спроса на частые запуски - реюз не оправдан.
Аноним 05/12/22 Пнд 13:03:25 736147 240
>>736145
>Кто из нас ребёнок?
Китай же будет строить - официально принятая программа.
Аноним 05/12/22 Пнд 13:18:53 736148 241
image.png 7Кб, 389x107
389x107
Аноним 05/12/22 Пнд 14:00:23 736152 242
Ну подскажите, пожалуйста, по этому посту >>736096
Аноним 05/12/22 Пнд 15:18:08 736157 243
отказ от физического тела это необходимое условие для нашей дальнейшей эволюции? Если например в будущем можно будет сознание помещать в биомашину, то и к звездам лететь будет не так сложно.
Аноним 05/12/22 Пнд 15:34:05 736159 244
Аноним 05/12/22 Пнд 15:56:21 736161 245
>>736157
>Если например в будущем можно будет сознание помещать в биомашину, то и к звездам лететь будет не так сложно.
Но однажды ты вернешься не таким, и "Фишхук" начнет на тебя охоту.
Аноним 05/12/22 Пнд 15:57:57 736163 246
>>736157
>отказ от физического тела это необходимое условие для нашей дальнейшей эволюции?
Эволюция движется по пути создания киборгов
>Если например в будущем можно будет сознание помещать в биомашину
Ты блядь и так биомашина, что тебя не устраивает? Размер пипирки?
Если уж помешать сознание, то в чисто механического робота - но это ж блядь нужно догадаться из чего выходить разум/сознание ))) Скопировав мозг ты не получишь Разум/Сознание - ибо это немного другая материя. Разум пораждает идеальная жидкость, которая заполняет промежутки между плотноприжатыми фотонами . А до этого хуй кто когда догадается - поэтому никогда не будет настоящего ИИ, никогда не будет сознания в роботах)))
Аноним 05/12/22 Пнд 16:09:34 736165 247
>>736147
Там и дома просто так строят, чтоб они пустыми стояли, держу в курсе.
Аноним 05/12/22 Пнд 16:11:06 736166 248
>>736163
>Эволюция движется по пути создания киборгов
Эволюция движется по пути создания идиотов.
https://youtu.be/4nyYv8R0jJM

Просто поверь. А я это наблюдаю уже много лет в режиме реального времени.
Аноним 05/12/22 Пнд 16:14:14 736167 249
>>736166
Факт.
А еще кто-то кукарекает что евгеника это плохо.
Аноним 05/12/22 Пнд 16:37:02 736170 250
>>736166
Да, все вокруг идиоты, один ты вумный.
Аноним 05/12/22 Пнд 16:43:38 736171 251
>>736170
Категоричность суждений не является признаком ума.
Речь всего лишь шла о тенденции.
Но раз ты так возбудился на в общем-то безобидный пост, то это уже что-то да значит.
Аноним 05/12/22 Пнд 16:54:13 736172 252
>>736171
>в общем-то безобидный пост
Ну да, всего-то назвал всё человечество тупеющими идиотами, подумаешь. Таких высокомерных ценников, считавших что новые поколения тупее предыдущих, в истории была тьма тьмущая, но что-то человечество никуда не делось и живет лучше чем когда-либо, должно быть из-за отупения, верно?
Аноним 05/12/22 Пнд 17:00:39 736173 253
>>736172
>и живет лучше чем когда-либо
Не факт, не факт.
Аноним 05/12/22 Пнд 17:04:47 736174 254
>>736170
Необязательно быть исключительно умным чтобы видеть идиотизм вокруг, даже среднего ума достаточно.
>>736172
В отличие от тогдашних ценников наблюдавших лишь свою непосредственную округу у нынешних есть пруфы со всего мира.
Аноним 05/12/22 Пнд 17:09:59 736175 255
>>736157
Меньше маняфантазируй.
>>736173
Медицина есть, тепло есть, вода есть (горячая и холодная), электричество есть, срешь не в дырку в полу, пользуешься самыми сочными плодами научного прогресса (компьютеры, смартфоны, интернет и т.п.). Ты правда такой овощ ебаный или просто троллишь тупостью?
Аноним 05/12/22 Пнд 17:16:08 736176 256
images.jpg 6Кб, 259x194
259x194
>>736172
Так ведь Идиократия как раз затрагивает короткий промежуток времени нынешнего технологического скачка - когда эти самые технологии и приводят к отуплению. Но это было предсказано ещё раньше фильма Идиократия - в любимом сериале моего детства «Чародей: Страна Великого Дракона», где всезнающий Оракул (аналог "ОК гугл") привел к деградации цивилизации. В этом сериале кстати были и другие прикольные модели развития человечества.
Ух как я залипал на эту Кэти)
Аноним 05/12/22 Пнд 17:30:11 736180 257
>>736175
>смартфоны, интернет

Порождают поколение идиотов, которое залипает в телефонах, вместо познания.

Успехи химической промышленности и рост населения закрыли доступ большинству к качественным натуральным продуктам.

Качественная медицина только для обеспеченных господ.

> вода есть (горячая и холодная), электричество есть,
Это всё было и в 1950-1970.
Аноним 05/12/22 Пнд 17:37:17 736181 258
>>736180
>закрыли доступ большинству к качественным натуральным продуктам
>Порождают поколение идиотов, которое залипает в телефонах, вместо познания
Бля, ебать ты шизик. Ты еще расскажи мне об ужасах ГМО и 5G.
>Качественная медицина только для обеспеченных господ
Я хуй знает в какой ты там пещере живешь, но в моем не самом большом городе я даже в ссаной государственной поликлинике могу себе поставить элементарные пломбы на зубы, которые будут хорошего качества (не был у зубного уже несколько лет, до сих пор все на месте).
Аноним 05/12/22 Пнд 17:38:15 736183 259
>>736180
>Это всё было и в 1950-1970
Распространено это раньше было меньше.
Аноним 05/12/22 Пнд 17:40:06 736185 260
Аноним 05/12/22 Пнд 17:42:25 736187 261
>>736181
>Бля, ебать ты шизик. Ты еще расскажи мне об ужасах ГМО и 5G.
Это другое.
Смартфон дает наркотик для ума в виде развлекательного контента - мозг как похотливая сучка хочет ещё и ещё. И не у каждого хватит силы воли чтобы контролировать мозг - подчинить его Разуму. Да многие даже не поймут, что мозг нужно контролировать - а отключение Разума у человечества есть высшая цель элиты.
Ну и кроме того обилие контента, пустой информации отвлекает мозг, не дает сосредоточиться на достижении поставленных целей.
Аноним 05/12/22 Пнд 17:46:05 736191 262
>>736181
> я даже в ссаной государственной поликлинике могу себе поставить элементарные пломбы на зубы
На отрезке 1/4 21 века ты называешь пломбы на зубы качественной медициной?
Аноним 05/12/22 Пнд 17:54:35 736194 263
>>736187
Тебе 15 лет?
>>736191
>ты называешь пломбы на зубы качественной медициной?
А как мне это называть, долбоеб? Стоматология - не медицина?
Аноним 05/12/22 Пнд 18:06:58 736196 264
>>736176
Это польская подростковая адвенчура с магией которую по телеку в конце 90-х - начале 00-х крутили?
Аноним 05/12/22 Пнд 18:08:11 736197 265
>>736187
Двачую. Смартфоны детям надо запрещать вообще.
Аноним 05/12/22 Пнд 18:19:57 736198 266
>>736194
Скорее тебе 30 - салабон ещё.
Аноним 05/12/22 Пнд 18:53:43 736202 267
>>736196
Ну да, про чародея Ашку)
Но смысл был заложен глубокий - как тупиковые пути развития человечества. Сейчас наверно такого уже не снимают ...
Аноним 05/12/22 Пнд 20:00:04 736205 268
Если на экваторе установить палку длиной в 1 световой год, и оставить планету вращаться как она сейчас вращается, то получится, что один конец палки будет вращаться быстрее скорость света, ока второй - со скоростью вращения Земли.
Можно из этой разницы извлечь энергию?
Аноним 05/12/22 Пнд 20:23:37 736208 269
>>736205
Палка не будет крутиться со скоростью выше скорости света. Она сломается.
инб4 она неломаемая
Тогда скрутится в спираль.
>Можно из этой разницы извлечь энергию?
Да, ты так землю замедлишь.
Аноним 05/12/22 Пнд 21:12:00 736214 270
Какого цвета Луна?
Аноним 05/12/22 Пнд 21:37:02 736217 271
>>736214
Серо-розовая (оче слабого оттенка)
Аноним 05/12/22 Пнд 21:44:21 736219 272
Какие пределы размеров ядерных реакторов?
Сильно мелкий, например, нереально сделать, цепной реакции не будет. Насколько маленькие они могут быть?
И насколько крупные? Можно вообще гиганистический заебенить и туда природного урану нерафинированного ебануть и шоб кипятил?
Аноним 05/12/22 Пнд 23:04:18 736226 273
>>736219
Вроде практический лимит 1 тонна, дальше становится нереально держать стабильную реакцию
Аноним 05/12/22 Пнд 23:19:28 736228 274
>>736219
А природный уран неспособен в критическую массу. Более того, сторонние примеси очень мешают процессу, не только снижая относительную массу урана, но и выступая как модератор реакции
Окло это другое, там уран сам собой рафинировался
Аноним 05/12/22 Пнд 23:39:05 736231 275
image.png 115Кб, 280x280
280x280
>>736226
>1 тонна
Нихуя себе, это ж меньше печи а энергии на три порядка больше.
А ты говоришь про обогащенный для реакторов уран или про любой?

>>736228
Я тупанул, не руду надо, а просто очищенный уран, но без обогащения. Можно?
Аноним 05/12/22 Пнд 23:50:22 736232 276
>>736231
Про готовое урановое топливо
>просто очищенный уран, но без обогащения
Да, в Окло вроде как раз такое примерно и было
Аноним 06/12/22 Втр 03:03:35 736236 277
>>736187
>И не у каждого хватит силы воли чтобы контролировать мозг - подчинить его Разуму.
ВОТ, вот блять ты и ответил на свой вопрос.
СИЛА ВОЛИ вот главный параметр на данной ступени эволюции, ну по крайней мере один из главных.
У кого есть сила воли, те будут наверху и будут руководить безвольными слизняками, которые в свою очередь будут всё более тупые и более безвольные и окончательно вымрут через несколько поколений.
Аноним 06/12/22 Втр 03:07:25 736237 278
>>736197
И? Безвольные хуесосы начнут искать другое, чем бы занять свой мозг, чем его задурманить, наркота, алкашка. Их безвольные мозги всегда будут одурманены, всегда будут под наркотой, допингом.
Запрещайте смартфоны, безвольные слизняки всего лишь вымрут быстрее на данном участке суши.
Аноним 06/12/22 Втр 03:17:52 736238 279
2886031000.jpg 470Кб, 1000x562
1000x562
2890001000.jpg 584Кб, 1000x667
1000x667
>>736231
>Нихуя себе, это ж меньше печи
Ну с разморозкой нахуй.
Аноним 06/12/22 Втр 08:34:48 736242 280
Чему отказались от атомолётов?
Можно было бы запилить что-то по типу летающих круизных лайнеров и отправляться в многодневные полеты.
Аноним 06/12/22 Втр 08:38:14 736243 281
>>736238
А на сколько мощей эти цилиндры?
Аноним 06/12/22 Втр 08:40:01 736244 282
>>736242
Биозащиты дохуища везти надо, если у тебя не круиз на тот свет, в итоге большой самбич будет иметь места с газельку.
Осло, если самолет бахнется, то потом по округе радиоактивные говны раскидает, а люди стараются такую возможность не допускать в корне.
Аноним 06/12/22 Втр 09:00:21 736246 283
>>736244
Подожди, но есть же вот эти ракеты, которые в ядерном реакторе разогревают набегающий воздух, а затем используют его для реактивной струи.
Аноним 06/12/22 Втр 09:20:51 736248 284
Аноним 06/12/22 Втр 09:40:12 736249 285
>>736248
А кроме США никто не разрабатывал?
Аноним 06/12/22 Втр 10:00:31 736250 286
>>736249
Говорят что есть еще экспериментальная ракета SSC-X-9 Skyfall, но у меня нет никакой конкретной инфы о ней.
Аноним 06/12/22 Втр 10:05:11 736251 287
Расскажите что-нибудь интересное.
Аноним 06/12/22 Втр 10:09:34 736252 288
image.png 1299Кб, 1706x840
1706x840
>>736251
В нейтронной звезде слои названы в честь видов макаронных изделий (если кто-то не поймет слово "паста")
Аноним 06/12/22 Втр 10:10:39 736253 289
>>736238
это контейнеры с отработанным топливом, а не реакторы
Аноним 06/12/22 Втр 10:13:07 736254 290
image.png 641Кб, 1200x676
1200x676
image.png 182Кб, 555x240
555x240
image.png 103Кб, 275x183
275x183
image.png 934Кб, 1192x795
1192x795
>>736253
Почему-то не могу найти такие же как он запостил.
Аноним 06/12/22 Втр 10:29:41 736256 291
Аноним 06/12/22 Втр 10:55:34 736257 292
Аноним 06/12/22 Втр 10:56:34 736258 293
image.png 3Кб, 491x51
491x51
>>736256
Смотрю там дозиметр ДРБП-03, думаю, о, круто, интересно, сколько стоит...
Нихуя ж себе.
Аноним 06/12/22 Втр 11:13:30 736259 294
Какие на Весте и Церере гравитация, период вращения, g? Есть атмосфера?
Какие там средняя температура и светимость солнца?
Есть произведения про колонизацию или исследования этих тел?
Аноним 06/12/22 Втр 11:23:30 736260 295
>>736259
>Какие на Весте и Церере гравитация, период вращения, g? Есть атмосфера?
0.25 м/с2 (0.025 g), 0.28 м/с2 (0.029 g)
5.342 ч, 9 ч
Атмосфера отсутствует
>Какие там средняя температура и светимость солнца?
75 K- 250 K, 110K - 235K
>Есть произведения про колонизацию или исследования этих тел?
Есть.
Аноним 06/12/22 Втр 12:00:29 736268 296
undefined 753Кб, 800x598
800x598
>>736260
>0.25 м/с2 (0.025 g), 0.28 м/с2 (0.029 g)
Кавпец, мало. Это зачит как высоко может прыгнуть стандартный человек весом в - для удобного счёта - 100кг? Какой вес можно поднять стандартному человеку не напрягаясь?
>75 K- 110K -
Холодновато
>250 K, 235K - холодно. Как морозной зимой в России в сибирских ебенях на крещенские морозы
>Есть.
<===
Аноним 06/12/22 Втр 12:20:30 736269 297
>>736268
>Кавпец, мало. Это зачит как высоко может прыгнуть стандартный человек весом в - для удобного счёта - 100кг?
20 метров.
>Какой вес можно поднять стандартному человеку не напрягаясь?
Вес? Такой же.
Масса? Тонна.
><===
https://en.wikipedia.org/wiki/Category:Fiction_set_on_Ceres_(dwarf_planet)
https://en.wikipedia.org/wiki/Marooned_off_Vesta
Аноним 06/12/22 Втр 12:42:39 736272 298
>>736219
>Насколько маленькие они могут быть?
У РИТЭГов вес десятки килограмм
>И насколько крупные?
РБМК можно было делать огроменных размеров, они хорошо маштабировались.
Аноним 06/12/22 Втр 12:45:32 736274 299
>>736272
>У РИТЭГов вес десятки килограмм
РИТЭГи не реакторы.
>РБМК можно было делать огроменных размеров, они хорошо маштабировались.
Насколько крупные?
Аноним 06/12/22 Втр 13:06:26 736275 300
RIANarchive3050[...].jpg 367Кб, 1024x707
1024x707
>>736274
>Насколько крупные?
Аноним 06/12/22 Втр 13:11:25 736276 301
Аноним 06/12/22 Втр 13:12:42 736277 302
>>736276
Игр про настоящий космос-то по пальцам пересчитать.
Аноним 06/12/22 Втр 16:59:46 736282 303
>>736275
Что означает цвет этих квадратиков?
Аноним 06/12/22 Втр 19:04:05 736286 304
>>736172
>всего-то назвал всё человечество тупеющими идиотами
Он просто факт озвучил.
Не секрет, что даже в развитых странах iq падает последние 2 поколения.
Аноним 06/12/22 Втр 19:20:06 736287 305
>>736277
Какие? КСП, СпейсЕнджин и чё-то там "...наследники..." про орбиту?
Мне бы и космоподобный сеттинг, идентичный натуральному бы подошёл
Аноним 06/12/22 Втр 19:45:21 736289 306
image.png 29Кб, 630x225
630x225
Аноним 07/12/22 Срд 04:50:22 736305 307
201011301163743[...].jpg 196Кб, 600x432
600x432
>>736253
Это реакторы подлодок. Просто без всякого обвеса и трубок.

>>736243
Тепловая мощность— 90 МВт.
Аноним 07/12/22 Срд 15:27:33 736324 308
какой радиус шварцшильда наблюдаемой вселенной
Аноним 07/12/22 Срд 15:32:41 736325 309
из чего состоит промежуток между ядром атома и электроном? Квантовая пена?
Аноним 07/12/22 Срд 16:30:44 736326 310
>>736325
Ни из чего, там вакуум, а в нем - квантовые флуктуации.
Аноним 07/12/22 Срд 16:45:03 736328 311
Аноним 07/12/22 Срд 17:05:10 736329 312
>>736325
из плотноприжатых фотонов
Аноним 07/12/22 Срд 17:53:19 736335 313
image.png 429Кб, 735x731
735x731
>>736328
Ну не знаю я, с ходу не могу сказать, считать нужно.
У тебя не тупой вопрос, понимаешь?
Аноним 07/12/22 Срд 20:25:33 736360 314
>>736328
В чем вопрос то? Так то ты все правильно прикинул, а чтобы понять нюансы, надо считать.
Аноним 07/12/22 Срд 20:36:19 736362 315
1. Почему под землей на небольшой глубине холодно?
Ведь с глубины поверхность непрерывно прогревается. Очевидный ответ- остывает зимой, а за лето не успевает прогреться не катит, потому что в тропиках в пещерах тоже холодно.
2. Если за короткий по астрономическим временам промежуток времени (несколько десятков или сотен лет) охладить атмосферу Венеры до земных температур то как долго будет остывать ее поверхность?
Аноним 07/12/22 Срд 20:51:38 736364 316
пространство-время дискретно? Или бесконечно аналогово? И есть ли доказательства и намеки на дискретность? Есть ли та конечная ячейка пространства-времени меньше которой нет ни длины пространства ни короче времени?
Аноним 07/12/22 Срд 20:52:55 736365 317
image.png 16Кб, 229x220
229x220
>>736096
>>736362
>>736364
Это не тупые вопросы по-моему.
Что-то зачастили с нормальными вопросами, вы там охуели что ли? Где дебилы с тупыми вопросами?
Аноним 07/12/22 Срд 21:00:12 736366 318
>>736364
>пространство-время дискретно? Или бесконечно аналогово? И есть ли доказательства и намеки на дискретность? Есть ли та конечная ячейка пространства-времени меньше которой нет ни длины пространства ни короче времени?
Очевидные планковская длинна и планковское время. Я примерно так чувствую что дискретно.
Аноним 07/12/22 Срд 21:14:09 736368 319
Что такое дискретность?
Аноним 07/12/22 Срд 22:02:43 736371 320
>>736364
>Есть ли та конечная ячейка пространства-времени меньше которой нет ни длины пространства ни короче времени?
Планковская длина, планковское время.
Аноним 07/12/22 Срд 22:11:52 736372 321
>>736366
>>736371
Рекомендую прочитать что означают планковские величины.
Аноним 07/12/22 Срд 22:34:26 736375 322
>>736372
Рекомендую тебе не рекомендовать.
Аноним 08/12/22 Чтв 00:51:19 736378 323
>>736329
А из чего состоят фотоны?
Аноним 08/12/22 Чтв 08:36:57 736390 324
>>736335
>>736360
>В чем вопрос то?
Ну, например, как максимальной высоты будет волна? С какой скоростью будет бежать по планете? Может ли быть морская жизнь всякая водорослевая, планктонная и рыбная в этой бегущей волне? Если не абстрактные "планета+спутник", а "Земля+Луна", то как близко к Земле должна быть Луна и как она будет смотреться в Земном небе? Что тогда с погодой в небе будет, раз с погодой на суше на уровне моря - гигантская моноволна - всё ясно? Будут ли ветра и туманы каждый раз предшествовать набегу вала воды? Или там только ветра и затем резка из-з горизонта появляется и несётся с рёвом стена воды, а всё остальные погодные явление упрощены, и никаких дождей туманов и т.д.?
Аноним 08/12/22 Чтв 09:16:53 736391 325
>>736390
>Ну, например, как максимальной высоты будет волна?
Честно даже хз как посчитать. Высота гор ограничена прочностью горных пород, а тут хз.
>С какой скоростью будет бежать по планете?
С угловой скоростью обращения спутника. Расчет скорости движения по орбите есть в школьном курсе физики.
> Может ли быть морская жизнь всякая водорослевая, планктонная и рыбная в этой бегущей волне?
Думаю дальше примитивных самореплицирующихся молекул дело не пойдет.
>Если не абстрактные "планета+спутник", а "Земля+Луна", то как близко к Земле должна быть Луна и как она будет смотреться в Земном небе?
Ну помести Луну на предел Роша и посчитай её угловой размер. Это тоже школьный курс.
> Что тогда с погодой в небе будет, раз с погодой на суше на уровне моря - гигантская моноволна - всё ясно? Будут ли ветра и туманы каждый раз предшествовать набегу вала воды? Или там только ветра и затем резка из-з горизонта появляется и несётся с рёвом стена воды, а всё остальные погодные явление упрощены, и никаких дождей туманов и т.д.?
Думаю что из-за такой хуйни атмосфера будет перенасыщена водой и будет вечный туман, как на Венере. Насчёт стены воды, опять же хз, может она и не может быть такой высокой как в кино.
Аноним 08/12/22 Чтв 10:22:34 736393 326
>>736126
Предсмертная агония. Можно убрать слово космос и оставить слово рос. Вопрос не имеет смысла, все и так понятно.
>>736129
Роскосый и есть лахта. Иначе тред вели бы полтора шиза и он не обновлялся бы у них.
Аноним 08/12/22 Чтв 10:52:42 736394 327
>>736378
>А из чего состоят фотоны?
Это не тупой вопрос!
Ответ не может быть простым, но я старался!

А вообще, фотон - это конечная, элементарная, более неделимая частица. Её конечность подразумевает то, что она не имеет составных частей, но статус частицы подразумевает, что фотон - кусок чего-то.
Итак, имеем фотон-кусок, то есть частицу обладающую объемом, сферическую и имеющую спин - то есть фотон-сфера вращается вокруг своей оси.
Конечно возникает вопрос - а кусок чего?
В средневековье это называли это первоматерией. То есть некая бесконечно протяженная субстанция (материя) обладающая следующими свойствами:
- первоматерия это абсолютно плотное тело, не способное к сжатию или расширению
- первоматерия способна к делению - если часть объема первоматерии приобретает движение под действием силы, то она становится самостоятельной частицей.
- если частица теряет движение, то она способна слиться с остальными частицами обратно в первоматерию.
То есть частица существует пока у неё есть движение, отсюда постоянными стали частицы, имеющие осевое вращение. Они образовались в вихрях во время зарождения солнечных систем - когда частицы в круговом движении стачивали углы , превращались в сферы-фотоны и приобретали осевое вращение.
Пространство между сферическими плотноприжатыми фотонами заполняет всё та же первоматерия - по сути идеальная жидкость, которая постоянно дробится на мельчайшие частицы без осевого вращения и объединяется обратно - это чисто геометрический процесс, фотоны не испытывают сопротивления со стороны идеальной жидкости.
Есть и третий тип частиц - винтообразные (как шпилька с резьбой), они движутся поступательно и наворачиваются в промежутки между сферами. Этот тип частиц в том числе отвечает за магнитное поле. Из оф физики известны например нейтрино - им как раз рисуют такую форму. То есть частицы тоже имеют спин как сферы-фотоны. Но если винтообразные частицы упрутся в тупик, то они потеряют движение и сольются с первоматерией - отсюда фотон может стоять на месте и наворачивать круги вокруг своей оси, а винтообразная частица всегда должна двигаться вперед.

Но у нас не средневековье - поэтому ты можешь представить это несколько иначе. Что наш Мир - виртуальный, запущенный на компе у некого Васьки. Тогда вся это первоматерия есть некая область пространства в его программе, а приобретенное объемом пространства движение есть дополнительный параметр в программе, который порождает частицу. Когда параметры движения неких объемов пространства опять падают до нуля, то они сливаются в программе как равные. и тд.

Есть и иной взгляд "схоластический" - то есть что внутри фотона - ты! По сути весь Мир состоит из единой Мыслеформы, добровольно разделенной на части для познания своей сути. Мысль мыслит форму - мысль порождает форму. То что внутри тебя осознает себя - это тоже мыслеформа. Ты мыслишь формами - твоё мышление суть геометрическое. Ты не способен помыслить что-то нематериальное. А любое мышление сводится в том или ином виде к поиску определенной упорядоченности частиц материи - ведь даже информация материальна, она не способна существовать без носителя. То есть Ты, как мыслящая субстанция есть некий объем той самой идеальной жидкости, которая имеет связь с телом. Само мышление базируется на свойствах идеальной жидкости подстраиваться под любой объем - она выступает в роли некого сверхскоростного компьютера, оперирующего геометрическими вычислениями. Поэтому твои мысли мгновенны - они вспыхивают как огонь.

Огонь - Солнце как буфер идеальной жидкости в центре вихря из плотноприжатых сфер - есть сосредоточение мыслящей субстанции. Отсюда поклонение древних Солнцу, поклонение огню как проявлению Божества, нашей божественной природы. А Душа рассматривается как искра. Частица огня, мыслящей субстанции, идеальной жидкости.
Аноним 08/12/22 Чтв 11:33:23 736396 328
>>736394
>А вообще, фотон - это конечная, элементарная, более неделимая частица. Её конечность подразумевает то, что она не имеет составных частей, но статус частицы подразумевает, что фотон - кусок чего-то.
>В средневековье это называли это первоматерией. То есть некая бесконечно протяженная субстанция (материя) обладающая следующими свойствами:
>- первоматерия способна к делению - если часть объема первоматерии приобретает движение под действием силы, то она становится самостоятельной частицей.
Ты только что на ноль поделил.
Аноним 08/12/22 Чтв 11:38:25 736397 329
На каком расстоянии от внутренней границы облака Оорта может располагаться гипотетическая планета, чтобы не задевать облако Оорта и не выпердивать камушки внутрь системы, при условии, что эта гипотетическая планета Оорта не приближается к центру солнечной системы, а всё время подпирает внутренние границы облака Оорта, и что масса такой планеты варьируется от Меркурия до Урана?
Аноним 08/12/22 Чтв 11:43:05 736398 330
>>736396
>Ты только что на ноль поделил.
Где?
Фотон - кусок первоматерии, то есть самостоятельная частица. Фотоны благодаря своему осевому вращению не способны к делению покуда что-то не остановит их вращение, а это невозможно. Значит фотоны постоянные, неделимые сферические частицы.
Аноним 08/12/22 Чтв 11:50:41 736399 331
>>736396
И добавлю, что это чисто геометрические (вычислительные) процессы - в мире элементарных частиц нет трения и сопротивления. Они возникают только на уровне мира из хим элементов.
Поэтому вращение фотона вокруг оси вечное.
Аноним 08/12/22 Чтв 12:03:05 736400 332
>>736399
Ты когда на нобелевку выдвигаться будешь? Ну или хотя бы на медаль имени Лысенко.
Аноним 08/12/22 Чтв 12:12:05 736402 333
>>736400
Зачем?
Это же по большей части пересказ чужой гипотезы, лишь слегка адаптированной под современные знания.
А первоисточнику по всей планете памятники стоят, систему координат назвали в его честь. Которую потом придурки натянули на вселенную и давай искривлять иллюзорные линии - мол мы воображаемые линии искривили - возникла гравитация - атасс!
Блядь, может он нахуй на тысячелетия опередил своё время, хз. Или может люди по своей природе настолько примитивны, что никогда его не поймут, хз.
Аноним 08/12/22 Чтв 12:12:25 736403 334
>>735695
Нет ни у обычного горожанина, ни у мультимиллионера. Так уж получилось, что ты состоишь из килограммов. Из множества килограммов. Погугли стоимость доставку одного килограмма тебя (только доставку) на МКС, и закрой тред. У мультимиллионера будет пройден этот этап, но и ему каюк. Потому что МКС сама по себе зависит от земли более чем полностью.
Аноним 08/12/22 Чтв 12:25:07 736405 335
Аноним 08/12/22 Чтв 12:35:24 736407 336
>>736397
Да на любом. Если она там есть, то она свой путь внутри облака расчистила ещё миллиарды лет назад.
Аноним 08/12/22 Чтв 12:38:45 736408 337
>>736394
теорию струн отменили уже?
Аноним 08/12/22 Чтв 12:48:19 736409 338
>>736408
Конечно. Теория бран рулит.
Аноним 08/12/22 Чтв 13:13:49 736410 339
>>736408
виттен её сам в квантово-чернодырковом говне утопил
Аноним 08/12/22 Чтв 14:33:43 736415 340
83049.jpg 105Кб, 831x728
831x728
83050.jpg 106Кб, 1112x570
1112x570
Jk1mne5y1EPZ5Aw[...].jpg 136Кб, 580x326
580x326
Cколько времени нужно людишкам чтоб застроить Землю аки Корускант?
Аноним 08/12/22 Чтв 14:38:38 736416 341
>>736405
>5.05*10^27 метров.
600 миллиардов световых лет? Ты даун чтоль?
Аноним 08/12/22 Чтв 14:50:13 736417 342
>>736324
13.7 млрд светолет
Аноним 08/12/22 Чтв 15:36:55 736420 343
>>736408
>теорию струн отменили уже?
она допускает существование абсолютной пустоты - следовательно ложна.
Аноним 08/12/22 Чтв 16:27:30 736425 344
>>736420
плотноприжатый плс
Аноним 08/12/22 Чтв 16:41:42 736427 345
>>736420
Когда в твоей жопе нет большого плотноприжатого хуя, там абсолютная пустота или все-таки остаются микроскопические плотноприжатые сферообразные хуи, которые создают на границе сред твоего очка световой вихрь?
Аноним 08/12/22 Чтв 17:32:25 736430 346
FZmaVL7akAA8j6P.jpg 159Кб, 1108x1478
1108x1478
Аноним 08/12/22 Чтв 18:11:45 736432 347
>>736408
Её и не принимали.
Аноним 08/12/22 Чтв 18:36:18 736436 348
>>736430
И все-таки там пустота или микроскопические плотноприжатые сферообразные хуи?
Аноним 08/12/22 Чтв 20:19:37 736447 349
Может ли быть жизнь на планете, у которой нет солнца, но которая находится в звёздном скоплении у которого яркость плюс минус как у солнца? То что там перманентно будет день не упиздошит потенциальную воду и жизнь?
Аноним 08/12/22 Чтв 20:42:43 736449 350
>>736447
>в звёздном скоплении у которого яркость плюс минус как у солнца
Сомнительно, что такие существуют.
Аноним 08/12/22 Чтв 20:43:55 736450 351
>>736447
> в звёздном скоплении у которого яркость плюс минус как у солнца?
Такого сейчас не наблюдается.
Может быть возле активного галактического ядра, но там вряд ли будет постоянное освещение, может яркость меняться на порядки, так что не думаю.
Аноним 08/12/22 Чтв 22:01:56 736453 352
image.png 57Кб, 861x551
861x551
Аноним 08/12/22 Чтв 22:46:34 736454 353
>>736447
у этой планеты есть внутреннее солнце или нет?
Аноним 08/12/22 Чтв 22:51:04 736455 354
image.png 885Кб, 1000x622
1000x622
>>736454
>внутреннее солнце
Это как?
Аноним 08/12/22 Чтв 23:42:27 736456 355
>>736455
ну вот как на пикче, только без дырки на полюсах.
Аноним 09/12/22 Птн 00:17:32 736458 356
>>736447

В таких скоплениях у планеты будут постоянные проблемы: то метеоритом уебёт, то вспышкой новой пожгёт, то мимо пролетающая звезда перегреет или просто спиздит атмосферу. Но вообще можно пофантазировать, конечно. Например, если планета вращается в системе двойной чёрной дыры, которая засасывает все метеориты, или рядом нейтронная звезда, которая охраняет от атак. А что до радиации-так привыкнут организмы.

В любом случае на такой планете возможно жизнь какое-то время, например сотни или тысячи лет после искусственного создания биосферы.
Аноним 09/12/22 Птн 06:14:52 736462 357
>>736415
71% поверхности это вода.
Аноним 09/12/22 Птн 07:33:12 736464 358
>>736462
Дай Голландцам время.
Аноним 09/12/22 Птн 07:53:45 736465 359
Могут ли в космосе разделяться изотопы? Скажем, в центрифуге диска вокруг НЗ/ЧД, да так, что в одну часть облака насрать конкретными изотопами, и в итоге исказить наше представление о распространенности природных изотопов?
Аноним 09/12/22 Птн 10:50:39 736476 360
>>736465
Да, полно процессов, где изотопный состав плывет. И далеко ходить не надо, тот же Марс или Юпитер.
Вообще изотопный состав элементов тяжелее гелия здорово гуляет от места к месту галактике.
Аноним 09/12/22 Птн 12:45:37 736480 361
>>736465
>Могут ли в космосе разделяться изотопы?
Такое и в солнечной системе сплошь и рядом, гелий, водород, углерод.
Аноним 09/12/22 Птн 13:06:26 736481 362
Планеты-пельмени возможны? ЧТобы прям неебаца высокий горный массив в пару сотен км высоты и в тыщу км ширины полоса горного хребта бы огибала экватор? Или это удел нанопланет и всяких лун?
Аноним 09/12/22 Птн 13:14:04 736484 363
>>736481
На Земле гора выше 20 километров давлением расплавить свое основание.
Аноним 09/12/22 Птн 13:50:21 736485 364
>>736481
>Планеты-пельмени
Я когда был маленький пельмени называл "маленькие сатурники".
Поэтому сатурн = пельмень.
Аноним 10/12/22 Суб 00:20:41 736530 365
Осуществляли ли взлёт против вращения земли? хз зачем, тупо назло. не считая боевых и непопавших на орбиту
Аноним 10/12/22 Суб 00:25:38 736531 366
>>736530
Да, Израиль так спутники запускает, чтобы не бросать ступени на бугуртящих арабов.
Аноним 10/12/22 Суб 00:36:49 736534 367
image.png 676Кб, 1280x720
1280x720
Аноним 10/12/22 Суб 00:37:51 736535 368
>>736531
Странно. Так же дороже выходит с ценой килограмма на орбите, надо шекели беречь.
Могли бы как Илоний назад ступени сажать.
Или дропать на арабов, че они им сделают?
Аноним 10/12/22 Суб 03:39:30 736540 369
post-3-13040578[...].jpg 23Кб, 550x412
550x412
оче смешно 11.png 202Кб, 547x383
547x383
>>736394
Бля, тварь, ты под чем? Как, как ты блять это придумываешь?
Не могу перестать орать с этого потока сознания.
Охуеть кто-то мощную химозу употреблял. Ну кто это всё написал.

Плотноприжатые сферы
Идеальная жидкость
лоооол нахуй
Аноним 10/12/22 Суб 03:41:13 736541 370
1311440567919.jpg 69Кб, 658x493
658x493
>>736464
Пол миллиарда лет им хватит думаю.
Аноним 10/12/22 Суб 03:44:56 736542 371
>>736447
>Может ли быть жизнь на планете, у которой нет солнца, но которая находится в звёздном скоплении у которого яркость плюс минус как у солнца?

Ну свет то там будет, а тепла нет. Без жидкой воды, по современным представлениям жизни нет. Есть планеты странники, как раз какую ты описал. Это тупо промёрзшие ледышки.
Аноним 10/12/22 Суб 03:49:05 736543 372
Sat-pic905-895x[...].jpg 23Кб, 895x505
895x505
>>736481
>Планеты-пельмени возможны?
>ЧТобы прям неебаца высокий горный массив в пару сотен км высоты
Нет, гравитация, неумолимая сука.
Так что да, пельмени только мелкие хуетовинки.
Аноним 10/12/22 Суб 06:55:49 736545 373
>>736541
Тащемта да.
Ты про испарение океанов чере полмиллиарда лет, содомит?
Аноним 10/12/22 Суб 08:00:57 736546 374
Аноним 10/12/22 Суб 08:49:09 736547 375
>>736546
>Не индульгируй
Говорил Мартин Лютер Кинг папе римскому.
Аноним 10/12/22 Суб 09:11:21 736548 376
>>736547
Тебе скучно, да.
Может все разбежались потому что ты такой токсичный?
Но всё же не индульгируй, как говорил дон Хуан.
Аноним 10/12/22 Суб 09:40:27 736550 377
>>736548
>Тебе скучно
Ага, где мои сжатые фотоны?
Аноним 10/12/22 Суб 10:06:44 736551 378
>>736542
Тепло может изнутри идти. Наши планеты гиганты излучают в космос больше тепла чем получают от солнца. У ледяных гигантов лёд на самом деле разогрет до тысяч градусов.
Аноним 10/12/22 Суб 11:13:24 736555 379
>>736550
Плотноприжатые фотоны бесконечно тебя ебут 24х7 - если тебя это успокоит.
Винтообразные частицы, которые своим движением порождают магнитное поле тоже классные! Про них не нужно забывать - иначе не будет основы для ЭМ поля.
Они зарождаются на полюсах Солнца. Всё дело в том, что вихрь из плотноприжатых фотонов работает как гигантский сепаратор. Плотноприжатым фотонам некуда лететь - они остаются в вихре, а вот идеальная жидкость в промежутках между фотонами выбрасывается наружу вихря центробежной силой. В итоге каждая солнечная система как гигантский насос засасывает идеальную жидкость на полюсах центрального Солнца и выбрасывает её в плоскости вращения на периферии.
Но когда идеальная жидкость засасывается на полюсах, то она проходит через узкий канал, сформированный плотноприжатыми фотонами - ось вращения вихря солнечной системы. Это обычно промежуток между тремя плотноприжатыми фотонами. И некоторому объему идеальной жидкости больше не нужно дробиться чтобы пройти через этот канал (ось вращения), но этот объем идеальной жидкости приобретает осевое вращение чтобы навинчиваться в промежутках между плотноприжатыми фотонами. В итоге имеем вытянутую, завинченную трехгранную частицу с осевым вращением - на входе в полюс Солнца. Обычно они надламываются поcле ~ 3 витков. На разных полюсах формируются частицы с правой и левой резьбой. Они сталкиваются в центре Солнца и разлетаются в плоскости экватора. Часть из них улетает в плоскости вращения плотноприжатых фотонов - навинчиваесь в промежутки, часть возвращается к полюсам Солнца формируя магнитное поле.
Бывает брак - четырехгранные частицы, пятигранные и тд - это когда на входе в солнце ось формируется из 4,5 и тд фотонов. Такие бракованные частицы не могут вылететь из Солнца - им сложно навинчиваться в промежутках между плотноприжатыми фотонами. В результате формируются темные пятна на поверхности Солнца. Знаешь, постоянный магнит сформирован трехгранными частицами с правой и левой резьбой - они сформировали готовые каналы при застывании - поэтому магниты притягиваются друг к другу - трехгранным частицам проще проходить через готовые каналы, чем навинчиваться в промежутки между плотноприжатыми фотонами. Но если мы поменяем полюса, то частица с правой резьбой не захочет идти в канал с левой резьбой - магниты будут отталкиваться. Так вот темные пятна при формировании пронзаются потоком винтообразных частиц с формированием множества готовых каналов - в итоге возникает мощное магнитное поле.
Пока пожалуй хватит)
Аноним 10/12/22 Суб 11:14:44 736556 380
>>736551
>Тепло может изнутри идти. Наши планеты гиганты излучают в космос больше тепла чем получают от солнца.
Потому что внутри них солнца?
Аноним 10/12/22 Суб 11:39:31 736558 381
>>736555
>Плотноприжатые фотоны бесконечно тебя ебут 24х7
>Пока пожалуй хватит
Нет! Больше и жестче! Ввинти в меня трехгранную частицу что идеальная жидкость брызнула!
Ты реинкарнация Святой Толстоты, не удивлюсь если ты ученный астрофизик
Аноним 10/12/22 Суб 13:17:10 736569 382
>>736558
>Ввинти в меня трехгранную частицу что идеальная жидкость брызнула!
не индульгируй
Аноним 10/12/22 Суб 13:22:29 736570 383
>>736556
Нет, потому что они тёплые и долго остывают.
Аноним 10/12/22 Суб 13:40:42 736573 384
>>736570
А чем плоха идея с внутренним Солнцем? Она бы как раз объяснила наличие мощных общепланетарных магнитных полей.
Аноним 10/12/22 Суб 13:41:15 736574 385
>>736570
Нет, они генерируют тепло, а не просто остывают.
Аноним 10/12/22 Суб 14:06:23 736578 386
>>736574
>они генерируют тепло, а не просто остывают.
внутреннее солнце, бинго!
Аноним 10/12/22 Суб 14:35:00 736582 387
>>736578
Да и называется оно ядром, у Земли - железно-никелевым.
Аноним 10/12/22 Суб 14:52:42 736586 388
>>736484
>>736485
>>736543
А если схитрить и пойти от обратного? Не нагромождаем на экваторе высокую и массивную гору кольцом, а полюсах роем яму до самого экватора так, чтобы у экватора разница между стандартным уровне суши и вырытым составлял сотню км?
Аноним 10/12/22 Суб 14:58:10 736589 389
>>736582
Нет, с ядром какая-то ерунда на костылях. Оно же тяжелое, его центробежной силой должно выбрасывать наружу и тд и тп - уффф, фигня.
С внутренним солнцем проще. Я же писал выше, что Солнца как такового нет, это просто центр вихря из плотноприжатых фотонов. Так что не нужно его пугаться - вихри для нашей Вселенной естественны.
Аноним 10/12/22 Суб 15:37:46 736593 390
>>736586
Кто на ком стоял? У меня от твоего описания фотоны разуплотнились. Схему нарисуй. Но скорее всего под воздействием гравитации и горы до небес и ямы до фотонных вихрей уровняются гравитацией до средних уровней.
Аноним 10/12/22 Суб 15:51:38 736594 391
Снимок экрана о[...].png 29Кб, 646x542
646x542
Аноним 10/12/22 Суб 15:58:51 736597 392
>>736569
У тебя там плотноприжатая сфера за щекой, сходи проверь.
Аноним 10/12/22 Суб 16:07:06 736600 393
>>736597
не индульгируй.
Кстати цикличность солнечной активности хорошо объясняется потоком идеальной жидкости. Ведь на полюсах Солнца должен быть приток жидкости от других солнечных систем и от взаимного расположения солнечных систем зависит объем прокачиваемой жидкости. Чем больший объем прокачивается - тем больше бракованных винтообразных частиц - тем больше пятен на Солнце.
О как!
Аноним 10/12/22 Суб 16:13:51 736603 394
>>736594
Так какая разница, итог один — горный хребет, слишком высокий для гравитации данной планеты. Он осыпется и станет более низким и пологим. Асло, куда ты землю из ям девать собрался?
Аноним 10/12/22 Суб 16:30:11 736604 395
>>736600
Фотоны уже не могли становиться плотнее, идеальная жидкость просто вытекала из солнца.
Аноним 10/12/22 Суб 16:33:44 736605 396
>>736603
>Асло, куда ты землю из ям девать собрался?
Не знаю.
Можно ещё больше насыпать на экваторный хребет. А можно вообще всё выкопанное пустить на удобрения, на кирпичи, стёкла, руду и пустить в производство всего
Аноним 10/12/22 Суб 16:37:12 736607 397
>>736605
Это сработает только если это все выкинуть в космос и уменьшить массу планеты.
Аноним 10/12/22 Суб 17:41:40 736614 398
>>736600
А потом у меня идеальная жидкость полит.. полис.. полилася. Много идеальной жидкости налилось. Я... ну когда плотноприжатые сферообразные фотоны излучал просто...
Аноним 10/12/22 Суб 19:59:40 736623 399
>>736605
Вот так берешь копаешь копаешь полюса срезая слои и остается у тебя диск, так и до плоской земли недалеко.
Аноним 11/12/22 Вск 04:35:20 736630 400
>>736589
>Оно же тяжелое, его центробежной силой должно выбрасывать наружу
Ты совсем ебанариум?
Аноним 11/12/22 Вск 04:36:30 736631 401
фотоны.jpg 12Кб, 200x173
200x173
Аноним 11/12/22 Вск 04:37:35 736632 402
>>736586
>а полюсах роем яму до самого экватора
гравитация опять превращает всё в сферу
Аноним 11/12/22 Вск 16:29:06 736665 403
>>736632
>превращает всё в сферу
В плотноприжатую?
Аноним 11/12/22 Вск 17:29:44 736667 404
>>736632
>гравитация опять превращает всё в сферу
а ты раскрути и получишь блин
или куски начнут отрываться?
Аноним 11/12/22 Вск 17:31:53 736668 405
>>736631
Ну вот кстати прикол в том, что в этом коробке реально будут плотноприжатые фотоны! )))
Аноним 11/12/22 Вск 19:23:16 736692 406
>>736668
А у тебя за щекой есть плотноприжатый хуй?
Аноним 11/12/22 Вск 19:27:38 736695 407
А почему для запуска ракет-носителей не используют катапульты?
Аноним 11/12/22 Вск 19:47:34 736701 408
>>736695
Ракета должна будет выдерживать огромное ускорение и потом не сгореть на большой скорости в атмосфере. Но вообще, идеи такие есть, даже демонстратор недавно сделали и испытали, гугли SpinLaunch. Но это ещё только мелкий тестовый образец, разгоняющий груз до 1 маха.
Аноним 11/12/22 Вск 19:51:41 736703 409
>>736701
>Ракета должна будет выдерживать огромное ускорение

Почему огромное? Я думаю несколько g, не более 10.

>SpinLaunch. Но это ещё только мелкий тестовый образец, разгоняющий груз до 1 маха.

Это лохотрон, который работать не будет. С тем же успехом можно и миномёт использовать.

Я имею в виду катапульту, которая поможет ракете тронуться и набрать первые несколько десятков м/с, максимум пару сотен.
Аноним 11/12/22 Вск 19:57:29 736704 410
>>736695
Оно не стоит того. Стартовый комплекс выйдет дороже чем все запущенные ракеты этой серии.
Аноним 11/12/22 Вск 19:57:43 736705 411
>>736703
>Я имею в виду катапульту, которая поможет ракете тронуться и набрать первые несколько десятков м/с, максимум пару сотен.
Ну тогда это просто хуйня, которая почти ничего не даст, но будет пиздец какая сложная и дорогая.
Аноним 11/12/22 Вск 20:26:06 736715 412
Почему ядра массивных звезд схлопываются так быстро? Откуда внезапность? Какое там ускорение?
Аноним 11/12/22 Вск 20:38:04 736719 413
>>736704
>Стартовый комплекс выйдет дороже чем все запущенные ракеты этой серии.

Откуда инфа такая?

>>736705

Обычная гравитационная катапульта с рабочим ходом в сотню метров, что-то вроде портового крана с поднимающейся площадкой.

>которая почти ничего не даст

Очень много даст, если говорить о скорости/выводимой массе. Первые 200м/с Флакон набирает целых 40 секунд, то есть жрёт четверть своего топлива на это. Секунда, сэкономленная на старте-это секунда, выигранная на высоте. За счёт эффекта Оберта это солидная прибавка. Кроме того, это позволит
-уменьшать шум у стартового комплекса
-взять больше топлива, так как высокое пусковое ускорение больше не нужно
Аноним 11/12/22 Вск 21:03:23 736725 414
>>736719
>Откуда инфа такая?
Примерно почувствовал.
Ну ты сам-то прикинь сколько будет стоить такая херабора, просто примерно дерьмометр откалибруй, ясен хуй что это тебе не бетону залить да стартануть.
Это минимум объем (буквальный) как у небоскреба, небоскреб стоит миллиард долларов.
Небоскреб статичный и должен только свой вес держать, а твоей хераборе надо ракету стартовать и выдерживать перегрузки и ракетный пердеж, так что умножай минимум на десять.
Хуяк - у тебя 10 гигабаксов на стартовую ебалу.
Какой-нибудь Безос или Илоний может быть в такую сумму и уложатся с помощью дешевых мексов, неоплачиваемых переработок и дешевых подрядов, но не НАСА.
Эти бюрократы еще двойные прочности, проверки и надежности заложат, так что умножай еще на десять.
Вуаля, у тебя стоимость программы Аполлон для одного только стартового стола который может наебнуться от неудачного пука ракеты.
Надо оно тебе такое?
Аноним 11/12/22 Вск 21:41:11 736730 415
>>736715
Ну там масса же пиздецовая, поэтому и коллапс продолжается вплоть до формирования нейтронных звезд и черных дыр. Ядра маломассивных звезд переходят в состояние белых карликов и давления вырожденного электронного газа в них хватает, чтобы противостоять гравитации - там масса не такая большая. А по скорости коллапс не должен отличаться у звезд разных масс - он в любом случае происходит быстро.
Аноним 11/12/22 Вск 22:25:14 736738 416
>>736730
А почему так ВНЕЗАПНО, почему так быстро? Если там горение замедляется из-за того что топлива меньше, то должно давление падать медленно с окончанием запаса. Не может же все одновременно прогореть равномерно, сгорая с нужной скоростью.
Аноним 12/12/22 Пнд 04:42:54 736768 417
image.png 70Кб, 279x181
279x181
image.png 881Кб, 1280x720
1280x720
image.png 626Кб, 1242x815
1242x815
Несколько вопросов по реюзам.
Илоний сдался со спасением обтекателей? Получалось же вроде, нет?
Старшип когда? Какие у него характеристики, сколько груза на НОО будет получаться?
Безосовские ракеты когда? Сколько он еще глину месить собирается?
Что там по китайсам, есть что-то в обозримом, что утрет нос Илону хотя бы на бумаге?
Кто еще имеет подвижки в реюзе?
Аноним 12/12/22 Пнд 07:29:51 736771 418
414280c3c59e332[...].jpg 57Кб, 1136x640
1136x640
298993143820542[...].mp4 2124Кб, 720x1280, 00:00:05
720x1280
>>736768
Ебать ты извращенец)
Спецом утопили SpaceX и China треды чтобы такими вопросами срать в ТТВ?
И та - тут и Blue-тред был - но нужно водолазов вызывать чтобы его достать.
Аноним 12/12/22 Пнд 12:46:09 736785 419
>>736771
Это не только про спасекс, это про реюзы в целом.
Аноним 12/12/22 Пнд 13:27:50 736787 420
>>736768
>Илоний сдался со спасением обтекателей? Получалось же вроде, нет?
Вроде решили что их можно просто из воды вылавливать.
Аноним 12/12/22 Пнд 13:30:30 736788 421
>>736785
А что про реюзы?
Маск доказал техническую возможность эффективных реюзов.
Но важен не столько сам многоразовый носитель, а прежде всего постоянный спрос.
Где будет спрос - там и будет эффективный реюз.
В США это Маск и Безос - всё, остальные тупо идиоты что туда лезут без денежного буфера и гарантированного спроса.
В Китае на гос уровне спрос обеспечат аналогом Старлинка и строительством солнечной электростанции. Может и частникам отдадут какую-то долю в этих проектах. Если нет - то китайские частники мертворожденные.
Аноним 12/12/22 Пнд 13:33:52 736789 422
mediaFc8bZ2iXoA[...].jpg 211Кб, 1536x2048
1536x2048
>>736768
>Старшип когда?
По моим прикидкам март-апрель.

>Какие у него характеристики, сколько груза на НОО будет получаться?
100-150, точно не известно.

>Безосовские ракеты когда?
Q4 2023, на деле может опять сдвинуться.

>Что там по китайсам, есть что-то в обозримом, что утрет нос Илону хотя бы на бумаге?
На бумаге у них дохуя всего есть, реально в обозримом будущем нет, только маняпланы на Long March 9.

>Кто еще имеет подвижки в реюзе?
Рокетлабы ближе всего наверное, а так говорят про реюз многие. В ЮЛЕ испытали надувной тепловой щит, на котором планируют возвращать двигатели, Арианспейс рендеры выкатил (пикрел).
Аноним 12/12/22 Пнд 14:39:43 736797 423
Screenshot20221[...].jpg 275Кб, 1080x1323
1080x1323
>>736738
>Если там горение замедляется из-за того что топлива меньше
Не так. Заканчивается водород -> ядро сжимается и нагревается -> зажигается гелий -> заканчивается гелий -> ядро снова сжимается и нагревается -> зажигается углерод и т.д.
Но есть две проблемы:
1) Каждая последующая реакция (H->He, He->C,...) дает меньше энергии, а значит горение происходит быстрее, а соответствующая стадия короче предыдущей (см. пикрил)
2) Эта последовательность не бесконечна: когда ядро догорает до железа (дни-десятки дней после зажигания кремния), то все, начинается коллапс.
Аноним 12/12/22 Пнд 15:06:35 736800 424
>>736797
>Заканчивается водород
Но он же не заканчивается, его дохуя в звезде. Его просто в ядре начинает не хватать и ядро потихоньку сжимается до тех пор пока не разогреется для сжигания гелия который внизу. Не могу понять почему последняя стадия такая внезапная. Вот потихоньку кремний догорает и звезда должна сжиматься посильнее и посильнее, но нет, она в какой-то момент БАМ и сжимается.
Аноним 12/12/22 Пнд 18:40:05 736819 425
на Земле из-за гравитации время течет медленнее чем в космосе?
Аноним 12/12/22 Пнд 18:52:07 736821 426
Аноним 12/12/22 Пнд 22:28:54 736827 427
В чем смысл парсеков?
Аноним 12/12/22 Пнд 22:48:45 736830 428
>>736827
Расстояние углом мерить легко.
Аноним 13/12/22 Втр 11:06:46 736847 429
Почему говорят что в космосе вакуум, если по соотношению между длиной свободного пробега молекул газа и межзвездными, а тем более межгалактическими расстояниям получается что это обычная газовая среда?
Аноним 13/12/22 Втр 11:35:24 736849 430
>>736847
Как получается? Покажи расчеты.
Аноним 13/12/22 Втр 11:51:40 736850 431
Откуда мочёные знают, что наша Вселенная асимметричная? Может половина галактик на самом деле состоит из антиматерии? Фотоны-то одинаковые с материей испускают. Или не одинаковые?
Аноним 13/12/22 Втр 12:15:04 736852 432
>>736850
Аннигиляцию было бы заметно.
Аноним 13/12/22 Втр 12:52:08 736853 433
Можно ли висеть на расстоянии от солнца в статичном положении, не на орбите?
Скажем, с помощью солнечного паруса?
Аноним 13/12/22 Втр 13:31:28 736856 434
>>736853
Да, можно. Но придётся постоянно корректироваться из-за разных возмущений. Но это можно тем же парусом делать, если он не статичный.
Аноним 13/12/22 Втр 14:52:45 736857 435
>>736853
Нет
Солнечный парус дает очень слабую тягу, ее не хватит. Например, Земля движется вокруг Солнца со скоростью 30 км/с, это очень много.
В теории, висеть можно с помощью постоянно работающих двигателей, но это совсем фантастика.
Чуть менее фантастично висеть на двигателях в совсем дальних ебенях, но хз какой в этом смысл. Даже у Плутона скорость около 4 км/с

А висеть на стационарной относительно поверхности Солнца тоже нельзя, так как эта орбита почти совпадает с самой поверхностью и там все сгорит.
Аноним 13/12/22 Втр 15:53:59 736860 436
>>736853
>Можно ли висеть на расстоянии от солнца в статичном положении
Зависит от точки отсчёта. Можно без проблем.
Аноним 13/12/22 Втр 16:45:38 736864 437
Безымянный.jpg 20Кб, 800x600
800x600
Если в космосе построить маленький шар радиусом десятки см. с зеркальной поверхностью внутри и включить внутри самую тусклую в мире лампочку будет ли внутри шара постепенно становится светлее и теплее?
Можно ли это считать сферой Дайсона?
Или свет и тепло нихуя не будут накапливаться?
Аноним 13/12/22 Втр 16:59:06 736866 438
>>736864
>Если в космосе построить маленький шар радиусом десятки см. с зеркальной поверхностью внутри и включить внутри самую тусклую в мире лампочку будет ли внутри шара постепенно становится светлее и теплее?
Светлее - нет, яркость практически сразу станет максимально возможной. Зеркало и лампочка будут поглощать тепло пока лампа не перегреет себя.
>Можно ли это считать сферой Дайсона?
Н-наверное? СД вокруг звезд строится, так что технически это не оно, но это некий аналог в каком-то роде.
>Или свет и тепло нихуя не будут накапливаться?
Не будут. Установится равновесие с наружной поверхностью которая будет излучать тепло.
Аноним 13/12/22 Втр 17:04:18 736868 439
>>736864
Свет будет накапливаться какое-то время, но поскольку 100%-ного отражения добиться нельзя, стенки будут нагреваться и шар будет излучать тепло. В итоге всё придёт к эквилибриуму, неотличимому от лампочки накаливания с той же температурой, что и лампочка внутри.

>Можно ли это считать сферой Дайсона?
Нет.
Аноним 13/12/22 Втр 18:19:10 736877 440
>>736857
>ее не хватит
Ускорение свободного падения на Солнце в радиусе Земли это 6 мм/с^2. Что, паруса даже такое ускорение гасить не могут?
Аноним 13/12/22 Втр 18:19:13 736878 441
>>736725
>Это минимум объем (буквальный) как у небоскреба, небоскреб стоит миллиард долларов.

Зачем, какой объём?

>должен только свой вес держать

Но вес небоскрёба-десятки и сотни тысяч тонн.
Вес ракеты-сотни тонн.

> который может наебнуться от неудачного пука ракеты

Любой стартовый стол может наебнуться.

>ракетный пердеж

Как раз нагрузки на конструкцию от струй будут гораздо меньше, так как ракета меньше времени проводит у комплекса с работающим двигателем. Ведь двигатель на полную мощность будет включаться в момент отсоединения ракеты от ведущего устройства.
Аноним 13/12/22 Втр 18:20:08 736879 442
>>736877
>в радиусе орбиты Земли
Быстрофикс.
Аноним 13/12/22 Втр 18:27:05 736881 443
>>736878
>Зачем, какой объём?
Объем пусковой установки. Ты в рамках обычного стартового стола не придашь ракете ускорение, надо грубо говоря километр размера. Это примерно небоскреб.
>>должен только свой вес держать
>Но вес небоскрёба-десятки и сотни тысяч тонн.
>Вес ракеты-сотни тонн.
Большая йоба которая не только должна держать ракету, но и ускорять ее и при этом держать себя и выдерживать непогоду.
>Любой стартовый стол может наебнуться.
И любой стартовый стол это относительно простая конструкция не включающая в себя ускоритель для ракеты, как бы ты его ни представлял - что-то вроде кучи механизмов которые необратимо выйдут из строя при незапланированной быстрой разборке.
>Как раз нагрузки на конструкцию от струй будут гораздо меньше, так как ракета меньше времени проводит у комплекса с работающим двигателем. Ведь двигатель на полную мощность будет включаться в момент отсоединения ракеты от ведущего устройства.
Струя все равно направлена на конструкцию. Единственное что упрощается - тебе не надо делать защиту ракеты(!) от реактивной струи.
Аноним 13/12/22 Втр 18:52:22 736885 444
image.png 594Кб, 911x1102
911x1102
>>736881
>ускорение
>километр размера

Так это, ускорение вообще с размером никак не связано. С длинной разгонного пути связана скорость. Но скорость выше скорости звука однозначно не нужна. Я вижу рациональную скорость ракеты на старте в диапазоне 50..200м/с.

> километр

Километр это слишком много. Ну то есть в отдалённой перспективе можно и километр, но вообще хватит сотни метров чтоб был эффект.

50м/с при стартовом ускорении 50м/с/с это разгонный путь всего в 25м. 100м это уже 100м/с.

>но и ускорять ее

Ну даже 1000т ракеты с ведущим устройством при перегрузке 10g это всего 10ктс. В то время как крупнейшие небоскрёбы весят 500ктс.

>Единственное что упрощается - тебе не надо делать защиту ракеты(!) от реактивной струи.

Не надо делать бетонную яму с выходом для газов.
Не надо делать шумозащиту поливалками.

>И любой стартовый стол это относительно простая конструкция не включающая в себя ускоритель для ракеты

Стартовый стол достаточно сложная конструкция, так как заправленную ракету нужно держать в термостатированном состоянии длительное время, что-то делая с наледью, в том числе. То есть в ракету не просто льют топливо-обеспечивают его циркуляцию. Потом резко отсоединяют все шланги-кабели. Насколько дороже станет эта конструкция с катапультой-сложно сказать. Но вообще - вот мостовой кран, смонтировать на космодроме, подвесить противовесы, разгоняющие ракету через систему блоков-и всё.
Аноним 13/12/22 Втр 19:18:46 736886 445
>>736864
>Если построить шар радиусом десятки метров с зеркальной поверхностью внутри и
поместить в центр человека со свечой в руках, то что он увидит?
Аноним 13/12/22 Втр 19:51:11 736893 446
>>736864

>включить внутри самую тусклую в мире лампочку будет ли внутри шара постепенно становится светлее и теплее?

2 процента энергии будет поглощаться при каждом отражении, 98 это ну очень большой коэффициент отражения. То есть внутри будет просто в несколько десятков/сотен раз светлее, чем если бы стенки были чёрными.

>Можно ли это считать сферой Дайсона?

Нет.

>тепло нихуя не будут накапливаться?

Вообще как бы да.
Аноним 13/12/22 Втр 19:54:48 736894 447
можно ли терраформировать Солнце?
Аноним 13/12/22 Втр 20:01:06 736895 448
>>736894
Ты сейчас живешь на терраформированном кусочке от солнечного ядра.
Аноним 13/12/22 Втр 20:03:19 736896 449
>>736895
а если остудить солнце и сбросить туда ледяные астероиды с водой? Можно будет сделать из него вторую Землю?
Аноним 13/12/22 Втр 20:10:02 736898 450
>>736896
Нет, оно нагревается само собой.
Аноним 13/12/22 Втр 20:36:45 736900 451
>>736886
Свечку и отражённый со всех сторон свети этой свечки.
Аноним 13/12/22 Втр 20:38:26 736901 452
>>736894
>можно ли терраформировать Солнце?
Ты сейчас живешь на Солнце. Земля - это Солнце
Аноним 13/12/22 Втр 20:45:34 736904 453
>>736896
Тебя не смущает огромная по сравнению с Землей масса? Даже если оно остынет, не уверен, что там даже ходить можно будет, не расплющившись под собственным весом.
Аноним 14/12/22 Срд 01:34:58 736925 454
>>736885
>Так это, ускорение вообще с размером никак не связано. С длинной разгонного пути связана скорость. Но скорость выше скорости звука однозначно не нужна. Я вижу рациональную скорость ракеты на старте в диапазоне 50..200м/с.
Связано, если ты хочешь приемлимое ускорение и небольшой рывок.
>Километр это слишком много. Ну то есть в отдалённой перспективе можно и километр, но вообще хватит сотни метров чтоб был эффект.
>50м/с при стартовом ускорении 50м/с/с это разгонный путь всего в 25м. 100м это уже 100м/с.
5 g это многовато для РКН, максимальное ускорение на таких ракетах 3 g ко времени опустошения баков, а учитывая что ты лупишь ускорительный комплекс, то ты по идее еще меньше хотел бы для минимизации рывков (м/с3) чтобы огурцы и ПН испытывали меньше проблем.
Но давай использовать 3g в дальнейших расчетах.
Чтобы добавить жалкие 500м/с дельты которую ты получишь при воздушном старте с самолета-носителя летящего на 1 махе (https://www.researchgate.net/publication/268564872_Air_Launching_Eart-to-Orbit_Vehicles_Delta_V_gains_from_Launch_Conditions_and_Vehicle_Aerodynamics) тебе надо 4 километра разгона с 3g или 2.5 километра с 5g.
100 м/с это смешно, ты огромную сложную и дорогующую конструкцию строишь для получения мизерной прибавки к дельте которую можно получить переоснащением ракеты, например, добавлением бустеров, что выйдет несравненно дешевле постройки целого комплекса.
>Ну даже 1000т ракеты с ведущим устройством при перегрузке 10g это всего 10ктс. В то время как крупнейшие небоскрёбы весят 500ктс.
Не знаю к чему ты это упомянул, тебе небоскрёб ускорять не надо, твой "небоскрёб" это то что должно ускорять ракету. И это небоскрёб, иначе выигрыш по скорости мизерный.
>Не надо делать бетонную яму с выходом для газов.
Да, та конструкция которую тебе надо гораздо сложнее.
>Не надо делать шумозащиту поливалками.
Это и есть защита ракеты.
>Стартовый стол достаточно сложная конструкция, так как заправленную ракету нужно держать в термостатированном состоянии длительное время, что-то делая с наледью, в том числе. То есть в ракету не просто льют топливо-обеспечивают его циркуляцию. Потом резко отсоединяют все шланги-кабели.
Ключевое слово было "относительно", потому что всё что ты упомянул должно теперь быть с учетом движения и удержания ракеты. Это не просто кран с парой блоков, но перечислять возможные проблемы не имеет смысла не зная конструкцию.
>Насколько дороже станет эта конструкция с катапультой-сложно сказать.
Крайне сильно дороже.
>Но вообще - вот мостовой кран, смонтировать на космодроме, подвесить противовесы, разгоняющие ракету через систему блоков-и всё.
Добавь также башню или ферменные конструкции для удержания ракеты, удлиненные системы заправки и обслуживания которые должны убираться с пути платформы (если ты не собираешься зачем-то и их ускорять), огнеупорные тросы, их дублирование, системы предотвращающие запутывание/блокировку, и многие многие другие подводные камни.
И это... этот мостовой кран стоит 100 миллионов долларов и на порядок ниже моей прикидки в небоскреб. Чтобы получить хоть какой-то профит от этой ебатни надо его увеличить, т.к. 100 метров подъема с ускорением даже 5g даст тебе всего лишь 100м/с.
100м/с это не то ради чего кто-то стал бы городить потенциально одноразовый кран.
Аноним 14/12/22 Срд 01:48:19 736926 455
>>736850
>Откуда мочёные знают, что наша Вселенная асимметричная?
Оттуда, что мы не видим антивещества.
Аноним 14/12/22 Срд 01:50:28 736927 456
>>736668
А между плотноприжатыми фотонами что?
Аноним 14/12/22 Срд 01:54:10 736928 457
>>736927
ничего, они же плотно прижаты
Аноним 14/12/22 Срд 01:55:49 736929 458
>>736800
>Вот потихоньку кремний догорает
Ты совсем ебанат по ходу и это не лечится.
На таблицу посмотри.
Аноним 14/12/22 Срд 02:03:45 736930 459
700-nw.jpg 88Кб, 700x700
700x700
>>736928
Они же сферы, а не кубы. Значит между ними пространство, пустоты, чем-то заполненные. Что меньше фотонов и видимо ещё более плотно прижато, да?
Аноним 14/12/22 Срд 02:17:59 736931 460
>>736929
Там написано что где-то месяц кремний горит, что не так?
Аноним 14/12/22 Срд 04:31:30 736932 461
image.png 1445Кб, 1920x768
1920x768
Мне нравится доктор Беки Сметхёрст. Я кумер, или она реально толковый астрофизик?
Аноним 14/12/22 Срд 06:17:49 736935 462
>>736926
А как определяете, что видите именно вещество, а не антивещество? Фотоны-то оба испускают.
Аноним 14/12/22 Срд 06:57:04 736937 463
>>736935
Наблюдали бы аннигиляцию антивещества с веществом. Представь, какой охуевшей мощности произошел бы взрыв при слиянии галактики с антигалактикой? Такой взрыв можно было бы зафиксировать на другом краю видимой части Вселенной. Но ничего такого не наблюдается.
Аноним 14/12/22 Срд 08:53:08 736944 464
флакон 40с.JPG 23Кб, 812x485
812x485
>>736925
> g это многовато для РКН, максимальное ускорение на таких ракетах 3 g ко времени опустошения баков

На старте можно толкать под каждый бак. Так можно повысить ускорение до 5джи как раз.

>Чтобы добавить жалкие 500м/с дельты

500м/с начальной скорости это не 500м/с дельты.

>100м/с это не то ради чего кто-то стал бы городить потенциально одноразовый кран.

"Потенциально одноразовый" любой самолёт. Пассажирский. Потенциально.И пассажиры тоже.

>100м/с

Это мегадохуя. Это больше, чем с крыши 20-ти километровой башни запускать. И 100м/с в начале-это не 100м/с дельты. Это дельты где-то 500м/с. Сложно считать. Вот по полезной нагрузке 100м/с на старте это не менее 10 процентов.
Ещё раз, вот Фэлкон-9, 40-я секунда работы.
Аноним 14/12/22 Срд 09:58:03 736946 465
>>736937
> слиянии галактики с антигалактикой?
А зачем им сливаться? Вещество с антивеществом проаннигилировало данным давно, и антигалактика висит сама по себе, окружённая пустотой.
Аноним 14/12/22 Срд 10:45:55 736951 466
Аноним 14/12/22 Срд 10:47:49 736952 467
>>736944
>На старте можно толкать под каждый бак. Так можно повысить ускорение до 5джи как раз.
Не имеет значения за что ты толкаешь, у тебя ускорение на ПН и огурцов не должно быть высоким.
От большего количества точек закрепа общее ускорение не изменится.
>500м/с начальной скорости это не 500м/с дельты.
Разумеется, нет. Более того, РКН с этим комплексом будет переиначена на другой профиль выведения и работы.
Но на старте со статичной позиции это примерно 500 м/с дельты если ты реально осуществимую стартовую конструкцию сделаешь.
>"Потенциально одноразовый" любой самолёт. Пассажирский. Потенциально.И пассажиры тоже.
И РКН. К чему ты это? При катастрофе рейса весь аэропортный комплекс перестраивать не надо. При разрушении РКН на стартовой площадке не надо целиком строить сложнейший комплекс сооружений, как было бы в случае с ускорительной башней.
>Это мегадохуя. Это больше, чем с крыши 20-ти километровой башни запускать. И 100м/с в начале-это не 100м/с дельты. Это дельты где-то 500м/с. Сложно считать. Вот по полезной нагрузке 100м/с на старте это не менее 10 процентов.
Разумееется из-за гравипотерь и профиля это не ровно столько же дельты. Но у нас общие прикидки а не конкретные цифры которых нет и быть не может без хотя бы примерной конкретики по функционированию придуманного сооружения.
Дашь конкретные цифры по высоте и принципам работы ускорителя - можно будет прикинуть профиты для РКН и примерную стоимость ускорителя (100 миллиардов долларов США)
>Ещё раз, вот Фэлкон-9, 40-я секунда работы.
Отличный пример, кстати. Уже два километра прошло, но ракета не спешит превышать барьер Маха, т.к. аэродинамические потери ухудшат эффективность. Из-за этого выдерживается баланс между аэродинамическими и гравитационными.
Если ты будешь запускаться на одном махе, то либо будешь выжигать очень много топлива для поддержания большого ускорения вопреки плотной атмосфере, быстро словив макс-кью, а также будешь иметь огромные перегрузки к моменту отделения первой ступени; либо будешь жечь на малом газу и иметь гравитационные потери.
Амеры давно могли бы построить космодром в четырех километрах над уровнем моря на ламповых горах Сьерры Невады где-нибудь, только факт в том что логистика играет очень немалую роль в эксплуатации объектов наземной инфраструктуры космических пусков и всякая экономия от высоты нивелируется сложностью и дороговизной строительства, эксплуатации и обслуживания.
Так и с пусковой йобой - огроменная херабора должная выдерживать регулярные ураганы, могущая ускорять целые ракеты космического назначения, просто не стоит того в общей совокупности расходов.
Аноним 14/12/22 Срд 11:31:05 736953 468
>>736951
Это может быть любая обособленная галактика/группа галактик. Узнать это сейчас мы принципиально никак не можем.
Аноним 14/12/22 Срд 11:32:27 736954 469
>>736946
>Вещество с антивеществом проаннигилировало данным давно, и антигалактика висит сама по себе, окружённая пустотой
Сам себе противоречишь. Если все антивещество аннигилировало с веществом, то откуда возьмется излишек антивещества для формирования из него таких больших структур как галактика?
Аноним 14/12/22 Срд 11:53:50 736956 470
>>736946
>Вещество с антивеществом проаннигилировало данным давно
И это было бы заметно. Даже сейчас. Ты же в курсе что мы можем в прошлое смотреть?
Аноним 14/12/22 Срд 12:04:46 736957 471
>>736954
>Если все антивещество аннигилировало с веществом,
На границах имеется в виду. Центральные области остались не тронутыми.
>>736956
>Ты же в курсе что мы можем в прошлое смотреть?
В первые секунды после БВ?
Аноним 14/12/22 Срд 12:08:04 736958 472
>>736957
>В первые секунды после БВ?
Спустя 300к лет после БВ.
Аноним 14/12/22 Срд 12:44:03 736960 473
>>736958
А аннигиляция явно была почти сразу после бв.
Аноним 14/12/22 Срд 13:05:15 736961 474
>>736960
Её-то никто не отрицает. В итоге по факту после этой аннигиляции осталось только вещество.
Аноним 14/12/22 Срд 13:23:03 736963 475
>>736961
Это хуйня из под коня, основная на вере в однородность вселенной.
Аноним 14/12/22 Срд 13:50:01 736965 476
>>736963
Если у тебя есть достаточно мощные пруфы неоднородности вселенной, то где твоя нобелевка?
Аноним 14/12/22 Срд 14:24:42 736967 477
>>736952
>тебя ускорение на ПН и огурцов не должно быть высоким.

Огурцы выдерживают 8джи, ПН вероятно больше (смотря какая ПН).

>Не имеет значения за что ты толкаешь

Имеет, так как самая слабая часть-это ступени.

>При катастрофе рейса весь аэропортный комплекс перестраивать не надо.

При неудачном рейсе погибает самолёт.

>которых нет и быть не может без хотя бы примерной конкретики по функционированию придуманного сооружения.

Так и запишем: любое сооружение, которое не опробовано, не может быть оценено численно. Как вообще в космос удалось попасть?

>же два километра прошло, но ракета не спешит превышать барьер Маха, т.к. аэродинамические потери ухудшат эффективность.

Потому что просто не смогла. Двигатели на полном дросселе всё время. Просто тупо ракета медленно разгоняется. При стартовой тяге 800т и массе ракеты 600т ускорение будет только 3.3м/с/с.

>для поддержания большого ускорения вопреки плотной атмосфере

Это ниже всякой критики. Аэродинамическое сопротивление Фэлкона на трансзвуке, даже на уровне моря-тонн 50 силы максимум.

>четырех километрах над уровнем моря

15 километров над уровнем моря дадут тот же эффект, что начальный толчкок примерно в 60м/с.
Аноним 14/12/22 Срд 14:32:44 736970 478
>>736965
Так это к моченым, а не ко мне. Я только прочитываю на досуге то что они пишут и мордочками в их же говно макаю.
Аноним 14/12/22 Срд 14:39:07 736971 479
>>736957
>Центральные области остались не тронутыми
Каким образом, если у нас еще в самом начале Вселенной должно быть поровну вещества и антивещества? Они должны аннигилировать друг с другом полностью. Тогда бы Вселенная оказалась пустой, но вещество есть - образовался небольшой перекос между кварками и антикварками в пользу кварков (один кварк на миллиард кварк-антикварковых пар). А где антивещество? Оно никак не должно было остаться, мы его и не видим. Вопрос на данном этапе в том, как именно возникла барионная асимметрия. Но в том, что она есть - сомнений нет, ведь она прямо перед глазами.
Аноним 14/12/22 Срд 14:44:11 736973 480
image.png 227Кб, 800x533
800x533
>>736967
>Огурцы выдерживают 8джи, ПН вероятно больше (смотря какая ПН).
То что они выдерживают такие перегрузки не значит что надо их им подвергать. 9г это предел на котором тебя вытаскивает САС. Если ты будешь ускорять саму ракету на околопредельных перегрузках, то у тебя САС не сможет утащить огурцов не сломав их к хуям.
Алсо ПН сильно усложняется и удорожается для больших перегрузок. Для поперечных нагрузок как в случае с воздушным стартом их приходится перерабатывать/пилить с нуля, например.
>Имеет, так как самая слабая часть-это ступени.
Не в этом суть, ты можешь хоть за что толкать, важно чтобы перегрузки на экипаж и/или пн не превышали стандартные. Сейчас не заря космонавтики и даже инвалиды в космос летают, на больше чем 3г не расчитывай.
>При неудачном рейсе погибает самолёт.
Не вижу смысла продолжать эту бессмысленное отвлечение. Речь о комплексе шла.
>Так и запишем: любое сооружение, которое не опробовано, не может быть оценено численно. Как вообще в космос удалось попасть?
Не записывай так. Ты даже с размерами не определился. Я - да. Сказал что примерно километр надо хоть на какую-то выгоду при старте.
>Потому что просто не смогла. Двигатели на полном дросселе всё время. Просто тупо ракета медленно разгоняется. При стартовой тяге 800т и массе ракеты 600т ускорение будет только 3.3м/с/с.
Ну в случае с флаконом с его дросселируемостью это не так актуально, он может 3г на пустых баках держать, но не факт что ты не упрешься в макс-кью на старте при очень мощной установке которая давала бы тебе ощутимый бонус.
>Это ниже всякой критики. Аэродинамическое сопротивление Фэлкона на трансзвуке, даже на уровне моря-тонн 50 силы максимум.
Все равно проебы которые стараются минимизировать.
Ты же сам ратуешь за уменьшение потерь, почему и хочешь стартовый ускорительный комплекс. Не забывай и про проебы от воздуха.

Как бы то ни было, давай по-простому, раз фермишные приближения с небоскребами не помогли осознать проблему:
ДОПУСТИМ, что твой комплекс позволяет удвоить полезную нагрузку выводимую ракетой (не позволяет)
100 миллионов долларов стоит запуск ракеты.
10 миллиардов стартовый комплекс (даже не 100, я скидку своим прикидкам сделал)
Чтобы комплекс окупился надо 200 безаварийных запусков с него сделать.
Это все без учета НИОКР новой ракеты, потому что существующие ты так запускать не будешь, обслуживания самого комплекса, и того очень заметного факта что комплекс не удвоит ПН, так что для окупаемости надо совершить сотни запусков.
И удешевить такое дело когда тебе позарез надо сотни запусков делать можно не прибегая к массивным сооружениям добавляющим еще одну точку отказа (не одну) в и без того сложном деле запуска ракет космического назначения.
Аноним 14/12/22 Срд 14:45:16 736974 481
>>736970
Довольно легко тыкать пальцем и говорить о том какие специалисты тупые когда не разбираешься в вопросе. Вроде даже термин для этого был.
Аноним 14/12/22 Срд 14:50:33 736975 482
image.png 13Кб, 1265x557
1265x557
image.png 13Кб, 1270x532
1270x532
image.png 13Кб, 1271x524
1271x524
>>736952
>Но у нас общие прикидки а не конкретные цифры

Вот итерациями считаем движение ракеты вертикально вверх (реально она наклоняется и всё сложнее).
Пик 1. Пуск округлённо Фэлкона вертикально вверх как есть (стартовая высота 100м, стартовая скорость ноль). 113.7км, 2267м/с скорости.
Пик 2. Массу пустой ракеты увеличиваем с 23т до 25т, массу нагрузки увеличиваем с 130 до 131 (усиливаем конструкцию), стартовую высоту увеличиваем до 200м, скорость до 100м/с. 131.1км высоты, 2356м/с скорости.
И того-более 170м/с общая прибавка.

Проблема в том, что так считать сложно. Так как есть 2 эффекта.
За катапульту: для катапульты рациональнее использовать другую ракету с другим соотношением масс первой и второй ступени, меньшим пусковым ускорением, с которым выигрыш будет ещё больше.
Против катапульты: я взял прибавку в массе 2т к первой ступени и 1т к остальным, но реальное увеличение массы ступеней сложно посчитать.


Пик 3.
Движение ракеты (округлённо Фэлкон) первые 40сек, расчётная скорость 594км/ч, высота 3.04км (2.94 от точки старта). Это очень близко к реальным данным телеметрии (2.8км. 604км/ч). То есть ни о каком дросселировании и "полёте специально на низкой скорости" речь не идёт. На низкой скорости летают самолёты, у которых тяга падает с ростом скорости и вообще гораздо меньше, для ракеты-носителя сопротивление воздуха это мелочь, её можно практические игнорировать.
Аноним 14/12/22 Срд 15:08:45 736979 483
image.png 12Кб, 1275x531
1275x531
>>736973
>то у тебя САС не сможет утащить огурцов не сломав их к хуям.

1. Сделать САС мощнее.
2. Сломать человека можно перегрузкой в 20джи, это минимум. 6джи это на карусели перегрузка, куда всех подряд пускают, в пластиковом кресле и без ППК. 14джи это в пилотажных самолётах (там ещё управлять надо). Ну и на катапульте капсулу надо вбок сбрасывать, очевидно же, а не вверх тащить.

>важно чтобы перегрузки на экипаж и/или пн не превышали стандартные.

Важно чтоб ступени не развалились. Именно для этого движки отключаются у Фэлкона в конце работы первой ступени.

>на больше чем 3г не расчитывай.

А как космонавты и ПН спускаются на Землю с перегрузкой в 8джи после месяцев в невесомости?

>Речь о комплексе шла.

Который многоразовый.

>Ты даже с размерами не определился.

Определился. От 50 до 200м/с, стандартно-100м, 5джи, 100м/с.

>Все равно проебы которые стараются минимизировать.

Именно поэтому нужно сразу разгоняться, так при тяге равной весу ракета не будет скорость набирать вообще.

>Алсо ПН сильно усложняется и удорожается для больших перегрузок.

5джи-это не большая перегрузка.

>упрешься в макс-кью на старте

Поэтому рационально залить больше топлива и сделать бак больше. Для обычной ракеты так сделать нельзя-она с места не сдвинется, сгорит на старте. Если ракету запускать с катапульты-то она будет набирать высоту даже при нулевом ускорении после отделения ведущего устройства.

Вот пример: увеличиваем время работы с 160 до 180с, растёт масса пустой ступени и топлива, растёт скорость. F

>10 миллиардов стартовый комплекс

Весь космодром Восточный-180млрд руб, то есть 3млрд длр, а тут одна катапульта-10. Не верю.
Аноним 14/12/22 Срд 15:13:29 736981 484
>>736974
Эффект Да́ннинга — Крю́гера
Аноним 14/12/22 Срд 15:29:33 736983 485
>>736974
Конечно. Для того чтобы понять что блюдо не вкусное не обязательно быть поваром, а для нахождения простых ошибок в формальной логике не обязательно быть моченым.
>>736971
>Каким образом, если у нас еще в самом начале Вселенной должно быть поровну вещества и антивещества
Ты скозал?
Ну вот образовался пузырь с антивеществом в относительно свободной от вещества области вселенной и все, так хоть целые галактики из антивещества могут быть, посреди какого-нибудь войда.
Аноним 14/12/22 Срд 15:35:25 736984 486
>>736983
>Для того чтобы понять что блюдо не вкусное не обязательно быть поваром, а для нахождения простых ошибок в формальной логике не обязательно быть моченым.
А ещё не обязательно быть поваром, чтобы понимать, что аналогия с поваром - хуита и не аргумент.
Аноним 14/12/22 Срд 17:19:14 736989 487
>>736983
>Ну вот образовался пузырь с антивеществом в относительно свободной от вещества области вселенной и все
Ты скозал?
Вселенная в начале была однородно заполнена веществом. В первые микросекунды существования вещество Вселенной представляло собой кварк-глюонную плазму, равномерно заполняющую расширяющееся пространство. Тогда же и произошла генерация барионной асимметрии. В тот момент не существовало "относительно свободных от вещества областей Вселенной". Рост первичных неоднородностей плотности, благодаря которым во Вселенной появились различные структуры, включая галактики, произошел позже.
Аноним 14/12/22 Срд 18:26:08 736994 488
image.png 436Кб, 890x500
890x500
image.png 978Кб, 1600x986
1600x986
>>736979
>1. Сделать САС мощнее.
Нельзя. Предельная перегрузка для САС - 9г, это то что тренированные люди выдержат. Дальше ты рискуешь убить огурцов, никто на такое не пойдет.
>2. Сломать человека можно перегрузкой в 20джи, это минимум. 6джи это на карусели перегрузка, куда всех подряд пускают, в пластиковом кресле и без ППК. 14джи это в пилотажных самолётах (там ещё управлять надо).
При 14 джи тренированный пилот быстро теряет сознание даже в противоперегрузочном костюме. А мы про космонавтов говорим, которые не всегда пилоты. У них и так строгий отбор, куда уж строже, только буквальные супермены останутся летать.
>Ну и на катапульте капсулу надо вбок сбрасывать, очевидно же, а не вверх тащить.
У тебя огурцы в ложементах спиной вниз для облегчения перегрузок лежат, это раз, поэтому всегда примерно вверх надо стрелять.
Во-вторых, если у тебя ракета сильно мощнее текущих, то в любом направлении надо с максимально допустимым ускорением пулять, потому что поломка РКН может привести к полету ступени/осколков в любом направлении, но лучше вверх, т.к. между тобой и нижними ступенями еще есть сервисный модуль и верхние ступени.
>Важно чтоб ступени не развалились. Именно для этого движки отключаются у Фэлкона в конце работы первой ступени.
Мы про нештатные вроде как раз тех случаев когда САС срабатывала ИРЛ.
>А как космонавты и ПН спускаются на Землю с перегрузкой в 8джи после месяцев в невесомости?
Максимальная перегрузка - менее 5g. 8g - это в Союзе по баллистической траектории в нештатной ситуации. Штатная посадка не по баллистической.
>Который многоразовый.
Ровно до тех пор пока не бахнет, что с ракетами происходит. Готов гарантировать 100% отсутствие происшествий на старте? Я - нет.
>Определился. От 50 до 200м/с, стандартно-100м, 5джи, 100м/с.
Насколько профитно добавлять 100м/с на старте и сколько будет стоить такой комплекс?
>5джи-это не большая перегрузка.
Для спутников расчитанных на 3г на старте - большая.
>Поэтому рационально залить больше топлива и сделать бак больше. Для обычной ракеты так сделать нельзя-она с места не сдвинется, сгорит на старте. Если ракету запускать с катапульты-то она будет набирать высоту даже при нулевом ускорении после отделения ведущего устройства.
Толково.
>>10 миллиардов стартовый комплекс
>Весь космодром Восточный-180млрд руб, то есть 3млрд длр, а тут одна катапульта-10. Не верю.
Ты прав, должно быть дороже, но я решил максимально упростить и сделать скидку чтобы хоть какой-то смысл был.
Ну давай еще разок скидку сделаю, почему бы и нет. Будем не на Канаверале под строгим контролем НАСА, а в Жихуяне на месте расселенной деревни-миллионника силами бывших жителей, чтоб подешевле. Будет всего лишь в 10 раз дороже крана Тайсун, считай миллиард долларов.
Прикинь хуй к носу, насколько окупаемая авантюра?
Аноним 14/12/22 Срд 21:19:12 737016 489
image.png 6Кб, 303x81
303x81
image.png 23Кб, 689x194
689x194
>>736994
>это то что тренированные люди выдержат

Тренированные люди в ППК и креслах управляют самолётом при 14 джи. Сколько нужно, чтоб убить-ХЗ, но двадцатки мало точно. В любом случае у меня перегрузка 5.

>У них и так строгий отбор, куда уж строже

Значит выдержат тем более.

>При 14 джи тренированный пилот быстро теряет сознание

При 14 джи тренированный пилот управляет самолётом. Космонавтам после активации САС ничем управлять не надо.

>У тебя огурцы в ложементах спиной вниз для облегчения перегрузок лежат, это раз, поэтому всегда примерно вверх надо стрелять.

Повернуть тягу после отделения. Далее. Есть некая мысль, что ракету лучше швырять катапультой не вертикально вверх, а под небольшим углом к вертикали. Тут следующие плюсы
-меньше разворачивать потом, сразу разгоняется на восток
-если что-шлёпнется мимо комплекса
-удобнее отстреливать САС
Но из минусов то, что для этого нужно немного иначе к самой ракете подходить, хотя я думаю крен в 6 градусов ничего страшного не сделает

>Максимальная перегрузка - менее 5g.

Ну вот. А тут 5 короткое время. А на карусельке без ППК подпивасный пенс, в пластиковом сиденье и с потенциальной гипертонией выдерживает 6.

>Ровно до тех пор пока не бахнет, что с ракетами происходит.

Так и самолёт многоразовый пока не разобъётся. Далее вопрос, насколько этот комплекс будет устойчив к взрыву. Потому что если сделать что-то вроде тянущего тросса, переброшеного через блок, то некая уязвимость имеется (если противовес и блоки близко от ракеты). Если же делать катапульту иначе, то я вижу варианты такой конструкции, которую даже взрывом ракеты не особо прошибёшь.

>и сколько будет стоить такой комплекс?

Примерно как мостовой кран на 10кт.
>Будет всего лишь в 10 раз дороже крана Тайсун, считай миллиард долларов.

Согласен, реалистично.

>Толково.

Спасибо.

>Для спутников расчитанных на 3г на старте - большая.

Тут вопрос: что конкретно в спутнике расчитано на такую перегрузку, потому что 3 джи это что-то вроде большого ухаба при езде на машине. То есть эту перегрузку выдержит практически любой гражданский предмет. Я даже не знаю, что развалится при такой перегрузке. Возможно. какие-то манипуляторы раскладывающие солнечные батареи, если говорить про спутник.

Далее вопрос-откуда взялось 3джи. Вот 5 по мерлинов по 82 тс в вакууме тащат 115т вторую ступень, 23т первую, 7т топлива, тонну обтекателей, 15тонн ПН с бОльшей перегрузкой. Они что, штатно заканчивают работу первой ступени на 5 движках? Потому что 7 движков дадут уже более 3.5джи. Чем больше движков-тем меньше гравитационных потерь. Потом остаётся один движок. Если ПН имеет массую 10тонн, то 80тс тяги пустотного Мерлина дадут больше 5джи ускорения, так как пустая ступень весит около 5т, а обтекателей уже нет. То есть движок снова надо дросселировать. И с дросселированием падает УИ, например, ну и оно имеет свои пределы.
Аноним 14/12/22 Срд 21:22:09 737017 490
>>736927
>А между плотноприжатыми фотонами что?
Идеальная жидкость.
Пустота невозможна в пространстве - обязательно должен быть физический объект с размерностью.
Это примерно как с жестким диском - где свободное пространство забито нулями. Да и любая трехмерная программа задает рабочую виртуальную трехмерную область по тому же принципу.
Наш Мир тоже виртуален. Только определяющими факторами для выделения "постоянной" частицы из первоматерии является наличие осевого вращения у некоторого объема первоматерии - то есть замкнутого движения. Потому что всё взаимодействие между простейшими частицами из первоматерии происходит чисто на геометрическом уровне - трения нет. А значит частицу с осевым вращением (спином) невозможно остановить. А все полости между плотноприжатыми фотонами заполняет идеальная жидкость из первоматерии - которая всегда заполняет весь предоставленный ей объем - путем бесконечного деления на частицы без осевого вращения и их последующего слияния. На самом деле это по факту чисто вычислительный процесс в рамках нашего виртуального Мира. Но осознание наличия первоматерии между плотноприжатыми фотонами важно - потому что при определенных условиях (когда некий объем приобретает осевое или замкнутое (кольцевое и тд) вращение) могут появляться новые "постоянные" частицы. И они появляются - я выше писал о рождении винтоообразных частиц на полюсах Солнца, в тч тех, которые называют нейтрино. Да все частицы с дробным спином - те самые винтообразные буравчики, которые навинчиваются между плотноприжатыми фотонами и всегда должны двигаться вперед (опять же трения на этом уровне нет)
Аноним 14/12/22 Срд 21:52:54 737020 491
Поясните, я чёт нихуя не понимаю. Заявили что вроде достигнули самоподдерживающейся ядерной реакции. Это типа как постоянно взрывающаяся атомная бомба без внешнего раздражителя?
В общем ничего не понимаю, поясните важность данной йобы и вообще.
Аноним 14/12/22 Срд 22:13:53 737022 492
image.png 689Кб, 780x438
780x438
>>737020
термоядерной
>постоянно взрывающаяся атомная бомба
термоядерная
взрыв тут неверное понятие, просто в бомбе энергия выделяется моментально, что дает взрыв, а в реакторе постепенно
>без внешнего раздражителя
нет, этот реактор работает на лазерах
суть достижения в том что энергия в процессе была больше, чем затрачено на создание и поддержание реакции
Аноним 14/12/22 Срд 22:30:28 737025 493
>>737020
>Заявили что вроде достигнули
Достигли или не достигли - это ещё вопрос. По их же словам датчики сломались, замеры требуют проверок и тд.
Аноним 15/12/22 Чтв 00:04:30 737042 494
>>737020
>Заявили что вроде достигнули самоподдерживающейся ядерной реакции. Это типа как постоянно взрывающаяся атомная бомба без внешнего раздражителя?
Нет. Просто в NIF получили выход энергии на 20% больше, чем затратили. Это как очень маленькая водородная бомба, которая сама по себе не взрывается, а требует ДОПИЗДИЩИ энергии, чтобы её лазерами обжать. При этом выделяется ЧУТЬ БОЛЕЕ ДОПИЗДИЩИ энергии. На эту разницу и живём - при условии, что разница покрывает все затраты на кпд и всю хуйню.
Аноним 15/12/22 Чтв 00:07:37 737043 495
>>737042
А разве там лазер не нужен только для того чтобы запустить реакцию?
Аноним 15/12/22 Чтв 00:08:38 737044 496
>>735673 (OP)
Скажите, пожалуйста, может ли такое быть, что крупный объект, столкнувшись к космосе в иным объектом, развалится на несколько частей, и они все обрушатся на одну планету?
Аноним 15/12/22 Чтв 01:52:26 737051 497
>>737005
Смотри на Титан.
Полетай в Space Engine и выбери пару планет что понравятся, напиши их характеристики здеся и спроси почему не взлетит.
Аноним 15/12/22 Чтв 02:10:49 737053 498
image.png 52Кб, 701x512
701x512
image.png 16Кб, 967x142
967x142
>>737016
>Тренированные люди в ППК и креслах управляют самолётом при 14 джи. Сколько нужно, чтоб убить-ХЗ, но двадцатки мало точно. В любом случае у меня перегрузка 5.
Не знаю где ты вычитал про 14g, я как видел так и вижу максимально разрешенные перегрузки в 9-10g.
>Значит выдержат тем более.
Нет, ограничение стоит как раз потому что уже даже тренированные начинают отбраковываться. Ты буквально можешь убить огурцов спасая их. Не из-за экономии на топливе не ставят САС на ракетах помощнее.
>Повернуть тягу после отделения. Далее. Есть некая мысль, что ракету лучше швырять катапультой не вертикально вверх, а под небольшим углом к вертикали. Тут следующие плюсы
САС и так отворачивает от ракеты, но все это время примерно огурцам ускорение в спину идет.
>-меньше разворачивать потом, сразу разгоняется на восток
Экономия на САС за счет увеличения горизонтальной перегрузки и усложнения САС? Боковые катапульты усложнят саму капсулу и добавят точку отказа. Катапульта утаскивающая всю капсулу вбок усложнит САС и добавит асимметрию.
Запили-ка САС вбок в огурцаче.
>-если что-шлёпнется мимо комплекса
Оно и так.
>-удобнее отстреливать САС
Оно и так.
>Но из минусов то, что для этого нужно немного иначе к самой ракете подходить, хотя я думаю крен в 6 градусов ничего страшного не сделает
Запили-ка САС вбок в огурцаче, любопытно как ты это представляешь. Для сранвения сперва нормальную САС, потом модифицированную по твоим заявкам, желательно.
>Так и самолёт многоразовый пока не разобъётся.
При чем тут самолеты? Самолеты дешевле аэропортов, и не уничтожают их при крушении.
>Далее вопрос, насколько этот комплекс будет устойчив к взрыву.
Даже если очень - его перебирать надо потому что ты не знаешь где что шрапнелью посечь могло, и можешь получить не отделяющийся зажим который не отпустит ракету в нужный момент.
>Потому что если сделать что-то вроде тянущего тросса, переброшеного через блок, то некая уязвимость имеется (если противовес и блоки близко от ракеты). Если же делать катапульту иначе, то я вижу варианты такой конструкции, которую даже взрывом ракеты не особо прошибёшь.
Если сделать просто и неломаемо то охуенная затея, не спорю!
>Примерно как мостовой кран на 10кт.
Ха-ха-ха, нет. В сто раз дороже МИНИМУМ.
Примерно как мостовой кран на 10кт будет стоить мостовой кран на 10кт.
>Тут вопрос: что конкретно в спутнике расчитано на такую перегрузку, потому что 3 джи это что-то вроде большого ухаба при езде на машине. То есть эту перегрузку выдержит практически любой гражданский предмет. Я даже не знаю, что развалится при такой перегрузке. Возможно. какие-то манипуляторы раскладывающие солнечные батареи, если говорить про спутник.
Проблема со спутниками в том что там что угодно может развалиться при такой перегрузке и ты ничего с этим не поделаешь после запуска. А т.к. у них каждый грамм на счету ты не можешь поставить стальные фермы и петли ШОБ НАДЁЖНА.
Если ты сделаешь РКН которая выходит с 5g, готовься значительно снижать цену на килограмм, потому что спутники придется делать крепче и тяжелее чем раньше.
>Далее вопрос-откуда взялось 3джи. Вот 5 по мерлинов по 82 тс в вакууме тащат 115т вторую ступень, 23т первую, 7т топлива, тонну обтекателей, 15тонн ПН с бОльшей перегрузкой. Они что, штатно заканчивают работу первой ступени на 5 движках? Потому что 7 движков дадут уже более 3.5джи. Чем больше движков-тем меньше гравитационных потерь. Потом остаётся один движок. Если ПН имеет массую 10тонн, то 80тс тяги пустотного Мерлина дадут больше 5джи ускорения, так как пустая ступень весит около 5т, а обтекателей уже нет. То есть движок снова надо дросселировать. И с дросселированием падает УИ, например, ну и оно имеет свои пределы.
Дросселируют же, да.
Аноним 15/12/22 Чтв 02:17:06 737054 499
image.png 25Кб, 334x251
334x251
image.png 259Кб, 1611x971
1611x971
p7gvpka7z2631.gif 5631Кб, 600x338
600x338
Аноним 15/12/22 Чтв 05:17:03 737055 500
image.png 323Кб, 512x640
512x640
Почему этот кот такой довольный эти линии такие вытянутые и прижатые друг к другу, разве не должны они веером разлетаться или петлями изгибаться падая обратно на слонце?
Аноним 15/12/22 Чтв 10:18:52 737058 501
>>737055
Ну вот там у самой поверхности петли видно.
Аноним 15/12/22 Чтв 12:42:51 737087 502
image.png 85Кб, 796x721
796x721
>>737053
>Не знаю где ты вычитал про 14g

У истребителей более 9 джи не бывает.

>Ты буквально можешь убить огурцов спасая их.

Ещё раз: тренированный пилот при 9 джи гарантированно может управлять самолётом. Он даже может управлять! То есть до потери сознания ещё далеко. А от потери сознания до смерти ещё дальше. В твоей же табличке: 46.2джи для человека на ракетных салазках. Был чел, на себе эксперименты ставил по перегрузке, 46.2 легко. При ударе в автомобиле бывает и больше и тогда бывают неприятные вещи: синяки, даже сломанные рёбра. При 80 джи.

>Боковые катапульты усложнят саму капсулу и добавят точку отказа.

Просто под углом РДТТ.

>При чем тут самолеты? Самолеты

Многоразовые, но если разобъются, то всё.

>Даже если очень - его перебирать надо потому что ты не знаешь где что шрапнелью посечь могло, и можешь получить не отделяющийся зажим

Это часть в любом случае под списание в случае взрыва. Но зажимы на ведущем устройстве, а сама катапульта это ещё куча систем обеспечивающих саму силу. Ведущее устройство это только часть катапульты.

>Если сделать просто и неломаемо то охуенная затея, не спорю!

Поплавок в шахте, который толкает ракету.

>В сто раз дороже МИНИМУМ.

Я думаю дешевле.

>Примерно как мостовой кран на 10кт будет стоить мостовой кран на 10кт.

Я решил, что это слишком сложно и дорого.

>в том что там что угодно может развалиться при такой перегрузке

Не видел такой информации.

>Дросселируют же, да.

Пруфца бы.
Аноним 15/12/22 Чтв 12:55:44 737089 503
>>737087
>У истребителей более 9 джи не бывает.
Примерно так, да. 9-10 г, пилот ловит потемнение зрения и потом теряет сознание.
>Ещё раз: тренированный пилот при 9 джи гарантированно может управлять самолётом. Он даже может управлять! То есть до потери сознания ещё далеко. А от потери сознания до смерти ещё дальше. В твоей же табличке: 46.2джи для человека на ракетных салазках. Был чел, на себе эксперименты ставил по перегрузке, 46.2 легко. При ударе в автомобиле бывает и больше и тогда бывают неприятные вещи: синяки, даже сломанные рёбра. При 80 джи.
К чему ты это споришь? Вообще был человек который 200g пережил, может ты и 200g предложишь ставить?
Единичные случаи не исправляют общее положение. Люди поумнее нас с тобой поставили ограничения на перегрузки потому что при больших перегрузках буквально есть риск убить человека.
>Просто под углом РДТТ.
Нет, не просто. Точку подвеса РДТТ под каждый запуск надо менять из-за развесовки, иначе получишь закручивание.
>Многоразовые, но если разобъются, то всё.
При чем тут самолеты? Они не стартовый комплекс.
>Это часть в любом случае под списание в случае взрыва. Но зажимы на ведущем устройстве, а сама катапульта это ещё куча систем обеспечивающих саму силу. Ведущее устройство это только часть катапульты.
Эта часть в любом случае на обслуживание даже без взрыва. Что удорожает эксплуатацию.
А в случае взрыва перебираешь вообще все узлы стартового комплекса без исключения.
>Поплавок в шахте, который толкает ракету.
Может без шахты обойдемся? В океане балластом подтопим, отцепим, она архимедовой силой ускорится?
>Я думаю дешевле.
Ты забываешь про бульон.
Посмотри и почитай бюджеты разных вещей связанных с комической отраслью, тогда может поймешь почему я говорю что делаю скидку когда говорю что кран всего лишь в 100 раз дороже портового будет.
>Не видел такой информации.
Ну наверное потому что спутники запиливают под расчетные перегрузки и проверяют перед запуском?
Аноним 15/12/22 Чтв 13:15:20 737091 504
>>737089
> Вообще был человек который 200g пережил
У меня и сейчас Athlon 200g, хули тут такого.
Аноним 15/12/22 Чтв 13:44:44 737096 505
>>737089
>при больших перегрузках буквально есть риск убить человека.

Да, более 40 для космонавта.

>Вообще был человек который 200g пережил, может ты и 200g предложишь ставить?

Нет. Это ты предлагаешь ставить 3, заверяя что 10 убъют. А даже 200 не всегда убивают.

>закручивание

Ну вот, закручивание и развернёт капсулу. Причём смотря как вообще эта САС будет реализована, тянущий РДТТ это не единственный вариант. Далее вопрос: какова обязательность включать САС в те 2 секунды, которые ракету разгоняет катапульта?

>При чем тут самолеты? Они не стартовый комплекс.

Они многоразовые.

>Эта часть в любом случае на обслуживание даже без взрыва. Что удорожает эксплуатацию.

Вообще многоразовая сложная система это плохо. Проще всего твердотопливная петарда.

>Посмотри и почитай бюджеты разных вещей связанных с комической отраслью

Falcon-9, пустой, 29 тонн обе ступени с обтекателем, стоимость около 90млн длр, 3 млн долларов за тонну.
Боинг-787, пустой, 120т, 300млн долларов, цена 2.5млн долларов за тонну.
При этом Дримлайнеров более 1000штук выпущено, гораздо больше, чем Фэлконов.
Космодром Восточный-180млрд руб.
Зенит-Арена 60млрд руб.

В чём дороговизна-не вижу, если честно. Самые обычные цены.

>В океане

Если пусковая платформа изначально в океане-то да.
Но в океане соль и погода.

>Ну наверное потому что спутники запиливают под расчетные перегрузки и проверяют перед запуском?

Ну покажи где 3джи увидел для спутника.
Аноним 15/12/22 Чтв 13:57:16 737097 506
>>737096
>Да, более 40 для космонавта.
>Нет. Это ты предлагаешь ставить 3, заверяя что 10 убъют. А даже 200 не всегда убивают.
Не заверяя что убьют, а говоря о риске. Риски минимизируют, ты не огурцов запускаешь.
>Ну вот, закручивание и развернёт капсулу.
И въебет обратно в ракету, отличный план.
>Причём смотря как вообще эта САС будет реализована, тянущий РДТТ это не единственный вариант.
Ну придумай еще САС вместо работающего.
>Далее вопрос: какова обязательность включать САС в те 2 секунды, которые ракету разгоняет катапульта?
Вот как раз в эти секунды она может быть нужнее всего.
>Они многоразовые.
Non sequitur.
>Вообще многоразовая сложная система это плохо. Проще всего твердотопливная петарда.
Ну да.
>Falcon-9, пустой, 29 тонн обе ступени с обтекателем, стоимость около 90млн длр, 3 млн долларов за тонну.
Это ракета.
>Боинг-787, пустой, 120т, 300млн долларов, цена 2.5млн долларов за тонну.
Это вообще самолет.
>При этом Дримлайнеров более 1000штук выпущено, гораздо больше, чем Фэлконов.
Это вообще самолет.
>Космодром Восточный-180млрд руб.
Это три унитаза и башня сборки.
>Зенит-Арена 60млрд руб.
Это стадион.
>В чём дороговизна-не вижу, если честно. Самые обычные цены.
У тебя нерилейтед примеры. Попробуй еще раз не прикидываясь долбоебом.
>Если пусковая платформа изначально в океане-то да.
>Но в океане соль и погода.
Похуй на соль для однораза.
>Ну покажи где 3джи увидел для спутника.
Буквально любой спутник.
Аноним 15/12/22 Чтв 16:57:17 737113 507
>>737097
>И въебет обратно в ракету

Нет! Топливо закончится когда капсула будет на нужном курсе.

>Ну придумай еще САС вместо работающего.

Так и РН новые не нужны, Союза хватает для всего.

>Риски минимизируют

Ну вот: 9джи у лётчиков, 5 на катапульте.

>Ну да.

Так зачем же тогда все эти наркоманские ЖРД делают?

>Вот как раз в эти секунды она может быть нужнее всего.

А может и не быть.

Это вообще самолет.

И?

>Это три унитаза и башня сборки.

Ну вот, космодром=3 унитаза и башня сборки. Откуда там высоким ценам-то?

> тебя нерилейтед примеры.

Релейтед.

>Попробуй еще раз не прикидываясь долбоебом.

Это ты прикидываешься.

>Похуй на соль для однораза.

Нет, не похуй. И потом какой вообще смысл в одноразовых ступенях сейчас. Они все придуманы.

>Буквально любой спутник.

Буквально беспруфные фантазии.
Аноним 16/12/22 Птн 07:34:44 737142 508
Несколько вопросов возникли у меня сегодня в ночном бреду, поясняйте:
1. Что будет если что-то сжимать с большой силой? Например остов машины.
Полодить машину в суперпресс и давить на неё со всех сторон с силой сколько бы весил Юпитер например.
2. Что если такое же делать с водой? Где-то слышал что вода не сжимаемая, поэтому происходят гидро разрывы и прочая хуйня.
3. А что если сжать пространство? Т.е. взять объём пространства и давить не давая выхода.
4. Что будет в случае расширения того же самого? Взять объём чего-либо в капсулу, и расширять её.
5. Пуля летил быстро, потому что в дуле давят пороховые газы, так? А как будет пуля лететь, если её например держать за гильзу и тукнуть так? Т.е. без дула.
Аноним 16/12/22 Птн 09:44:42 737152 509
>>733169 →
Не понимаю.
Смотри ситуация, я лечу со скоростью света к звезде которая в двух световых годах от земли. Светит летит туда два земных года, я лечу туда два земных года, на земле проходит два года, у меня проходит два года. Почему работает не так?
Аноним 16/12/22 Птн 10:58:35 737154 510
>>737142
1. ничего не будет. металлы вроде как не меняют свои свойства при сжатии, пока не дойдет до хуйни типа нейтронных звезд - похуй.
2. будут всякие номерные варианты льда. например, в уране и нептуне внутри очень горячий лед, который из-за давления не может "расстаять"
3. будет черная дыра лол. ну или двигатель альбукерке
4. ничего не будет, будет неидеальный вакуум
5. хуево будет лететь, в разы. гильза намного легче пули и вся скорость будет на ней, ну и плюс сама гильза может порваться. то есть это будет просто как будто взрыв рядом с пулей, а не выстрел в привычном понимании. даже если зафиксировать гильзу от движения и разрыва - энергия пули будет очень хуевой
Аноним 16/12/22 Птн 11:42:26 737155 511
каток давит соб[...].webm 218Кб, 360x360, 00:00:11
360x360
>>737142

1. Прессовка из металла. Кубик получается обычно.
2. Вода сжимается плохо. Давление будет расти, потом из всех щелей брызнет.
3. Пространство нельзя вот так взять и сжать. Оно проходит через пресс. Вот будет как не шебм.
4. Давление понизится.
5. Гильза порвётся, пуля полетит плохо, очень медленно. Насколько именно-от патрона зависит.
Аноним 16/12/22 Птн 13:09:07 737156 512
>>737022
Нет. В этом реакторе она тоже выделяется "моментально"(в том же смысле, в каком это слово использовал ты). Только количество "топлива" в синтезе доли или единицы граммов.
Аноним 16/12/22 Птн 14:04:30 737160 513
Допустим что мир в нолановском доводе был бы реален и возможно на одном отрезке времени существование как нормальных так и инвертированных объектов и событий с ними. Рассмотрим нормальную пулю и кусок стекла который в инверсии проходит перед ней. С точки зрения норм мира пуля летит в проходящее на ее пути стекло, касается его и что в этот момент должно происходить? Если стекло в инверсии по идее оно должно восстановиться с точки зрения норм мира, а перед этим быть разбитым, иначе оно тогда бы не было инвертированным. Но в этом случае тогда непонятно куда пуля девается, то ли она должна как-то расплыться в стекле то ли ещё что. Если мы смотрим с точки зрения стекла, тогда весь мир движется в обратном направлении вместе с пулей и тогда стекло наоборот должно разрушаться после того как пуля обратно полетела в ствол после касания. В общем либо я что-то не понимаю, либо такого мира не может существовать, т.к. не может быть таких противоречивых событий, когда причинно следственные связи рвутся в каком-то событии. Что вы думаете по этому поводу?
Аноним 16/12/22 Птн 15:15:28 737164 514
>>737152
>Светит летит туда два земных года, я лечу туда два земных года, на земле проходит два года
Да, для наблюдателя на Земле проходит два года.

>у меня проходит два года
Нет, у тебя не проходит и секунды.
Аноним 16/12/22 Птн 15:57:08 737167 515
>>737142
>А что если сжать пространство? Т.е. взять объём пространства и давить не давая выхода
Черная дыра.
>>737152
>у меня проходит два года
Нихуя подобного. Когда ты вернешься на Землю и сравнишь показания своих часов с показаниями часов земных, то обнаружишь огромную разницу: по земным часам прошло несколько лет как минимум, а по твоим прошло намного меньше. Это релятивистский эффект замедления времени.
Аноним 16/12/22 Птн 18:49:00 737189 516
Я верно понял что если сделают самподдерживающийся что-то там, термояд вроде, то это будет прорыв уровня колеса, когда будет бесконечная энергия?
Кстати, а откуда берётся энергия у термояда-то?
Аноним 16/12/22 Птн 18:59:28 737191 517
Binding-energy-[...].jpg 49Кб, 818x576
818x576
>>737189
>то это будет прорыв уровня колеса, когда будет бесконечная энергия?
Не будет бесконечной энергии, но будет дешёвая. Но прорывы в принципе не работают так, что в одночасье раз и бесплатная энергия, всё будет постепенно.

Речь о ядерном синтезе (он же термояд). Современные реакторы работают на реакциях распада, они не так много энергии дают.

>Кстати, а откуда берётся энергия у термояда-то?
Из энергии связи частиц в ядрах атомов, как и у обычной ядерки, только с другого края графика, где разности энергий гораздо больше.
Аноним 16/12/22 Птн 19:02:47 737193 518
Почему никто не исследует новые методы выведения на орбиту, а вместо этого уже целый век дрочат химические двигатели? Ведь все прекрасно понимают, что уже фактически достигнут физический предел химических двигателей. мертворожденный спинлаунч не считается
Аноним 16/12/22 Птн 19:25:43 737195 519
>>737193
ДОРАХА.
Нет столько нужды на орбиту что-то совать чтобы была ощутимая экономия от альтернативы.
Вон выше ускорительный космодром анон хочет, хотя он окупится только при сотнях запусков.
Аноним 16/12/22 Птн 19:33:13 737197 520
>>737193
>а вместо этого уже целый век дрочат химические двигатели?
Потому что на данный момент это самый практичный метод, лучше пока ничего не придумали.
Аноним 16/12/22 Птн 20:52:00 737212 521
>>737193
>все прекрасно понимают, что уже фактически достигнут физический предел химических двигателей
Потому что ИРЛ это игра с деревом технологий, если ты полностью освоил какую-то технологию то это не значит что скоро должна открыться новая.
Реальных альтернатив нет на данный момент. И даже теоретические методы выглядят непрактичными и нецелесообразными до тех пор пока на начнут строить космические колонии.
Аноним 17/12/22 Суб 01:12:17 737228 522
>>737212
>Реальных альтернатив нет на данный момент.
Как нет? Даже на химии полно.
Спаржа.
Аэроспайки.
Детонационники.
ССТО (скулон)
...
...
...
Металлический водород.
Аноним 17/12/22 Суб 01:17:23 737230 523
>>737228
Всё хуйня с кучей проблем и не летает.
Аноним 17/12/22 Суб 03:48:23 737237 524
>>736931
Ну так месяц для звезды это практически мгновенно.
Что ты вообще не одупляешь, ты понятными словами даже выразить не можешь?
-Медленно догорает
-Да нет же быстро, вот данные.
-Догорает медленно.
-Ну и хуй с тобой сиди тупи дальше.
Аноним 17/12/22 Суб 03:48:49 737238 525
>>736932
Ты просто не подрочил.
Аноним 17/12/22 Суб 03:56:01 737239 526
>>737017
>Идеальная жидкость.
Из чего она состоит?
Аноним 17/12/22 Суб 03:59:12 737240 527
>>737087
>Ещё раз: тренированный пилот при 9 джи гарантированно может управлять самолётом.
Несколько секунд. лол, не позорился бы ты дальше, анон.
Аноним 17/12/22 Суб 04:36:02 737243 528
>>737237
Так месяц а не секунды же.
Аноним 17/12/22 Суб 07:46:05 737254 529
>>737239
Из идеальной жидкости.
Аноним 17/12/22 Суб 10:55:41 737262 530
>>736932
>младший научный сотрудник
Тут в общем-то рано говорить, толковый или нет.
Аноним 17/12/22 Суб 11:39:17 737270 531
>>737262
Это баба, а значит, вероятность, что она представляет из себя что-то путное, меньше. Ведь у баб по понятным причинам хуже получается в естественные науки, чем у мужиков.
Аноним 17/12/22 Суб 11:43:57 737273 532
image.png 117Кб, 670x477
670x477
image.png 81Кб, 609x345
609x345
image.png 122Кб, 742x1041
742x1041
image.png 43Кб, 740x521
740x521
>>737240

Да, время виража. Например летел самолёт в одну сторону со скоростью 400м/с, нужно повернуть назад, то есть набрать 400-(-400)=800м/с. 80с минимум, интуитивно чую что должно быть 40Пи секунд. А катапульта это только 100м/с при 5г, то есть 2с. И САС тоже не 20 секунд работает.

>не позорился бы ты дальше, анон.

Так где пруфы-то на 3г у спутника? У старлинка, джеймс уэбба, крю дрэгона? Пока я вижу только твои фантазии и твой позор. Пруфов на дросселирование двигателей у Фэлкона на взлёте нет. Твои фантазии про то что ракета специально медленно взлетает чтоб аэродинамическое сопротивление меньше было это отдельный лул.
3-й это из мануала к Фэлкону, 6джи максимальное осевое ускорение.
4-й тоже, только для пустой ракеты. Сколько там ускорение в состоянии разглядеть? А перегрузка больше ускорение на 1 как минимум.
Для того что бы умничать, надо быть умным, иначе хуйня получается.
Аноним 17/12/22 Суб 12:01:25 737274 533
>>735673 (OP)
Как объекты отражают свет, если длинна волны больше атома?
Аноним 17/12/22 Суб 12:23:11 737282 534
>>737228
>Спаржа.
В смысле с перекачкой топлива из боковушек в центр? Ну такое, не сильно влияет на общую картину.
>Аэроспайки.
Там вообще копеечная разница в УИ
>Детонационники
Это да, но вроде как разница чисто в цене двигателя выходит, сами параметры не сильно лучше
>ССТО (скулон)
ССТО заебись только на планете типа Кербина, на Земле это хуета
>Металлический водород
Это уже ближе к фантастике
Аноним 17/12/22 Суб 13:02:28 737285 535
>>737193

Потому что как не крути-всё равно должен быть ракетный двигатель.
Лучшее, что сейчас можно сделать-сделать ракету полностью многоразовой, чтоб все ступени возвращались.
Аноним 17/12/22 Суб 13:22:29 737286 536
>>737195
>Нет столько нужды на орбиту что-то совать
Почему? Вот например навскидку: орбитальный коллайдер. Вокруг планеты. Или та же лунная база. Нужно очень дохуя массы выводить.
Туристический космос.
>Потому что как не крути-всё равно должен быть ракетный двигатель.
С хуя ли?
>Лучшее, что сейчас можно сделать-сделать ракету полностью многоразовой, чтоб все ступени возвращались.
До полноценного ядерного двигателя всего несколько шагов.
Аноним 17/12/22 Суб 13:35:21 737287 537
>>737286
>орбитальный коллайдер. Вокруг планеты. Или та же лунная база
До строительства таких объектов нам натурально как до Луны. И дело даже не в необходимости выводить такие массы- у нас банального сопромата нет, которым можно было бы хуйню такого размера посчитать.
Аноним 17/12/22 Суб 13:41:35 737288 538
>>737286
>Вот например навскидку: орбитальный коллайдер. Вокруг планеты.

Ты представляешь сколько стоить будет такой коллайдер, даже если запуск будет полностью бесплатным?
Аноним 17/12/22 Суб 15:00:37 737289 539
>>737288
Дохуя. Только при нынешней стоимости вывода грузов, общая стоимость увеличивается раз в 5.
Аноним 17/12/22 Суб 15:24:10 737291 540
>>737287
пчел, там небоскребы километровые строят.
арабы вон вообще на 200-километровый гигахрущ замахнулись.
Аноним 17/12/22 Суб 17:07:19 737299 541
>>737291
>пчел, там небоскребы километровые строят.
Между километровым небоскребом и орбитальным коллайдером разница в длине в 4 порядка.
>арабы вон вообще на 200-километровый гигахрущ замахнулись.
Вот когда построят, тогда и поговорим.
Аноним 17/12/22 Суб 17:30:44 737301 542
>>737299
>Между километровым небоскребом и орбитальным коллайдером разница в длине в 4 порядка.
По толщине тоже разница большая, но в обратную сторону. А еще на орбите похуй на вес, т.к. не нужен охуительный фундамент и т.д.
Аноним 17/12/22 Суб 18:21:15 737306 543
>>737289
>Дохуя.

Короче современный коллайдер длинной 27км, не менее 6млрд евро. Коллайдер по периметру Земли будет больше в 13тыс раз и его ориентировочная стоимость 10трлн евро. То есть как 6 программ Ф-35.
Аноним 17/12/22 Суб 20:42:53 737314 544
Кстати, а нужен ли в космосе полноценный коллайдер? Сегментированный не пойдет?
Аноним 17/12/22 Суб 22:47:30 737328 545
>>735673 (OP)
Может ли случится, что от столкновения планеты с большим обьемом антиматерии, в коре планеты образуются залежы радиоактивных сверхдолгоживущих изотопов?
Аноним 18/12/22 Вск 00:31:43 737337 546
image.png 69Кб, 224x225
224x225
>>737306
> То есть как 6 программ Ф-35
Обкекался почему-то.
Аноним 18/12/22 Вск 01:36:31 737343 547
Аноним 18/12/22 Вск 02:04:48 737344 548
Аноним 18/12/22 Вск 07:27:48 737348 549
>>737301
>на орбите похуй на вес, т.к. не нужен охуительный фундамент
Не похуй, если у тебя конструкция сворачивается в бублик вокруг планеты, и от падения удерживается только упиранием в само себя, он сам для себя фундамент.
Аноним 18/12/22 Вск 07:31:12 737349 550
>>737314
А нужен ли вообще новый коллайдер? Какие ещё там частицы они не нашли, которые не получить в текущих коллайдерах?
Аноним 18/12/22 Вск 08:26:19 737350 551
>>735673 (OP)
Роскосмос и сатанаса реально решили прекратить хуцпить и сливают в унитаз какую-то МКС, которая в беспилотном режиме летает?
Аноним 18/12/22 Вск 09:27:52 737352 552
image.png 831Кб, 1280x720
1280x720
>>737349
Миниатюрную черную дыру.
Wait, oh shi-
Аноним 18/12/22 Вск 10:31:49 737358 553
>>737352
Кстати, а что будет если действительно в коллайдере из-за столкновения двух частиц получится черная дыра? Сколько времени займет коллапс Земли? Секунды, часы, дни или годы?
Аноним 18/12/22 Вск 10:39:59 737360 554
>>737349
Долгоживущие сверхмассивные изотопы.
Аноним 18/12/22 Вск 11:02:39 737361 555
>>737358
ЧД испарится моментально.
Аноним 18/12/22 Вск 11:08:46 737362 556
Аноним 18/12/22 Вск 11:12:53 737364 557
>>737362
Тогда сколько-то лет будет вихляться вокруг барицентра земля-луна медленно отжираясь и экспоненциально увеличивая тектоническую активность земляшки.
Аноним 18/12/22 Вск 11:37:33 737368 558
>>737364
>вокруг барицентра земля-луна
Это же под поверхностью земли. Мне вот интересно как это будет выглядеть с точки зрения просто землянина.
Аноним 18/12/22 Вск 12:11:42 737379 559
Аноним 18/12/22 Вск 12:17:51 737381 560
>>737368
>Это же под поверхностью земли.
Дыа. Там и будет крутиться изредка сталкиваясь с атомами и фоня рентгеном, первые годы активность будет мизерная, будет очень долго отжираться до размера когда будет заметно фонить.
Аноним 18/12/22 Вск 12:31:55 737382 561
>>737358
>>737362
Если ЧД достаточно большая, чтобы не испариться мгновенно (ну то есть вообще, чтобы баланс поглощения-излучения был положительным), то она в первую очередь упадёт на пол и начнёт двигаться в сторону центра Земли, оставляя микроскопическую дырку. Вообще, надо бы конечно оценить её размеры, но мне лень, буду исходить из предположения, что она сравнима с размером атома. Таким образом, первое время она будет отжираться за счёт фотонов, затем последуют электроны и ядра атомов (электроны могут быть раньше, если у дыры положительный заряд), это может занять несколько дней. После этого она станет отъедаться очень быстро, на Земле начнёт увеличиваться тектоническая активность, и в течение пары часов поверхность развалится к хуям и упадёт в дырку, попутно спагеттифицируясь.
Аноним 18/12/22 Вск 12:37:55 737383 562
>>737382
>>737381
>>737364
Говорят что микроскопическая ЧД попав в сраную земляшку будет отжираться так долго что это на порядки дольше возраста вселенной. 1023 лет где-то.
Аноним 18/12/22 Вск 13:01:20 737384 563
>>737383
Ну хуй знает, чот нереалистично, а сурс есть? Вообще, зависит от размеров. Если она размером с атом, то она может эффективно поглощать рентген, а он может испускаться после электронного захвата и других процессов, то есть внутри земли доступен. Если дырка сильно меньше атома, то да, трудно будет.
Аноним 18/12/22 Вск 13:22:20 737386 564
>>737349
Мне тоже кажется, что можно что-то другое придумать намного более эффективное и дешевое. Регистрируют же частицы с колоссальными энергиями, которые попадая в атмосферу дают целый дождь разных частиц. Вот надо выяснить откуда они летят и просто регистрировать их попадание, когда они попадут в зону действия детекторов. А если две такие частицы встретятся? Таких энергий ни на одном коллайдере не получить.
Аноним 18/12/22 Вск 13:29:15 737387 565
>Подобные частицы с очень высокой энергией весьма редки. Обычно энергия частиц космических лучей находится в диапазоне от 10 МэВ до 10 ГэВ. Более новые исследования с помощью Telescope Array project (англ.) позволили предположить, что источник этих частиц находится в 20-градусной зоне в направлении созвездия Большой Медведицы

Что если всю тряхомудию с коллайдера без ускорителя разместить где-нибудь типа там где вебб и направить в эту сторону и просто ждать?
Аноним 18/12/22 Вск 13:39:58 737388 566
>>737306
>То есть как 6 программ Ф-35.
И что здесь нереализуемого? США даже в одно рыло потянет в принципе.
Аноним 18/12/22 Вск 13:42:22 737389 567
>>737349
>А нужен ли вообще новый коллайдер?
>Какие ещё там частицы они не нашли, которые не получить в текущих коллайдерах?
Глюоны. Нужно открыть теорию ВСЕГО, чтобы объединить гравитацию с остальными фундаментальными силами.
Правда для этого нужен коллайдер с орбитой чуть больше орбиты Плутона, чтобы получить необходимую энергию частиц.
Аноним 18/12/22 Вск 13:45:39 737391 568
photo17736@17-0[...].jpg 790Кб, 1114x1280
1114x1280
>>736603
>Так какая разница, итог один — горный хребет, слишком высокий для гравитации данной планеты. Он осыпется и станет более низким и пологим. Асло, куда ты землю из ям девать собрался?
Вопрос, конечно, тупой (но и тред ведь для этого), но ведь есть на Марсе гора Олимп. В принципе, если Олимп Ctrl+C и Ctrl+V Олимп по всей планете кольцом, то получится как раз планета-пельмень, и гора эта никоим образом не обрушится, ведь она уже развалилась и заняла стабильную форму кучи, которая, тем не менее, без малого 30км в высоту
Так почему же гора-кольцо не возможна и не поулчится планета-пельмень?
Аноним 18/12/22 Вск 13:45:41 737392 569
>>737387
Нихуя не будет, потому что чем больше энергия частицы, тем меньше сечение взаимодействия. Чтобы поймать такую частицу нужен столб плотного вещества хотя бы воды длиной несколько десятков километров.
>>737386
>Таких энергий ни на одном коллайдере не получить.
Нихуя, сейчас получают энергии значительно больше тех, которые вылетают из нейтронных звезд и всяких магнетаров.
Аноним 18/12/22 Вск 13:51:59 737393 570
>>737391
Высота Олимпа двадцать с хвостиком километров. По сравнению с диаметром планеты это легкая шерховатость.
Аноним 18/12/22 Вск 13:55:17 737395 571
>>737393
26,5км это ближе к 30, чем к 20
Аноним 18/12/22 Вск 13:56:56 737397 572
>>737395
Шерховатость с легкой шерховинкой.
Аноним 18/12/22 Вск 14:01:40 737399 573
>>737392
Бак - 13,6 ТэВ
Частицы ОМГ 3⋅10^8 ТэВ
Аноним 18/12/22 Вск 14:07:49 737404 574
>>737397
Ну а ты что хочешь? Гору по экватору на 1/3 радиуса общей средней по планете?
Аноним 18/12/22 Вск 14:14:15 737405 575
>>737404
Ну так в этом и есть смысл планеты-пельменя, а длиннющие горные хребты и на Земле есть.
Аноним 18/12/22 Вск 14:15:42 737406 576
>>737391
Ну так-то блин можно вообще Марс назвать планетой-грушей, у него северное полушарие ниже южного.
Аноним 18/12/22 Вск 14:29:52 737408 577
>>737406
Если уж так, тогда и у нас на Земле так же, ибо одно полушаерие это типа как широкая с впуклостью у полюса, а другое полушарие - типа как горка с выпуклостью на полюсе
Аноним 18/12/22 Вск 15:48:27 737416 578
>>737328
От такого столкновения никакой планеты не останется.
>>737389
>Глюоны
Глюоны уже давно открыты, долбоебина.
>>737349
>Какие ещё там частицы они не нашли, которые не получить в текущих коллайдерах?
Суперсимметрию не нашли. Вообще сейчас в физике элементарных частиц настало такое время, когда никто не знает, что искать. Все хотят просто найти новую физику, хоть какую-то. Короче, движение вслепую.
Аноним 18/12/22 Вск 17:22:51 737422 579
>>737389
А что если я тебе скажу что гравитация это не фундаментальное взаимодействие и взаимодействий два три? Ядерное и электрослабое Сильное, электромагнитное и слабое.
Аноним 18/12/22 Вск 17:23:45 737423 580
>>737391
Олимп шире чем выше, с подножия не видно вершину, с вершины не видно то что не является склонами горы.
Аноним 18/12/22 Вск 18:56:01 737431 581
image.png 2185Кб, 1920x1080
1920x1080
Как должно быть расположены планеты в некоей системе, если нужно добиться, чтобы у планеты, на которой находится наблюдатель, всегда на небе солнечное затмение?
Аноним 18/12/22 Вск 19:08:09 737432 582
>>737416
>Глюоны уже давно открыты, долбоебина.
Проебался, я имел в виду гравитоны.
Аноним 18/12/22 Вск 19:21:28 737434 583
>>737432
Гравитоны не то что коллайдеры следующего поколения не откроют - их вообще никогда не откроют, скорее всего. И даже не понятно, есть ли они или их нет. Очень похоже это на маняфантазию моченых.
Аноним 18/12/22 Вск 19:36:20 737437 584
275px-Galileanm[...].gif 1973Кб, 275x185
275x185
image.png 764Кб, 2743x2400
2743x2400
image.png 189Кб, 655x346
655x346
>>737431
Это невозможно при обычных планетах или лунах, даже если их движение связано
Функцию затмения может выполнять искусственная "шторка" в Л1
Аноним 18/12/22 Вск 19:38:25 737438 585
>>737434
Какая маняфантазия? Сейчас никаких работ по гравитонам нет и не предвидится, это такая же идея как червоточины или пузырь альбукерке - на бумаге почиркать да попиздеть друг с другом, ничего серьезного в отличие от Хиггса.
Аноним 18/12/22 Вск 19:41:52 737439 586
>>737438
>Какая маняфантазия? Сейчас никаких работ по гравитонам нет и не предвидится
Это и называется "маняфантазия". Никакой теории нет, никаких экспериментальных данных нет. Есть только вера в то, что типа гравитоны должны быть. А почему должны быть, как - хуй знает. Ну гравитация же должна квантоваться, значит гравитоны точно есть. А то, что мы понятия не имеем о том, как гравитация квантуется, у нас нет даже наметков теории квантовой гравитации - об этом моченые, которые любят попиздеть про гравитоны, обычно забывают.
Аноним 18/12/22 Вск 19:44:06 737440 587
>>737439
Ну а какой смысл об этом говорить, это как ученый изнасиловал журналиста.
Аноним 18/12/22 Вск 19:47:44 737444 588
>>737440
Какой смысл говорить про гравитоны в контексте строительства коллайдеров следующего поколения?
Аноним 18/12/22 Вск 19:52:14 737445 589
>>737444
Никакого, гравитоны хуйня.
Аноним 18/12/22 Вск 19:54:54 737446 590
>>737437
А если орбита "Тундра"?
Тогда планета на орбите "Тундра" будет луной вечного затмения над определённой областью, нет?
Аноним 18/12/22 Вск 19:57:23 737447 591
Аноним 18/12/22 Вск 19:58:38 737448 592
>>737437
А если большие порядки разницы масс? Или расстояния ебически другие?
Скажем Гигагаз крутится близко около звезды за пару дней, а у него в Л2 висит луняша которая в итоге в тени?
Аноним 18/12/22 Вск 20:32:16 737453 593
image.png 211Кб, 602x585
602x585
>>737446
Там слишком сильное отклонение же, будет ебанутое мелькание
>>737448
В точках Лагранжа не может быть объектов с достаточной массой, чтобы стать шарообразными и так далее
Более того, у Юпитера в Л2 почти ничего нет
Аноним 18/12/22 Вск 20:43:29 737455 594
>>737453
>Более того, у Юпитера в Л2 почти ничего нет
Это потому что у тебя Юпитер на орбите Юпитера, а за Юпиетром вообще его ближайший конкурент Сатурн. А вот если бы у нас был Юпитер на орбите Меркурия или Венеры?
Аноним 18/12/22 Вск 21:02:41 737456 595
>>737455
Похуй, в Л2 планет до Юпитера тоже ничего нет. Только в поясе астероидов есть Церера, все остальное круглое в ебенях.
Аноним 18/12/22 Вск 21:47:13 737458 596
Поясните, каким образом ракеты наводятся по gps за тысячу километров от места пуска?
Аноним 18/12/22 Вск 22:45:08 737462 597
image.png 1900Кб, 1759x862
1759x862
Бля, хуле Европа так далеко от юпитера. Я думал там юпитер пол неба занимает. А по факту хуйня.
Аноним 18/12/22 Вск 22:55:01 737463 598
>>737462
Метиду глянь, вот у ней Юпитер нормально так на небе.
А вообще когда у тебя Юпитер на полнеба твоей луны - ты в пределе Роша и твоя луна в данный момент распидорашивается.
Аноним 19/12/22 Пнд 04:08:59 737478 599
>>737458
Ну вы там в ГРУ совсем обленились.
Аноним 19/12/22 Пнд 08:02:24 737481 600
>>737462
Не так красиво, как в Аватаре, да.
Аноним 19/12/22 Пнд 09:39:26 737484 601
>>737463
>и твоя луна в данный момент распидорашивается.
А если это 200-метровый астероид?
Аноним 19/12/22 Пнд 09:43:26 737485 602
image.png 1587Кб, 1920x1080
1920x1080
>>737481
Не смотрел аватар, просто в игре охуенно выглядит.
Аноним 19/12/22 Пнд 09:48:38 737487 603
>>737484
Тогда не распидорашивается.
Аноним 19/12/22 Пнд 10:10:11 737490 604
>>737458

Точно так же как и ты. От ГСП ракета откуда она летит и куда. От гироскопов знает в какую сторону направленна. После чего остается лишь подруливать чтоб не сбиваться с курса.
Аноним 19/12/22 Пнд 11:33:23 737499 605
Влияет ли угол падения метеорита на мощность взрыва и насколько сильно (при одинаковой скорости столкновения)? На глазок-то кажется, что при почти касательном столкновении контакт сильно размажется по времени, да ещё куча говен по инерции в том же направлении улетят, так что часть импульса обратно в космос уйдёт. А при лобовом ебанёт как следует, мощнейше. Динозаврам как бы лучше было?
А гамма-всплеск от столкновения нейтронных звёзд лоб в лоб намного сильнее будет, чем после долгих танцев вокруг?
Аноним 19/12/22 Пнд 13:03:05 737510 606
>>737499
Энергия все равно выделится, хоть при трении об атмосферу, хоть при ударе о твердь. Вон тунгусский и челябинский метеориты даже до земли не долетели, а ебануло так что Новиграде огни было видать.
Аноним 19/12/22 Пнд 14:14:32 737517 607
>>737485
Игра так себе, а виды шикарные.
Аноним 19/12/22 Пнд 16:19:06 737522 608
>>737510
> Вон тунгусский и челябинский метеориты даже до земли не долетели, а ебануло так что Новиграде огни было видать.
А если б долетели под прямым углом, ещё лучше бы ёбнуло ведь? Как минимум оттого, что метеорит не успел бы перегреться, сгореть и взорваться на высоте, а приложился бы как селдует.
Аноним 19/12/22 Пнд 18:11:44 737534 609
>>737517
>Игра так себе,
Не понимаю хейта, по мне так топовая часть.
Аноним 19/12/22 Пнд 18:19:29 737536 610
>>737490
Во-первых гсп не точное, там точность порядка 10 метров. Во-вторых ракета должна не пиздануться об какую-нибудь гору или другую хуйню по дороге. В третьих она должна за километр до цели или типа того начать снижаться чтобы не пролететь цель, опять же у гпс не точное определение высоты, а значит попадание сильно зависит от угла под которым ракета падает к земле.
Ну и в четвертых где гарантия, что цель не съебалась пока ракета летела? Даже если спутниковые снимки свежие, полет ракеты до цели это немало времени.
Очень много факторов должно сложиться, чтобы ракета попала в цель, если её просто "пустили и забыли". Для меня это удивительно. Насколько я понимаю после пуска её уже нельзя корректировать.
Аноним 19/12/22 Пнд 18:48:59 737539 611
А вот все движки цилиндрические - и ракетные и самолетные.
А могут быть они не цилиндрические, в длину например?
Для ракет вроде можно аэроспайку, а может ли воздушный движок линейным быть? Чтоб в крыло запихнуть и во всю ширь ебашил?
Аноним 19/12/22 Пнд 18:53:58 737540 612
>>737539
Нет, турбина как крутиться будет?
Аноним 19/12/22 Пнд 18:56:10 737541 613
image.png 27Кб, 1005x688
1005x688
>>737540
Ну вот как-то так, между двух валов на цепи/ленте.
Аноним 19/12/22 Пнд 19:21:04 737548 614
>>737541
А ещё из буханки хлеба можно сделать троллейбус но нахуя?
Аноним 19/12/22 Пнд 20:59:16 737555 615
>>737536
>Во-первых гсп не точное, там точность порядка 10 метров
Ты не забывай, что есть гражданский, а есть военный gps
Аноним 19/12/22 Пнд 21:02:35 737557 616
16710043402820.mp4 1372Кб, 352x640, 00:00:15
352x640
Аноним 19/12/22 Пнд 21:08:32 737560 617
>>737555
Это тебе в ЦРУ сказали?
Аноним 19/12/22 Пнд 21:10:26 737561 618
Аноним 19/12/22 Пнд 22:11:55 737572 619
>>737536
>у гпс не точное определение высоты
Точное.
Аноним 20/12/22 Втр 00:03:47 737576 620
>>737540
А без турбины никак?
Аноним 20/12/22 Втр 01:50:06 737577 621
>>737358
>а что будет если действительно в коллайдере из-за столкновения двух частиц получится черная дыра?
У них уже такое было несколько раз. Нано ЧД моментально испаряется.
Аноним 20/12/22 Втр 01:54:27 737578 622
123.jpg 9Кб, 222x227
222x227
Аноним 20/12/22 Втр 10:49:00 737591 623
>>737539
Прямоточный вроде как может быть прямоугольным
Аноним 20/12/22 Втр 10:57:22 737592 624
Какой самой низкой температуры может быть жидкая вода?
Аноним 20/12/22 Втр 12:14:30 737598 625
>>737592
Где-то читал что доводили до минус сорока или типа того.
Аноним 20/12/22 Втр 12:27:42 737600 626
>>737536
>там точность порядка 10 метров
Если у тебя полтонны взрывчатки то думаю это не проблема
>не пиздануться об какую-нибудь гору
В ракету можно изначально заложить высоту полета выше предполагаемых препятствий на маршруте, или даже карту местности чтобы ракета меняла высоту.
>снижаться чтобы не пролететь цель
Они вроде у цели резко пикируют
>где гарантия, что цель не съебалась пока ракета летела?
Когда заводы, электростанции и мосты начнут съебываться с места то нужно срочно капитулировать на любых условиях
>Очень много факторов должно сложиться, чтобы ракета попала в цель, если её просто "пустили и забыли"
Фашистские диды запускали свои теплые ламповые ракеты по Лондону и в половине случаев таки попадали. Прогресс не стоит на месте.
Аноним 20/12/22 Втр 12:46:02 737603 627
Waterphasediagr[...].gif 123Кб, 1844x1246
1844x1246
>>737592
По этой диаграмме около 250 К.
Аноним 20/12/22 Втр 12:47:58 737604 628
>>737598
> до минус сорока
По какой шкале?
Аноним 20/12/22 Втр 12:51:33 737605 629
>>737604
Кельвина! Цельсия конечно же
Аноним 20/12/22 Втр 12:58:53 737606 630
>>737536
> Во-первых гсп не точное, там точность порядка 10 метров.
У военного порядка метра.
> Во-вторых ракета должна не пиздануться об какую-нибудь гору или другую хуйню по дороге.
Именно поэтому перед запуском в ракету загружается карта местности. Не бог вест какая новая технология, в томагавках уже в начале 90-х было.
> В третьих она должна за километр до цели или типа того начать снижаться чтобы не пролететь цель, опять же у гпс не точное определение высоты, а значит попадание сильно зависит от угла под которым ракета падает к земле.
Некоторые ракеты пикируют на цель. И даже крылатые бомбы.
> Ну и в четвертых где гарантия, что цель не съебалась пока ракета летела? Даже если спутниковые снимки свежие, полет ракеты до цели это немало времени.
Кто съебался, электростанция?
> Очень много факторов должно сложиться, чтобы ракета попала в цель, если её просто "пустили и забыли". Для меня это удивительно. Насколько я понимаю после пуска её уже нельзя корректировать.
Можно, там есть спутниковый канал связи.
Аноним 20/12/22 Втр 13:02:47 737607 631
>>737603
Ниже, переохлажденная жидкость бывает.
Аноним 20/12/22 Втр 13:05:24 737608 632
Аноним 20/12/22 Втр 14:41:28 737620 633
перекат пилите пидары
Аноним 20/12/22 Втр 17:11:29 737627 634
Аноним 20/12/22 Втр 17:36:50 737630 635
image.png 125Кб, 746x512
746x512
Аноним 20/12/22 Втр 18:45:11 737641 636
>>737600
>Фашистские диды запускали свои теплые ламповые ракеты по Лондону и в половине случаев таки попадали.
Там попадали тупо на похуй. Ебануло в какой-то сарай? Заебись, засчитываем. Просто примерно туда пускали и всё.
Аноним 21/12/22 Срд 02:52:09 737687 637
>>737641
Россия в СВО примерно также запускает. Причем ещё не всё ракеты через границу перелетают.
Аноним 22/12/22 Чтв 18:22:16 737925 638
>>737160
Мой мозжечок не выдержал этого поста, но гугли ответ в вопросе о том, кто построил здание, которое в конце Довода при штурме существовало в целом виде одну секунду, а в остальное время либо его верхняя часть была взорвана, либо нижняя
Аноним 22/12/22 Чтв 18:25:04 737926 639
CEE94AEA-549C-4[...].png 288Кб, 1242x2208
1242x2208
Аноним 22/12/22 Чтв 18:27:36 737927 640
0BA751D5-85A9-4[...].png 355Кб, 1242x2208
1242x2208
>>737926
Сложна нипанятна
Аноним 12/01/23 Чтв 15:16:01 739911 641
Аноны, тут мы получили фото туманности NGC 346 с телескопа ДВ. А много там газа осталось для звездообразвания? Лично на мой взгляд, она как то уже исчерпала себя или так выглядит большинство туманностей? Вообще во вселенной сейчас образуются новые туманности?
Ответить в тред Ответить в тред

Check this out!

Настройки X
Ответить в тред X
15000
Добавить файл/ctrl-v
Стикеры X
Избранное / Топ тредов